{"text": "### Human: Chronic urethral obstruction due to benign prismatic hyperplasia can lead to the following change in kidney parenchyma ### Assistant: Chronic urethral obstruction because of urinary calculi, prostatic hyperophy, tumors, normal pregnancy, tumors, uterine prolapse or functional disorders cause hydronephrosis which by definition is used to describe dilatation of renal pelvis and calculus associated with progressive atrophy of the kidney due to obstruction to the outflow of urine Refer Robbins 7yh\/9,1012,9\/e. P950"} {"text": "### Human: Which vitamin is supplied from only animal source: ### Assistant: Ans. (c) Vitamin B12 Ref: Harrison's 19th ed. P 640* Vitamin B12 (Cobalamin) is synthesized solely by microorganisms.* In humans, the only source for humans is food of animal origin, e.g., meat, fish, and dairy products.* Vegetables, fruits, and other foods of nonanimal origin doesn't contain Vitamin B12 .* Daily requirements of vitamin Bp is about 1-3 pg. Body stores are of the order of 2-3 mg, sufficient for 3-4 years if supplies are completely cut off."} {"text": "### Human: All of the following are surgical options for morbid obesity except - ### Assistant: Ans. is 'd' i.e., Roux en Y Duodenal Bypass Bariatric surgical procedures include:a. Vertical banded gastroplastyb. Adjustable gastric bandingc. Roux-en Y gastric bypass (Not - Roux-en Y Duodenal Bypass)d. Biliopancreatic diversione. Duodenal switcho The surgical treatment of morbid obesity is known as bariatric surgery.o Morbid obesity is defined as body mass index of 35 kg\/m2 or more with obesity related comorbidity, or BMI of 40 kg\/m2 or greater without comorbidity.o Bariatric operations produce weight loss as a result of 2 factors. One is restriction of oralintake. The other is malabsorbtion of ingested food.o Gastric restrictive procedures include Vertical banded gastroplasty & Adjustable gastric bandingo Malabsorbtive procedures include Biliopancreatic diversion, and Duodenal switcho Roux-en Y gastric bypass has features of both restriction and malabsorptionBariatric Operations: Mechanism of ActionRestrictiveVertical banded gastroplastyLaparoscopic adjustable gastric bandingLargely Restrictive\/Mildly MalabsorptiveRoux-en-Y gastric bypassLargely Malabsorptive\/Mildly RestrictiveBiliopancreatic diversionDuodenal switch"} {"text": "### Human: Following endaerectomy on the right common carotid, a patient is found to be blind in the right eye. It is appears that a small thrombus embolized during surgery and lodged in the aery supplying the optic nerve. Which aery would be blocked? ### Assistant: The central aery of the retina is a branch of the ophthalmic aery. It is the sole blood supply to the retina; it has no significant collateral circulation and blockage of this vessel leads to blindness. The branches of this aery are what you view during a fundoscopic exam. Note: The infraorbital aery is a branch of the maxillary aery. It comes through the infraorbital foramen, inferior to the eye. It supplies the maxillary sinus, the maxillary incisors, canine and premolar teeth, and the skin of the cheek below the orbit. The supraorbital aery is another branch of the maxillary aery. It comes through the supraorbital foramen and supplies blood to the muscles, skin and fascia of the forehead. The lacrimal aery is a branch of the ophthalmic aery that supplies the lacrimal gland. The nasociliary aery doesn't exist, but there is a nasociliary nerve that travels with the ophthalmic aery. Ref: Moon D.A., Foreman K.B., Albeine K.H. (2011). Chapter 18. Orbit. In D.A. Moon, K.B. Foreman, K.H. Albeine (Eds), The Big Picture: Gross Anatomy."} {"text": "### Human: Growth hormone has its effect on growth through? ### Assistant: Ans. is 'b' i.e., IGI-1GH has two major functions :-i) Growth of skeletal system :- The growth is mediated by somatomedins (IGF). Increased deposition of cailage (including chondroitin sulfate) and bone with increased proliferation of chondrocytes and osteocytes.ii) Metabolic effects :- Most of the metabolic effects are due to direct action of GH. These include gluconeogenesis, decreased peripheral utilization of glucose (decreased uptake), lipolysis and anabolic effect on proteins."} {"text": "### Human: Scrub typhus is transmitted by: September 2004 ### Assistant: Ans. C i.e. Mite."} {"text": "### Human: Abnormal vascular patterns seen with colposcopy in case of cervical intraepithelial neoplasia are all except ### Assistant: Abnormal vascular pattern include punctation, mosaicism and atypical vessels."} {"text": "### Human: Per rectum examination is not a useful test for diagnosis of ### Assistant: PILONIDAL SINUS\/DISEASE (Jeep Bottom; Driver's Bottom) Pilus--hair; Nidus--nest It is epithelium lined tract, situated sho distance behind the anus, containing hairs and unhealthy diseased granula- tion tissue. It is due to penetration of hairs through the skin into subcutaneous tissue. It forms granuloma\/unhealthy granulation tissue in the deeper plane. It is of infective origin and occurs in sacral region between the buttocks, umbilicus, axilla. Other indications of per rectal examination are:Carcinoma rectum, Stricture rectum, Polyps, BPH and carcinoma prostate, Secondaries in the rectovesical pouch (Blumer shelf), Sphincter tone, Pelvic abscess (is felt as boggy swelling), to feel the internal opening of anal fistulas, in bimanual palpation of the bladder or pelvic tumours, in acute abdominal conditions--it reveals dilated empty rectum with tenderness.Reference : page 967 and 952 SRB's manual of surgery 5th edition."} {"text": "### Human: Characteristics of Remifentanyl \u2013 a) Metabolised by plasma esteraseb) Short half lifec) More potent than Alfentanyld) Dose reduced in hepatic and renal diseasee) Duration of action more than Alfentanyl ### Assistant: Remifentanil is the shortest acting opioid due to its metabolism by plasma esterase \u2192 dose adjustment is not needed in liver or kidney disease. It is more potent than alfentanil : Order of potency is Sufentanil > Fentanyl = Remifentanil > Alfentanil."} {"text": "### Human: Hypomimia is ? ### Assistant: Ans. C. Deficit of expression by gestureHypomimiaHypomimia or amimia is a deficit or absence of expression by gesture or mimicry.This is usually most obvious as a lack of facial expressive mobility (mask - like facies).This is a feature of frontal subcoical disease."} {"text": "### Human: Naglers reaction is shown by ### Assistant: Nagler's reaction - when Clostridium perfringens is grown in media containing 6 % agar, 5% Fildes peptic digest of sheep blood and 20% human serum, with antitoxin spread on one half of plate, colonies in the other half without antitoxin will be surrounded by a zone of opacity. No opacity around the colonies on the half of plate with antitoxin, this is due to the specific neutralization of the alpha-toxin. This specific lecithinase effect, known as the Nagler's reaction. Reference: Textbook of Microbiology; Anathanarayan and paniker's; 10th edition; Page no: 263."} {"text": "### Human: Which of the following statements are True\/False? 1. Hirsutism, which is defined as androgen-dependent excessive male pattern hair growth, affects approximately 25% of women. 2. Virilization refers to a condition in which androgen levels are sufficiently high to cause additional signs and symptoms. 3. Frequently, patients with growth hormone excess (i.e., acromegaly) present with hirsutism. 4. A simple and commonly used method to grade hair growth is the modified scale of Ferriman and Gallwey. 5. Scores above 8 suggest excess androgen-mediated hair growth. ### Assistant: Here statement 1 & 3 are wrong. It is actually a direct pick from harrison, an impoant topic for entrance examinations hirsutism is seen in 10% of woman. Acromegaly rarely causes hirsutism. Ref: Harrisons Principles of Medicine, 18th Edition, Pages 380-382"} {"text": "### Human: The pharmakokinetic change occurring in geriatric patient is due to ### Assistant: One of the most impoant pharmacokinetic changes associated with aging is decreased renal elimination of drugs. After age 40, creatinine clearance decreases an average of 8 mL\/min\/1.73 m2\/decade; however, the age-related decrease varies substantially from person to person. Serum creatinine levels often remain within normal limits despite a decrease in GFR because older adults generally have less muscle mass and are generally less physically active than younger adults and thus produce less creatinine. Maintenance of normal serum creatinine levels can mislead clinicians who assume those levels reflect normal kidney function. Decreases in tubular function with age parallel those in glomerular function. Refer katzung 11\/e p1039"} {"text": "### Human: True regarding lag phase is? ### Assistant: Lag phase: Immediately following the seeding of a culture medium, there is no appreciable number, though there may be an increase in the size of the cells. This initial period is the time required for the adaptation to the new environment, during which the necessary enzymes and metabolic intermediates are built up in adequate quantities for multiplication to proceed. The duration of the lag phase varies with the species, sizes of the inoculation, nature of the culture media and the environmental factors such as temperature REF:Ananthanarayan & Panicker's Textbook of Microbiology 8th edition pg no: 24"} {"text": "### Human: A 60 yr old chronic smoker presents with painless gross hematuria of 1 day duration. Investigation of choice to know the cause of hematuria ### Assistant: Urine microscopy for malignant cytology \"Painless haematuria is by far the most common symptom of bladder cancer and should be regarded as indicative of a bladder carcinoma until proven otherwise.\" Bailey & Love 25\/e p1336 (24\/e p1363) Cigarette smoking is the main etiological factor and accounts ,for about 50% of bladder cancers Among the given options 'urine microscopy' is the best choice for investigating a pt. with high suspicion for bladder ca. This is not a good screening test but a positive result is highly specific. The best investigation would be cystoscopy & biopsy. Investigations for bladder cancer 1. Urinary cytology - Exfoliated cells from both normal and neoplastic urothelium can be readily identified in voided urine. Examination of cytological specimens can detect the malignant cells either at the time of initial presentation or during follow-up. Cytological examination may be especially useful in screening high-risk population and assessing the response to treatment. 2. Cystourethroscopy and tumor resection - The diagnosis and initial staging of bladder cancer is made by cystoscopy and transurethral resection. 3.Imaging (IVP, CT, MRI) - Although various imaging techniques can detect bladder cancer, its presence is confirmed by cystoscopy & biopsy. - Therefore imaging modalities are used to image the upper urinary tract and, when infiltrating bladder tumors are detected to assess the depth of muscle wall infiltration and the presence of local & distant metastasis. - Intravenous urography remains one of the most common imaging tests for the evaluation of hematuria. However, intravenous pyelography is increasingly being replaced by computed tomography (CT) urography, which is more accurate, for evaluation of the entire abdominal cavity, renal parenchyma, and ureters in patients with hematuria. Bladder tumors may be recognized as pedunculated, radiolucent filling defects projecting into the lumen; nonpapillary, infiltrating tumors may result in fixation or flattening of the bladder wall. 4. Urinary tumor markers - - Several new tests have been developed in order to overcome the shocomings of urinary cytology such as the low sensitivity for low-grade superficial tumors and inter-observer variability. - Commercially available tests include, the BTA test, the BTA stat test, the BTA TRAK assay determination of urinary nuclear matrix protein (NMP22), Immunocyt and UroVysion . - These tests can detect cancer specific proteins in urine (BTA\/NMP22) or augment cytology by identifying cell surface or cytogenetic markers in the nucleus. Other tests under investigation include identification of the Lewis X antigen on exfoliated urothelial cells, and the determination of telomerase activity in exfoliated cells. - These tests have been demonstrated to enhance detection of bladder cancer when used either individually or in combination with cytology. They have been used to detect both new index tumors as well as recurrent tumors."} {"text": "### Human: Hyper viscosity is seen in ### Assistant: Ref William hematology 6\/e p1268 The term cryoglobulinemia refers to the presence in the serum of proteins that precipitate at temperatures below 37 degrees C and redissolve on rewarming. ... The elective treatment for hyperviscosity syndrome, whether associated with monoclonal, mixed, or polyclonalcryoglobulinemia, is plasma exchange."} {"text": "### Human: The process of hardening a cement matrix through hydration with oral fluids to achieve greater mechanical strength is known as: ### Assistant: Maturation (cement)\u2014The process of hardening a cement matrix through hydration with oral fluids\u00a0to achieve greater mechanical strength.\nRef: Phillip\u2019s Science of Dental Materials ed 12 pg 309"} {"text": "### Human: Superior vena cava is derived from: ### Assistant: Ans. C Cardinal veinRef; hangman's essesntial medical Embroyology pg. 57# During early embryonic development (through 4th week), paired cardinal veins drain the body. Anterior cardinal veins drain the head and upper limb buds, while posterior cardinal veins drain the body. Both the anterior and posterior veins on each side unite at the common cardinal veins that flow into the sinus venosus and ultimately into the common atrium (Image).# Anterior cardinal veins are retained. An anastomosis between the two forms the left brachiocephalic vein, and anterior segments from both form the jugular system for the head and neck (Image).# Most of the posterior segment on the left disappears except for that forming the left superior intercostal vein, whereas the right posterior segment forms the superior vena cava.Extra mileDefects of SVC Development* Double inferior vena cava occurs when the left supracardinal vein persists, thereby forming an additional inferior vena cava below the level of the kidneys.* Left superior vena cava occurs when the left anterior cardinal vein persists, forming a superior vena cava on the left side. The right anterior cardinal vein abnormally regresses.* Double superior vena cava occurs when the left anterior cardinal vein persists, forming a superior vena cava on the left side. The right anterior cardinal vein also forms a superior vena cava on the right side."} {"text": "### Human: Testicular artery usually arises from ### Assistant: Ans) a (Abdominal aorta ....) Ref Chaurasia vol 2 pg 186Testicular artery is a branch of abdominal aorta given off at the level of vertebrae L2.Venous drainageVeins emerging from the testis forms pampiniform plexus. Plexus condenses into 4 veins at the superficial inguinal ring & into 2 veins at the deep inguinal ring.Ultimately one vein is formed which drains into IVC (on right side) or left renal vein (on left side)."} {"text": "### Human: Position of vocal cord in cadaver is: ### Assistant: The vocal fold of the excised larynx is known to be located at an intermediate position(the so-called cadaver position)."} {"text": "### Human: Normal waist hip ratio of a female is below ### Assistant: WHO CUT-OFF POINTS OF WHR Indicator CUT-OFF points Risk of metabolic complications Waist circumference >94 cm (M) >80 cm (W) Increased Hip cirrcumference >102 cm (M) >88 cm (W) Substantially increased Waist-Hip ratio >= 0.95(M) >= 0.80(W) Substantially increased Ref: Park 23rd edition Pgno : 400 WC & WHR, WHO Consultation 2008"} {"text": "### Human: The most common cause of renal scaring in a 3 year old child is - ### Assistant: Chronic pyelonephritis is characterized by renal inflammation and scarring induced by recurrent or persistent renal infection, vesicoureteral reflux, or other causes of urinary tract obstruction. VUR is a congenital condition that results from incompetence of the ureterovesical valve due to a sho intramural segment Ref Harrison20th edition pg 234"} {"text": "### Human: A 6hours old snake bite patient comes to emergency with mild local edema at the injury site. On examination no abnormalities detected and lab repos are normal. Most appropriate management is ### Assistant: All patients with a history of snake bite should be observed for 8-12 h after the bite, if the skin is broken and the offending snake cannot be positively identified as non-poisonous. Ref: Krishnan vij ; 5th ed; Page no: 484"} {"text": "### Human: Which of the following agents is most commonly associated with recurrent meningitis due to CSF leaks? ### Assistant: Intracranial CSF leaks cause bacterial meningitis, about 80% are caused by S. Pneumoniae. Other causative organisms are meningococcus, Hemophilus species and S.aurues. Ref: Clinical Pediatric Neurology By Ronald B. David, Page 217"} {"text": "### Human: Treatment of choice in traumatic facial nerve injury is: ### Assistant: (b) Facial nerve repair(Ref. Scott Brown, 6th ed., 1404)Since generally following trauma the facial nerve injury occurs as sudden onset. Facial decompression should be the best option."} {"text": "### Human: What is the best method of informing the rural population about Oral rehydration technique: March 2013 (d, f) ### Assistant: Ans. B i.e. Demonstrations Demonstration involves showing by reason or proof explaining or making clear by use of examples or experiments. Put more simply, demonstration means to clearly show"} {"text": "### Human: Risk factors associated with post-operative nausea and vomiting following strabismus surgery are all except - ### Assistant: Ans. is 'a' i.e., Age < 3 years Post operative nausea and vomiting (PONY) following strabismus surgery It can cause post - op wound dehiscence, hematoma, orbital hemorrhage and aspiration. 30 % of all procedures are associated with PONV. Major risk factors are: Age > 3years Duration of anesthesia > 30 mins Personal or family history of post - op nausea and vomiting Personal or family history of motion sickness Medical management of PONV includes: dimenhydrinate, metoclopramide, droperidol, dexamethasone, ondansatron"} {"text": "### Human: Anterolateral ahroscopy of knee is for: ### Assistant: Standard poals in knee ahroscopy Anterolateral poal Almost all the structures within the knee joint can be seen except- posterior cruciate ligament anterior poion of the lateral meniscus periphery of the posterior horn of the medial meniscus in tight knees. Anteromedial poal Additional viewing of lateral compament Posteromedial poal Used for viewing the posteromedial structures For repair or removal of the displaced posterior horn of meniscal tears Superolateral poal Used for diagnostically viewing the dynamics of patella-femoral joint, excision of medial plica."} {"text": "### Human: Regarding vaginal candidiasis which one of the following is false: ### Assistant: Ans. C. Most common in non-pregnant womenCandidiasis is caused by Candida albinism which thrives in an acidic medium with an abundant supply of carbohydrates. It is found commonly in pregnancy, and in patients on oral contraceptives, antibiotics and. corticosteroids. It is also seen in patients with diabetes. It causes a profuse discharge and intense pruritus with soreness of vagina and dysuria. Vagina and vulva are edematous and excoriated and the white patches of cheesy material on the vagina and vulva can be easily identified. A 10%KOH smear shows the buds and hyphae of the Candida organism. Nystatin or ketoconazole vaginal tablets used for one week are curative when used along with a cream of the same medication for External use."} {"text": "### Human: 25 year old patient Suspected to have a pneumoperitoneum. Patient is unable to stand. Best x-ray view is ### Assistant: Left lateral decubitus view demonstrates\u00a0the air between dense shadow of liver and the abdominal wall."} {"text": "### Human: What is the most probable poal of entry of Aspergillus? ### Assistant: Aspergillus species are widely distributed on decaying plants, producing chains of conidia. Aspergillus species unlike Candida species do not form the pa of normal flora of humans. They are ubiquitous in the environment; hence transmission of infection is mostly exogenous. Aspergillus transmission occurs by inhalation of airborne conidia. Risk Factors for invasive aspergillosis are: Glucocoicoid use (the most impoant risk factor) Profound neutropenia or Neutrophil dysfunction Underlying pneumonia or COPD, tuberculosis or sarcoidosis Antitumor necrosis factor therapy."} {"text": "### Human: For TOF management in antenatal period includes ? ### Assistant: Ans. is 'c' i.e., KaryotypingTOF has been associated with untreated maternal diabetes, phenylketonuria, and intake of retinoic acid.Associated chromosomal anomalies occur in 30% cases and include trisomies 21, 18, 13 and 22q11 microdeletion, especially in pulmonary atresia and absent pulmonary valve syndrome (APVS).There is also a high association with extra - cardiac anomalies in paicular, abdominal and thoracic abnormalities.When a TOF is diagnosed during fetal life, karyotyping and targeted morphologic ultrasound examination should be offered.Isolated TOF - Good prognosis in TOF with pulmonary stenosis (>90% survival after surgery).However, other forms of TOF such as pulmonary atresia and absent pulmonary valve syndrome do not have a good prognosis (moderate prognosis for pulmonary atresia and bad prognosis for APVS)."} {"text": "### Human: What change will be seen in vertebral column in ochronosis- ### Assistant: Ans. is 'a' i.e., Calcification of discAlkaptonuria# It is due to deficiency of homogentisate oxidase. As a result homogentisic acid (homogentisate) is excreted execessively in urine, There are three important characteristic features in alkaptonuria-Urine becomes dark after being exposed to air. It is due spontaneous oxidation of homogentisate into benzoquinone acetate, which polvmerse to form black-brown pigment alkapton which imparts a characteristic black-brown colour to urine.Alkapton deposition occurs in sclera, ear, nose, cheeks and intervertebral disc space. A condition called ochronosis. There may be calcification of intervertebral discs.Onchronosis arthritis affecting shoulder, hips, knee.# Benedict's test is strongly positive in urine and so is the ferric chloride (FeClyl test. Benedict's reagent gives a greenish brown precipitate with brownish black supematent. Feh ling's reagent (FeClj gives blue green colour."} {"text": "### Human: Pearsonian measure of skewness - ### Assistant: Ans. is b' i.e., Mean-Mode Measures of Skewness o There are following measures of skewness 1. Karl pearson's measure The formula for measuring skewness is divided into a) absolute measure Skewness = Mean - Mode b) relative measure The relative measure is known as the Coefficient of Skewness and is more frequently used than the absolute measure of skewness. Fuher, when a comparison between two or more distributions is involved, it is the relative measure of Skewness which is used."} {"text": "### Human: Following are the causes of sudden loss of vision except ? ### Assistant: Ans. is 'd' i.e., Corneal ulceration"} {"text": "### Human: All of the following are features of Lymph node histology except ### Assistant: Red pulp and White pulp are present in spleenHistologically, a lymph node is subdivided into three regions: coex, paracoex, and medulla. All three regions have a rich supply of sinusoids, enlarged endothelially lined spaces through which lymph percolates.The afferent lymph vessels pierce the capsule on the convex surface of the node and empty their lymph into the subcapsular sinus. This sinus is continuous with the coical sinuses and delivers the lymph into the medullary sinuses, eventually to enter the efferent lymphatic vessels.Reference: Krishna Garg Histology; 5th edition; Page no: 92"} {"text": "### Human: During Sx for meningioma, the left paracentral lobule was injured. It would lead to paresis of ### Assistant: Ans. is 'a' i.e Rt. Leg and perineum Paracentral lobule:On the medial surface of cerebral hemisphere, the U shaped gyrus around the end of the central sulcus is the paracentral lobule.Motor cortex is located in the precentral gyrus on the superolateral surface of the hemisphere and in the anterior part of the paracentral lobule.Stimulation in this area results in movements in the opposite half of the body.The body is represented upside down in this area.A look at the motor homunculus in Ganong will show that the leg and perineum are represented in the paracentral lobule."} {"text": "### Human: Which of the following bacterial meningitis is associated with sudural effusion? ### Assistant: Chronic Subdural Empyema It may be primary infection of subdural space from sinusitis focus causing suppuration and pus formation. It can be complication of the chronic subdural haematoma. It is due to secondary bacterial infection of collected clot\/fluid. Infection is from sinusitis scalp (common)\/through earlier trauma wound\/haematogenous. Commonly Gram positive organisms cause empyema like streptococci (viridans\/milleri) but other virulent organisms like Gram negative bacteria (H.influenzae) occasionally can cause. There is coical venous thrombophlebitis and coical infarction. Headache, fever, meningism and convulsions are the features. MRI is ideal than CT to diagnose. Treatment: Antibiotics, craniotomy and drainage; anticonvulsants, ICU care, proper monitoring, regular follow up. Condition has got 10% moality. Ref: SRB's Manual of Surgery 5th edition Pgno : 1097"} {"text": "### Human: All of the following statements are true regarding hyperophy, except: ### Assistant: The increased size of the cells is due to the synthesis and assembly of additional intracellular structural components. Hyperophy refers to an increase in the size of cells, which results in an increase in the size of the affected organ. Cells capable of division may respond to stress by undergoing both hyperplasia and hyperophy. The hyperophied organ has no new cells, just larger cells."} {"text": "### Human: Glucose sympo occurs with: ### Assistant: A i.e. Na+"} {"text": "### Human: In chronic alcoholism the rate limiting component for alcohol metabolism excluding enzymes is\/are : (PGI Dec 2008) ### Assistant: Ans : B (NAD+) & C (NADPH) In chronic alcoholism rate limiting component for alcohol metabolism is NAD* & NADPHOxidation of ethanol by alcohol dehydrogenaseQ & NADQ leads to excess production of NADHSome metabolism of ethanol takes place via a cytochrome P450 dependent microsomal ethanol oxidising systme (MEOS) Q involving NADPH & O2:, This system increase in activity in chronic alcoholism & may account for the increased metabolic clearnce in this condition.Ethanol + NADPH + H+ O2 MEOS Acetaldehyde"} {"text": "### Human: Minimum level of iodine iodized salt reaching the consumer level according to iodine programme should be:- ### Assistant: National Iodine deficiency disorder programme , 1992: - National goitre control programme, 1962 is conveed to NIDDCP, 1992 - Impact indicators: Major indicator - Urinary iodine excretion levels ( generally measured in pregnant women over 24hrs) Others - Neonatal hypothyroidism , Goitre - Level of salt iodinisation: 30 ppm at production level 15 ppm at consumer level - Two-in-one salt: 40mcg iodine + 1mg iron\/gm of salt MBI kits- Field Test Kits that allow Iodised salt manufacturers, quality controllers, health inspectors, social workers, NGO activists and even school children to assess the amount of iodine in iodised salt."} {"text": "### Human: True statement regarding specific death rates ### Assistant: (A1I of the above) (54- Park 20th)Specific death rates - may be (a) cause or disease specific e.g. TB, cancer, accidents (b) Related to specific groups e.g. age specific, sex specific, age and sex specific etc.* Rates can also be made specific for many others variables such as income, religion, race, housing etc.* Specific death rates can help us to identify particular groups or groups \"at risk\" for preventive action. They permit comparisons between different causes within the same populations* Specific death rates are obtained mainly in countries in which a satisfactory civil registration system operates and in which a high proportion of death is certified medically."} {"text": "### Human: Pre-exposure prophylaxis for rabies is given on days ### Assistant: New recommended regimen\/Schedule (New guidelines( Type of prophylaxis Regimen Post exposure Intramuscular Essen Regimen (1-1-1-1-1) Day 0,3,7,14,28 Post exposure intradermal Updated Thai Red Cross Regimen (2-2-2-0-2) Day 0,3,7,28 Post exposure in vaccinated individuals Day 0,3 Pre-exposure prophylaxis Day 0,7,21\/28 Minimum potency: 2.5 IU per IM dose Zagreb regimen of rabies Vaccine (Intramuscular Post-exposure) : 1. 4-dose abbreted multisite regimen 2-0-1-0-1 (2 doses on Day 0,1 dose on Day 7, 1 dose on day 21 Ref: Park 25th edition Pgno : 297-298"} {"text": "### Human: Regarding Sjogren's syndrome, all are true except: September 2010 ### Assistant: Ans. C: Epiphora SjOgren's syndrome (also known as \"Mikulicz disease\" and \"Sicca syndrome\", is a systemic autoimmune disease in which immune cells attack and destroy the exocrine glands that produce tears and saliva SjOgren's syndrome can exist as a disorder in its own right (Primary Sjogren's syndrome) or it may develop years after the onset of an associated rheumatic disorder such as rheumatoid ahritis, systemic lupus erythematosus, scleroderma, primary biliary cirrhosis etc. (Secondary SjOgren's syndrome)"} {"text": "### Human: Best indicator of antemoum drowning is : ### Assistant: A i.e. Fine froth in mouth & nostril"} {"text": "### Human: Splenomegaly may be a feature of: March 2013 ### Assistant: Ans. B i.e. Sickle cell anemia Splenomegaly Splenomegaly refers strictly to spleen enlargement, and is distinct from hyperspineism , which connotes overactive function by a spleen of any size. Splenomegaly and hypersplenism should not be confused. Each may be found separately, or they may coexist. Clinically if a spleen is palpable, it means it is enlarged as it has to undergo enlargement by at least two folds to become palpable. However, the tip of the spleen may be palpable in a newborn baby up to 3 months of age Early sickle cell anemia may present with splenomegaly"} {"text": "### Human: Gp2b3A inhibitors are all except - ### Assistant: Ans. is 'd' i.e., Prasugrel Glycoproteins lIb \/ IIIa inhibitorso The platelet glycoprotein mediates platelet aggregation via binding of adhesive proteins such as fibrinogen and Von Willebrand factor.o GP lib \/ Ilia inhibitors, inhibit platelet aggregation by blocking GPIIb \/ IlIa.o They are more complete inhibitors than either aspirin or clopidogrel \/ ticlopidinejj because they inhibit final pathway in platelet aggregation (whether it is mediated by ADP or TXA2), while aspirin blocks only TXA2 pathway and clopidogrel blocks only ADP pathway,o Drugs are -o In addition to inhibiting Gp lib \/ Ilia receptor, abciximab also inhibits ay(33 receptor (which binds vitronectin) and amb2 (a leukocyte integerin). This action is responsible for anti-inflammatory and antiproliferative properties of abciximab."} {"text": "### Human: All of the following could include the mechanism or function of oxigenases, EXCEPT: ### Assistant: Carboxylation is a function of carboxylase enzymes which belong to the class of 'Ligases'. It is not a function of oxygenase enzymes. Monoxygenases incorporate one atom of oxygen and play an impoant role in hydroxylation of steroids. Dioxygenases incorporate 2 atoms of oxygen into the substrate. Ref: Harper's Illustrated Biochemistry 27th Edition, Page 96, 97; Enzyme Assays: High-Throughput Screening, Genetic Selection and Fingerprinting edited by Jean-Louis Reymond, 2006, Page 85."} {"text": "### Human: According to recent SC judgment, doctor can be charged for medical negligence under 304-A, only if: AIIMS 12 ### Assistant: Ans. Gross negligence"} {"text": "### Human: Which of the following is very difficult to induce antibody - ### Assistant: An antigen is any substance that causes an immune system to produce antibodies against it. Antigens are usually peptides, polysaccharides or lipids. In general, molecules other than peptides (saccharides and lipids) qualify as antigens but not as immunogens since they cannot elicit an immune response on their own. Ref: Ananthanarayan & Panikers textbook of microbiology 9th edition pg:88"} {"text": "### Human: Prevalence is a: ### Assistant: Tools of measurement in epidemics: - Rate = x 1000\/ 10000\/ 100000 ; a is pa of b - Ratio = a\/b ; a is not a pa of b - Propoion = x 100 = % ; a is pa of b Prevalence = No. of all current cases of a disease at a time \/Estimated total population at that time X 100 Hence, Prevalence is a propoion (Prevalence is NOT a Ratio): Numerator is a pa of denominator & is always expressed in percentage. Incidence is rate."} {"text": "### Human: S2 is best appreciated in - ### Assistant: Ans. is 'a' i.e., 3rd left intercostal space o Best areas to auscultate for both components of the second heart sound (A2 and P2) are either the left sternal border at the level of second intercostal space (Pulmonic area) or the left sternal border at the level of third intercostal space (Erb s point)."} {"text": "### Human: Position of limb in posterior dislocation of hip - ### Assistant: Ans. is 'b' i.e., Flexion, adduction & internal rotation Hip conditionDeformitySynovitisArthritisPosterior dislocationAnterior dislocationFemoral neck fractureIntertrochantric fractureFlexion, abduction, external rotation, apparent lengtheningFlexion, adduction, internal rotation, true shorteningFlexion, adduction, internal rotation, apparent & true shorteningFlexion, abduction, external rotation, true lengtheningExternal rotation, later adduction & flexionMarked external rotation, later adduction & flexion"} {"text": "### Human: Drug causing malignant hyperthermia:(Asked twice in the exam) ### Assistant: Ans. b. SuxamethoniumSuxamethonium is the drug most commonly responsible for causing malignant hyperthermia.'Drugs causing malignant hyperthermia are- succinylcholine (most common cause), halothane (MC inhalational agent), iso\/des\/sevo\/methoxy- fluranes, lignocaine, TCA, MAO inhibitors and phenothiazines.''Thiopentone and pancuronium are protective drugs for malignant hyperthermia, as these raise the triggering threshold.''In malignant hyperthermia susceptible patients, safe anesthetics are N2O, Non-depolarizing muscle relaxants (atracurium), opiates, tranquilizers, etomidate, barbiturates (thiopentone) and propofol.'Malignant Hyperthermia* Autosomal dominant genetic disorder of skeletal muscle* Occurs in susceptible individuals due to exposure to some triggering agents, typically Suxamethonium or volatile agents, which cause an abnormal increase in intracellular calcium.* It is caused by dysregulation of excitation-contraction coupling in skeletal muscle.Triggering Agents:* Succinylcholine (most common cause)* Halothane (MC inhalational agent)* Halogenated Anesthesia:- Fluranes: Isoflurane, desflurane, sevoflurane. methoxy-flurane- Ether- Cyclopropane* Lignocaine* TCA* MAO inhibitors* PhenothiazinesClinical Features:* Rise in end tidal CO2 (1st sign)* Masseter muscle rigidity (MMR) and tachycardia are earliest sign* Hypermetabolic features, metabolic acidosis, fever, hyperthermia* Sympathetic over activity: Tachycardia, arrhythmia, unstable BP. initial hypertension* Muscle damage: Masseter muscle spasm, generalized rigidity, hyperkalemiaTretment:* Triggering agent must be stopped and hyperventilate with 100% oxygen* The mainstay of therapy is immediate administration of IV dantrolene.* Dantrolene is the only drug effective in reversing the symptoms and preventing the episode."} {"text": "### Human: Household insectiside used for malaria- ### Assistant: Ans. is 'a' i.e., Malathion* Insectisides used for control of malaria are-i) Residual sprapy-# In residual spray, spraying of houses with residual insectisides is done. Residual insectisides remains active over extended periods i.e., they have residual action even after the time of spray.# Commonly used residual insectisides are - Malathion, DDT, Lindane, propoxure (OMS-33).ToxicantDosage in g\/m2Average duration of effectiveness (months)DDT1 to 26 to 12Lindane0-53Malathion23OMS-3323ii) Space spray# Space sprays are those where the insecticidal formation is sprayed into the atmosphere in the form of a mist or fog to kill insect. Action is short lived and temporary since there is no residual action.# The most commonly used space spray insecticide is Pyrethrum.# New equipment has been developed to use residual insectiside as space spray by ultra low volume (ULV) technique. Malathions and fenthion are most commonly used."} {"text": "### Human: False about obstructive jaundice ### Assistant: Obstructive jaundice Most common surgical cause of obstructive jaundice is CBD stones Characterized by dark urine, pale coloured stools, icterus and Pruritus Presence of urobilinogen in urine rules out obstructive jaundice USG is the best test to differentiate medical from surgical jaundice Unconjugated bilirubin is normal Ref: Harrison's 19th edition Pg no : 1998"} {"text": "### Human: All of the following structure lies outside the cavernous sinus except ### Assistant: structures outside the sinus: 1. Superiorly: optic tract, optic chiasma, olfactory tract, internal carotid aery and anterior perforated substance. 2.inferiorly: foramen lacerum and the junction of the body and greater wing of the sphenoid bone 3.medically: hypophysis cerebri and sphenoidal air sinus 4.laterally: temporal lobe with uncus 5.below laterally: mandibular nerve 6.anteriorly: superior orbital fissure and apex of the orbit 7.posteriorly: apex of the petrous temporal and the crus cerebri of the midbrain Ref BDC volume 3:sixth edition pg 193"} {"text": "### Human: Which of the following is the best procedure done for intrapaum fetal monitoring: ### Assistant: Electrical Fetal hea monitoring is useful as: (1) Provides accurate information, (2) Helpful in diagnosing fetal distress, (3) Directs about intervention to prevent fetal death or morbidity, (4) It is superior to intermittent monitoring methods."} {"text": "### Human: Which of the following is measured by the device, Bellow's spirometer? ### Assistant: The spirometer is a simple device for measuring gas volumes. The frequently used water spirometer, rolling seal spirometer and bellows spirometer. The last two are not water-filled and are more poable. The VT, IRV, ERV, IC, and VC can all be measured with a spirometer (as can the forced expiratory volume in 1 second , forced vital capacity , and forced expiratory flow ). The RV, the FRC, and the TLC, however, cannot be determined with a spirometer because the subject cannot exhale all the gas in the lungs. The lung volumes not measurable with spirometry can be determined by the nitrogen-washout technique, by the helium-dilution technique, and by body plethysmography. The FRC is usually determined, and RV (which is equal to FRC minus ERV) and the TLC (which is equal to VC plus RV) are then calculated from volumes obtained by spirometry. Ref: Levitzky M.G. (2007). Chapter 3. Alveolar Ventilation. In M.G. Levitzky (Ed), Pulmonary Physiology, 7e."} {"text": "### Human: Which of the following is a newly emerging food poisoning organism ### Assistant: Ans: a (Salmonella typhimurium) Ref: Ananathanarayan,7th ed, p. 303Of the above given, S. typhimurium causes food poisoningSalmonella gastroenteritisS. typhimurium is the most common cause of food poisoning due to salmonella species.Incubation period--6-24 hrsFood poisoning is never caused by S. typhi. Source of transmission: poultry, meat, milk and milk products.C\/F-nausea, vomiting, diarrhoea, abdominal cramps, fever.Blood culture is usually negativeNoteS. aureus is the most common cause of food poisoning in the westIP and clinical features of various bacteria causing food poisoiningIncubation period Organism responsible 1- hrs S. aureus-nausea, vomiting, diarrheaB. cereus-nausea, vomiting 8-16 hrs C. perfringes-abdominal cramps, diarrhoea (vomiting rare)B. cereus-abdominal cramps, diarrhoea, vomiting > 16 hrs Vibrio cholera-watery diarrhoeaETEC-watery diarrhoeaEHEC-bloody diarrhoeaSalmonella species-inflammatory diarrhoeaCamphylobacter jej uni-inflammatory diarrhoeaShigella species- dysentryV. parahaemolyticus-dysentry"} {"text": "### Human: Atherosclerosis initiation by fibroblast plaque is mediated by injury to ? ### Assistant: Ans. is 'd' i.e., Endothelium The most acceptable hypothesis for the pathogenesis of atherosclerosis is \"the response to injury hypothesis\". According to this hypothesis, atherosclerosis is a chronic inflammatory response of the aerial wall initiated by injury to endothelium. Pathogenesis of atherosclerosis Following stages occurs in the pathogenesis of Atherosclerosis: Endothelial injury Earliest stages of the development of atherosclerosis are mediated by the inflammatory cascade. Inflammation mediated injury to endothelium is the cornestone in the development of atherosclerosis. After injury, endothelium is activated and there is increased expression of adhesion molecule-VCAM-1 and there is increased permeability to endothelium. TNF is the major cytokine to induce this expression. Migration of leukocytes When VCAM-1 is expressed on endothelium, leukocytes adhere to the endothelium. Leukocytes than cross the endothelial barrier and begin to accumulate in subendothelial intimal space. Macrophages engulf LDL cholesterol and form foam cells - formation of earliest lesion, i.e. fatty streak. Macrophages also form oxygen free radicals that cause oxidation of LDL to yield oxidized LDL (modified LDL). Smooth muscle cell migration and proliferation Inflammatory cells in subendothelial intimal space secrete cytokines, mainly PDGF, TGF-ct and FGF which cause migration of smooth muscle cells from media to subendothelial intimal space as well as their proliferation. Maturation of plaque Smooth muscle cells synthesize extracellular matrix (especially collegen) and conve a fatty streak into a mature fibrofatty atheroma, and contribute to the progressive growth of atherosclerotic lesions."} {"text": "### Human: The WBC that is considered \u201csecond line of defence\u201d is: ### Assistant: Monocyte is considered as the second line of defence.\n\u00a0\nPhagocytosis by Macrophages.\nMacrophages are the end-stage product of monocytes that enter the tissues from the blood. When activated by the immune system, they are much more powerful phagocytes than neutrophils, often capable of phagocytizing as many as 100 bacteria. They also have the ability to engulf much larger particles, even whole RBCs or, occasionally, malarial parasites, whereas neutrophils are not capable of phagocytizing particles much larger than bacteria. Also, after digesting particles, macrophages can extrude the residual products and often survive and function for many more months.\nReference: GUYTON AND HALL TEXTBOOK OF MEDICAL PHYSIOLOGY, THIRTEENTH EDITION(INTERNATIONAL EDITION ) page no 458"} {"text": "### Human: Mark the true statement regarding use of Lithium in maniac-depressive psychosis? ### Assistant: Lithium Use : Prophylaxis of mania alone, with sedatives in acute mania. Monitoring of S. concentration is significant to know the toxicity of drug S\/E of Lithium Lithium use for a long time causes benign and reversible T-wave flattening Seizures Polyuria and compensatory polydipsia Dermatitis, folliculitis, vasculitis Weight gain"} {"text": "### Human: Determination of Pulp vitality by pulse oximeter based on: ### Assistant: Pulse oxymetry-\n\nBased on Beer Lambert law\nUtilization of a probe that transmits red (640 mm) and\u00a0infrared light\nPhoto detector as a receptor site\nDifferent absorption ability of light by oxygenated and\u00a0deoxygenated haemoglobin leads to changes in the\u00a0amount of light differently absorbed by the vascular bed\u00a0before reaching the photo detector."} {"text": "### Human: All of the following are causes of congenital blindness Except ### Assistant: Causes of congenital blindness:ToxoplasmosisCongenital cataractsGenetic diseaseCongenital rubellaCongenital glaucomaUsher syndrome(Refer: Nelson's Textbook of Pediatrics, SAE, 1st edition, pg no. 318 - 319)"} {"text": "### Human: Hypersecretory glaucoma is seen in \u2013 ### Assistant: \"In epidemic dropsy, glaucoma is wide angle glaucoma, with normal chamber and angle. Outflow is within normal limits. The queous shows raised level of histamin, prostaglandin and protein suggesting that glaucoma is hypersecretory\". _\u00a0Mukherjee"} {"text": "### Human: A 14-year-old girl presents with quadriparesis, facial palsy, winging of scapula and ptosis. There is h\/o similar illness in father and brother but less severe. Her CPK level is also raised (500IU\/L). She is probably suffering from? ### Assistant: Facial muscle involvement is not seen with Becker and Limb-girdle muscular dystrophy Emery Dreifuss and Becker are XLR disease so can't be present in a girl child. Hence by exclusion the diagnosis is Dx: Scapulo-Faciohumeraldystrophy. Facio-Scapulo-humeral dystrophy Autosomal dominant inheritance and has an onset in childhood or young adulthood. Facial weakness is the initial manifestation-->inability to smile, whistle or fully close the eyes. Weakness of the shoulder girdles-->makes arm elevation difficult. Scapular winging becomes apparent with attempts at abduction and forward movement of the arms. The serum CK level may be normal or mildly elevated. No specific treatment is available, ankle-foot ohoses are helpful for foot-drop."} {"text": "### Human: Which of the following is true statement regarding human eye ### Assistant: Ans. b (normal rye medium will permit wavelength of 400- 700 ntn) Ref: yanoff 3rd ed p 383, internet sitesCornea and lens act as special filters absorbing the more energetic wavelength of the electromagnetic spectrum that can damage the retina. Cornea absorbs wavelength below 295nm where as lens absorbs strongly in the long U V B (300 - 315) and most of the UV A (315 - 400) wave-lengths. Eye light transmittance is relatively high in the 400nm-700nm range."} {"text": "### Human: Which of the following muscle is not a hybrid muscle ### Assistant: Extensor digitorum Hybrid muscles are the muscles having more than one nerve supply Pctoralis major: lateral & medial pectoral nerves Brachialis: additional proprioceptive fibres from radial nerve, in addition to its muscular nerve, musculocutaneous nerve. Extensor digitorum is supplied by the posterior interosseous nerve only."} {"text": "### Human: Delayed union of fracture of a bone follo-wing a surgical treatment may be due to ### Assistant: D i.e. All"} {"text": "### Human: What is the pressure of inspiration? ### Assistant: Ans: a (Intrapleural) Ref: Ganong,22nd ed,p. 6651The main pressure in lung related to inspiration is intrapleural pressure.Intrapleural pressure.The pressure in the intrapleural space between the lungs and chest wall is intrapleural pressure.Intrapleural pressure changes from -2.5 to -6 mmHg during inspiration. During strong inspiratory effort it may reach up to -30 mmHg.Transpulmonary pressureIt is the difference between alveolar pressure and intrapleural pressure. It is the measure of elastic forces in the lungs.Alveolar pressure or intrapulmonary pressureIt is the pressure inside the lung alveoli. When no air flows the alveolar pressure is considered to be equal to the atmospheric pressure, i.e., 0 cm of water.During inspiration it decreases to -1 cm of water.During expiration it increases to+1 cm of water."} {"text": "### Human: One standard detion in normal standard curev includes value- ### Assistant: Ref:Parks 23rd edition pg 849. 1SD : confidence limit = 68% 2SD : confidence limit = 95% 3SD : confidence limit = 99.7%"} {"text": "### Human: Foam cells are seen in infection with which virus ? ### Assistant: Ans. is 'b' i.e., EBV EBV encephalomyelitis shows :Abnormal white matter with perivascular infiltrates of inflammatory cells and foam cells.Some myelin is replaced by lipid-filled macrophages and hyperplastic astrocytes."} {"text": "### Human: Rescue P.C.I is done for which of the following? ### Assistant: Option C = Rescue P.C.I. After thrombolysis ST Elevation begin to reduce If ST Elevation persisting in spite of thrombolysis. So it's failure of thrombolysis | within 90 minutes Rescue PCI Types of PCI Primary PCI done for STEMI (Door to device time < 90 minutes). Delayed PCI done for Unstable Angina NSTEMI Rescue PCI Failure of thrombolysis (within 90 minutes)."} {"text": "### Human: Fever associated with infection ### Assistant: Fever right shifts the oxygen dissociation curve, which allows for a greater release of oxygen to the tissue, making oxygen easily available to neutrophils and monocytes for use in the oxygen-dependent myeloperoxidase system of bactericidal killing. Having more oxygen available to tissue also enhances wound healing. In addition, fever interferes with bacterial and viral replication by providing a hostile environment for incubation. Fever is produced by the release of interleukin-I from macrophages. Interleukin-I, in turn, stimulates the hypothalamus to synthesize prostaglandins, which interact with the thermoregulatory center to produce fever."} {"text": "### Human: Last organ to putrefy in females is? ### Assistant: Ans. is 'b' i.e., Uterus Order of putrefactiono Order of appearance of putrefaction from earliest to last is : larynx, trachea > stomach, intestine > spleen, liver > brain, lungs > heart > kidney, bladder > uterus\/prostate > bone.o Amongst the soft tissues uterus in female and prostate in malesm are last to undergo putrefaction, as uterus and prostate resist putrefaction. Otherwise bones are last to undergo putrefaction, overall."} {"text": "### Human: Hamartomatous lung tissue is? ### Assistant: Ans. (d) Congenital cystic adenomatoid malformation(Ref: Robbins 9th\/pg 670; 8th\/pg 679)Congenital cystic adenomatoid malformation (CCAM): Hamartomatous Q or dysplastic lung tissue, usually confined to one lobe."} {"text": "### Human: Test for diagnosis of pyogenic meningitis is? ### Assistant: ANSWER: (C) CSF examinationREF: Harrison 17th ed chapter 376The diagnosis of bacterial meningitis is made by examination of the CSF. The need to obtain neuroimaging studies (CT or MRI) prior to LP requires clinical judgment.A broad-range PCR can detect small numbers of viable and nonviable organisms in CSF and is expected to be useful for making a diagnosis of bacterial meningitis in patients who have been pretreated with oral or parenteral antibiotics and in whom Gram's stain and CSF culture are negative"} {"text": "### Human: Causes of thickened gallbladder wall on ultrasound examination are all except: ### Assistant: Ans: D (Kawasaki disease) Ref: Manual of Emergency Critical Care Ultrasound Vicke E Noble. 2nd ed. Pg 156Explanation:Kawasaki disease is associated with hydrops of gallbladder.Gallbladder Wall Thickening A gallbladder wall that is thickened is a sign of inflammation. This is a non-specific finding, and many other pathologic processes, as well as a postprandial gallbladder, can give you falsely elevated measurements.It is important to measure the wall of the anterior gallbladder surface because of the acoustic enhancement artifact mentioned.Because sound waves travel through a fluid-filled structure, no attenuation occurs. Thus, when those sound weaves hit the back of the gallbladder, they will be so strong that they will obscure an accurate picture of the wall thickness.For gallbladder wall thickness, greater than 3 mm is abnormal.Differential for Thickened Gallbladder WallPostprandialRenal failureAscitesHepatitisHypoalbuminemiaHIV\/AIDSAdenomyomatosisMultiple myelomaCholecystitisCongestive heart failure"} {"text": "### Human: In caloric test, cold water stimulation causes movement of eye - ### Assistant: Ans. is 'b' i.e., Opposite side [Ref: Dhingra 5th\/e p. 48}o Each ear is irrigated twice : once with cold water (70C below the body temperature, i. e. 300C) and then with warm water (70C above the body temperature, i.e. 440C).o Cold water provokes nystagmus towards the opposite ear, while warm water provokes nystagmus towardssame ear (COWS Cold - oppsite, Warm - same)."} {"text": "### Human: Which of the following is not false about annual risk of TB ### Assistant: Incidence of TB infection (Annual infection rate, Annual risk of Infection ARI) : Percentage of population under study who will be newly infected (not diseased or cases) with TB among Non-infected in 1 year. Expresses attacking force of TB in community In developing countries 1% ARI corresponds to: 50 SS +ve cases per 100,000 general population Tuberculin conversion index is the \"best indicator for evaluation of TB problem and its trend\" in the community. Current ARI in India : 1.5% Ref: Park 25th edition Pgno : 190-191"} {"text": "### Human: A 12-year-old girl presents to the casualty with symptoms of fever, shoness of breath, and cough. A chest X-ray reveals complete consolidation of the lower lobe of the left lung. Cultures of the lower lobe of the left lung would most likely reveal which organism? ### Assistant: The history and findings are consistent with a lobar pneumonia. The most common etiologic agent of lobar pneumonia is Streptococcus pneumoniae. ESSENTIALS OF DIAGNOSIS Productive cough, fever, rigors, dyspnea, early pleuritic chest pain Consolidating lobar pneumonia on chest radiograph Lancet-shaped gram-positive diplococci on Gram stain of sputum Ref: Papadakis MA, McPhee SJ, \"Pneumonia, Pneumococcal.\" Quick Medical Diagnosis & Treatment"} {"text": "### Human: IgA deposits on skin biopsy ### Assistant: A.i.e. Henosch Schonlein purpura"} {"text": "### Human: Which of the following hormonal activity is expected immediately prior to Ovulation? ### Assistant: Ovulation occurs due to the effect of increased concentration of LH. In the pre-ovulatory phase of the menstrual cycle, the ovarian follicle will undergo a series of transformations called cumulus expansion, which is stimulated by FSH. This is followed by LH surge."} {"text": "### Human: Following are fibrous joints : ### Assistant: B. i.e. Frontoparietal sutureSkull sutures (except sphen-occipital), vomer- sphenoid rostrum schindylesisQ, inferior tibiofibular syndesmosis and dentoalveolar gomphosis (tooth socket) joints are fibrous joints. A Schindylesis is a specialized suture where a rigid bone fits into a groove on a neighbouring element eg a cleft between the alae of vomer, which receives the rostrum of the sphenoid. (i.e. vomer - sphenoidal rostrum junction)Q- Epiphyseo-diaphyseal growth plateQ, costochondral, 1st chondrosternal and sphenoocipital joints are synchondrosis\/ syncendrosis or primary cailaginous joints; whereas, symphysis pubis, symphysis menti, sacrococcygeal, interveebral and manubriosternal jointsQ are symphyses or secondary cailaginous joints.Aicular cailage of most (not all) synol joints are lined by hyaline cailage Q. In some synol joints the cavity is paially or completely subdivided into two by an aicular disc, made up of fibrocailage eg. Temporomadibular jt, sternoclavicular jt, menisci of knee joint. More mobile the joint the more commonly it discloates eg shoulder jtQ. MCP Jt. is a condylar - synol joint"} {"text": "### Human: For exhumation the order should be passed by - ### Assistant: The body is exhumed only when, there is written order from the executive magistrate."} {"text": "### Human: The radiograph of a 32 year old patient is shown below. The patient is asymptomatic and the lesion revealed in the\u00a0radiograph is an accidental finding. The most likely diagnosis is: ### Assistant: Radiological signs:\nThe lesion presents as a chance radiographic finding. It is a round or an ovoid (<3 cm) uniform radiolucency with a well-defined, usually corticated, margin. Stafne\u2019s bone cavity is non-expansile and is found below the mandibular canal just anterior to the angle of the mandible. The location of Stafne\u2019s bone cavity distinguishes it from odontogenic lesions, whose origin is within the alveolar bone."} {"text": "### Human: Magistrate inquest not needed in: JIPMER 14 ### Assistant: Ans. Homicide"} {"text": "### Human: Which of the following disease with bird, ahropod and human chain ### Assistant: Agent factors :- * Caused by group B arbovirus (flavivirus) * The main reservoirs are Ardeid birds and pigs * Pigs are amplifiers of the virus and they do not manifest ove symptoms but circulates the virus Host factors :- * Most common in the age group of <15 years of age * Man - incidental dead end host * Affects other animals also and horse is the only animal which does not shows symptoms of encephalitis Vector:- * Transmitted by culex tritaeniorrhynchus - most impoant vector, others are C.Vishnuii and c. Gelidus * Incubation period is 5-15 days * Case fatality rate - 20-40% Ref:- K Park, 24th ed; pg num:- 302-303"} {"text": "### Human: Potter syndrome is not assoiated with ### Assistant: Since there is b\/l kidney agenesis or cystic disease there is no urine production\nleading to oligohydromnios"} {"text": "### Human: Increase in pauwel's angle indicate ### Assistant: Pauwel's classification - depending on angle (Pauwel's angle) formed by the line with the horizontal. More the Pauwell's angle, more unstable is the fracture with poorer prognosis Type I : <30deg Type II : 30-50deg Type III: >50deg(worst prognosis)"} {"text": "### Human: A 28 year old female teacher is found to have a prolonged bleeding time and normal platelet count. She had a bleeding of \"easy bleeding\" with frequent bleeding of the gums, epistaxis, cutaneous bleeding and menorrhagia, further testing revealed a deficiency of Von willebrand factor. Which of the following thrombogenic processes involving platelets is most directly impaired? ### Assistant: Disruption of the endothelium exposes subendothelial Von willebrand factor (VWF) and collagen, which promotes platelet adherence and activation.\nDeficiency of Von willebrand factor (VWF) causes\n\n\n\u2193\u00a0Platelet adhesion (\u2191 BT)\n\n\n\u2193 Intrinsic pathway activity (\u2191 aPTT)"} {"text": "### Human: Ifa Panchayat is dissolved, elections are to be held within? ### Assistant: Ans. is 'c' i.e., 6 months"} {"text": "### Human: Stomach is derived from ? ### Assistant: Foregut"} {"text": "### Human: Which of the following is the main enzyme responsible for activation of xenobiotics? ### Assistant: Xenobiotics are compounds which may be accidently ingested or taken as drugs or compounds produced in the body by bacterial metabolism. The cytochrome P-450 enzyme family is involved in the biotransformation reaction of xenobiotics. They are heme containing membrane proteins localized in the smooth endoplasmic reticulum of liver. They are so named because they absorb light at wavelength of 450nm, when exposed to carbon monoxide. Ref: Vasudevan Biochemistry, 3rd Edition, Pages 310-2"} {"text": "### Human: In Familial hypercholesterolemia there is deficiency of ### Assistant: LDL receptors\u00a0\n\nFamilial hypercholesterolemia (type IIa) is due to deficiency of functional LDL receptors as a result of different types of mutations."} {"text": "### Human: Botulinum affects all of the following, EXCEPT: ### Assistant: Diplopia, dysphagia, dysahria, dry mouth, vomiting, thirst, constipation are the clinical features associated with botulinum toxin. Patients experience minimal or no CNS effects and usually there's no significant alteration in their mental status. Ingested, inhaled or formed botulinum toxin travels through the vascular system and affects neuromuscular junction, pre ganglionic junction and post ganglionic nerves. Ref: Textbook of Microbiology, Ananthanarayan and Paniker, 7th Edition, Chapter 28, Page 263 & 264 ; Harrisons Principles of Internal Medicine, 16th Edition, Page 843."} {"text": "### Human: Which of the following is a poor prognostic factor for childhood ALL. ### Assistant: Answer is C (Testicular involvement) Testicular involvement \/ enlargement is a typical poor prognostic factor for childhood ALL Age < 2 years versus Testicular Involvement Typically Age < 1 year is considered a poor prognostic factor Although Age < 2 year has been mentioned as a poor prognostic factor in ceain textbooks including Hoffman's Hematology (4th\/1158) most textbooks use the criteria of Age < 1 year to define a poor prognostic factor Since testicular involvement provided amongst the option is an established poor prognostic factor, this is selected as the single best answer of choice."} {"text": "### Human: Which of the following condition is necessary prerequisite for this change? ### Assistant: Ans. (C). Adequate liquor amniiThe photograph shows maceration, a sign of intrauterine death.Conditions favoring maceration: Intact membranes; adequate liquor amnii & NO air."} {"text": "### Human: Savita is 32 weeks pregnant presents in causality and diagnosed as a case of APH. Vitals are unstable with BP 80\/60 which of the following is next step in M\/n ### Assistant: Ans. is b i.e. Blood transfusion"} {"text": "### Human: All are true about Cancer breast, except: ### Assistant: Ans. (d) Increased incidence with prolonged breast feeding* Paget's Disease is a superficial manifest of an underlying Intra ductal carcinoma breast"} {"text": "### Human: Which of the following vitamin reduces the risk of insulin resistance, obesity, and the metabolic syndrome? ### Assistant: Higher Intakes of Vitamin D May Be Beneficial There is evidence that Vitamin D intake considerably higher than that required to maintain calcium homeostasis reduce the risk of insulin resistance, obesity, and the metabolic syndrome, as well as various cancers. There is growing evidence that higher vitamin D status is protective against various cancers, including prostate and colorectal cancer, and also against prediabetes and the metabolic syndrome. Desirable levels of intake may be considerably higher than current reference intakes, and ceainly could not be met from unfoified foods. While increased sunlight exposure would meet the need, it carries the risk of developing skin cancer. Reference: Harper; 30th edition; Page no: 552, 553"} {"text": "### Human: The following drugs have significant drug interactions with digoxin except: ### Assistant: Ref:KDT 6\/e p499 Cholestyramine inhibits itestinal absorption of digoxin. Thiazides result in hypokalemia and thus make precipitate digitalis toxicity by pharmacodynamic interaction. Quinidine and verampil reduces the exceion of digoxin and does make precipitate toxicity (pharmacokinetic interaction)"} {"text": "### Human: Antiboiotic Prophylaxis for infective endocarditis is indicated in: ### Assistant: Answer is D (Coarctation of Aoa) Coarctation of Aoa is a high risk cardiac lesion fior the development of infective endocarditis and an indication fir antibiotic prophyloxis."} {"text": "### Human: Adder Head\" appearance is found in: ### Assistant: VP"} {"text": "### Human: Ortolani's test is done for ### Assistant: B. i.e. (Congenital dislocation of hip joints) (410 - Apley 8th)Important Tests**Ortolani's and Barlow's test Galeazzi test (604 CDT 4th)Congenital dislocation of hip joints** Apprehension testRecurrent dislocation of shoulder joints* Bryant's Call ways Hamilton ruler testHamilton ruler testAnterior dislocation of shoulder joints* **Pivot-shift testAnterior cruciate ligament* Lachman testAnterior cruciate ligament* Drawer testCruciate ligament (Ant & Post)* **Mc-Murray's testMeniscus injury* Appley's grinding testMeniscus injury* **Thomas testHip flexion deformity* ** Ankle joint has LEAST chances for recurrent dislocations"} {"text": "### Human: A 50 year old female patient complains of difficulty in swallowing. Patient gives h\/o multiple diagnostic CT-scans of the head and neck region. This patient may be predominantly susceptible to which of the following? ### Assistant: The incidence of thyroid carcinomas, predominantly\u00a0papillary thyroid carcinomas, increases in humans after radiation exposure. There is strong dependence on age at exposure\u2014susceptibility to radiation-induced thyroid cancer is higher in children than adults. There is little evidence for a dose response for individuals exposed during adulthood. Females are 2 to 3 times more susceptible than males to radiogenic and spontaneous thyroid cancers.\n\nReference: White and Pharoah, Oral radiology, ed 8th, pg-92"} {"text": "### Human: Which of the following may be seen in second degree Hea block (select three options): ### Assistant: Answer is A, B, and C Second degree AV block (Type I and \/or Type II) may be associated with prolonged conduction time (prolonged PR interval), Atrial rate more than ventricular rate and change in QRS complex morphology (Tipe II). Name Rhythm Rate (beats\/minute) P waves (lead II) PR Interval QRS complex First-degree atrio- ventricular (AV) block Regular That of underlying sinus rhythm; both atrial and ventricular rates will be the same Sinus origin: one P wave to each QRS complex Prolonged (more than 0.20 second); remains constant Normal (0.10 second or less) Second-degree AV Atrial: regular Atrial: that of Sinus origin Varies; progressively Normal (0.10 block, Mobitz I Ventricular: irregular underlying sinus rhythm lengthens until a P wave isn't conducted second or less) Ventricular: depends on number of impulses conducted through AV node: will he less than atrial rate (P wave occurs without the QRS cmplex); a pause follows the dropped QRS complex Second-degree AV Atrial: regular Atrial: that of Sinus origin: two or Normal or Normal if block at block, Mobitz II ventricular: usually underlying sinus three P waves prolonged; remains level of bundle of regular, but may be rhythm (sometimes more) constant His; irregular if Ventricular: depends before each QRS conductions ratios vary on number of Impulses conducted through AV node; will be less than atrial rate complex bundle branches Third-degree AV block Atrial: regular Atrial: that of Sinus P waves with Varies greatly Normal if block at Ventricular: regular underlying sinus rhythm Ventricular:40 to 60 if paced by AV junction; 30 to 40 (sometimes less) if paced by ventricles; will be less than atrial rate no constant relationship to the QRS complex; P waves found hidden in QRS complexes and T waves level of AV node or bundle of His; wide if block in bundle branches No constant Relationship between P wave and QRS complex is a feature of Third degree or complete hea block. One P wave to each QRS complex is a feature of First degree block"} {"text": "### Human: An ill 16 days old baby girl is brought to the emergency. On examination pallor and dyspnoea present with a respiratory rate of 85 per minute. Her HR is 200 bpm, hea sounds are distant and a gallop is heard. X-ray showed cardiomegaly. An echocardiogram shows dilated ventricles and dilation of the left atrium. An ECG shows ventricular depolarization complexes that have low voltage. Which of the following is the most likely diagnosis? ### Assistant: In CHF pallor, dyspnoea, tachypnoea, tachycardia and cardiomegaly are common regardless of the cause.The most common causes of CHF in children include myocarditis caused by adenovirus and coxsackievirus B.The echocardiogram shows ventricular and left atrial dilatation as well as poor ventricular function. With glycogen storage disease of the hea muscle thickening would be expected. With pericarditis- pericardial effusion is seen. On ECG, the voltages of the ventricular complexes seen with aberrant origin of the left coronary aery are not diminished, and a pattern of myocardial infarction can be seen. *"} {"text": "### Human: In inversion of the foot, the sole will face: ### Assistant: Ans: d (Medially) Ref: BDC, Vol. II, 4th ed, p. 155; Kieth L. Moore, 4th ed, p. 638Inversion is the movement in which the medial border of the foot is elevated, so that the sole faces medially.Eversion is the movement in which the lateral border of the foot is elevated. So that the sole faces laterally.Joints of the foot and the Movements taking place there:JOINTTYPE OF JOINTMOVEMENTSAnkle jointHinge jointPlantar flexion, dorsiflexionSubtalar joint (talocalcaneal)Plane synovialInversion, eversionT alocalcaneonavicularBall and socketGliding and rotator movementsCalcaneocuboidPlane synovialInversion, EversionTarsometatarsalPlane synovialGliding or slidingIntermetatarsalPlane synovialLittle movement possibleMetatarsophalangealCondylarFlexion, extension, adduction, abduction and circumductionInterphalangealHinge varietyFlexion, Extension"} {"text": "### Human: Where does the \"knot\" is put in the neck during judicial hanging? ### Assistant: In judicial hanging, a rope to allow a drop of five to seven metres according to the weight, build and age of the person, is looped round the neck, with the knot under the angle of the jaw. This causes fracture-dislocation usually at the level of the second and third, or third and fouh cervical veebrae. Ref: The essentials of forensic medicine and toxicology by Dr K S Narayan Reddy, 27th edition, Page 306."} {"text": "### Human: Seal finger and whale finger are associated with - ### Assistant: Ans. is 'b' i.e., Erysipelothrix"} {"text": "### Human: Which of the following muscle acts as unlocker of knee? ### Assistant: Popliteus rotates the tibia medially on the femur or, when the tibia is fixed, rotates the femur laterally on the tibia. At the beginning of flexion of the fully extended knee, lateral femoral rotation by popliteus muscle unlocks the joint.Must know:Locking of knee joint is due to the the action of quadriceps femoris, that brings about medial rotation of femur on tibia in later stages of extension. Ref: Snells, Clinical anatomy, 7th Edition, Page 650."} {"text": "### Human: A 60 yr old chronic smoker presents with painless gross hematuria of 1 day duration. Which is the investigation of choice to know the cause of hematuria? ### Assistant: A chronic smoker with painless gross hematuria should be suspected as having bladder cancer. The most common form of bladder cancer is transitional cell carcinoma (TCC). Tobacco use, followed by occupational exposure to various carcinogenic materials such as automobile exhaust or industrial solvents are the most frequent risk factors. Hematuria is the presenting symptom in 85-90% of patients with bladder cancer. It may be gross or microscopic, intermittent rather than constant. Exfoliated cells from both normal and neoplastic urothelium can be readily identified in voided urine. Ref: Scher H.I., Motzer R.J. (2012). Chapter 94. Bladder and Renal Cell Carcinomas. In D.L. Longo, A.S. Fauci, D.L. Kasper, S.L. Hauser, J.L. Jameson, J. Loscalzo (Eds), Harrison's Principles of Internal Medicine, 18e."} {"text": "### Human: The most sensitive method for detecting cervical Chlamydia trachomatis infection is: ### Assistant: Ans. is 'c' i.e., Polymerase chain reactionRef: Harrison, 17th\/e, p. 1074,19th\/e, p. 1165 and Ananthanarayan, 9th\/e, p. 415Amplification assays such as ligase chain reaction and polymerase chain reaction are the most sensitive chlamydial diagnostic method.Other methods are:(a) Cell culture: Sensitivity (60-80%).(b) Direct immunofluorescent antibody test: Sensitivity 70-80% and quite specific.(c) ELISA: 60-80%."} {"text": "### Human: Congestive hea failure in an infant is characterized by all except ### Assistant: Congestive Cardiac Failure in InfantsSymptomsSignsuFeeding difficultyuTakes less volume per feeduDiaphoretic while suckinguForehead sweating.uSuck-rest-suck cycle.uPoor weight gainuPeriorbital edema uClothes no longer fituIncreasing cloth sizeuRapid breathing\/nasal flaring\/cyanosis\/chest retractions.uTachycardia.uHepatomegaly.uOccasionally Splenomegaly.uPeriorbital edemauEdema in flanks. uDependent edema (Refer: Nelson's Textbook of Paediatrics, 19thedition, pg no:1530)"} {"text": "### Human: Congenital hydrocele is best treated by ### Assistant: Congenital hydrocele is best treated with herniotomy if they do not resolve spontaneously. Established acquired hydrocele often have thick walls. They are treated by subtotal excision of the sac. If the sac is small, thin walled and contains clear fluid, Lord's operation is done. Jaboulay's operation where the sac is eveed with placement of testis in a pouch created by dissection in the fascial planes of the scrotum, is an alternative. Reference : page 1328 Bailey and Love's sho practice of surgery 25th edition and page 1072 SRB's manual of surgery 5th edition"} {"text": "### Human: Rate limiting step in cholesterol synthesis is catalysed by which of the following enzyme? ### Assistant: Conversion of HMG CoA to mevalonate by HMG CoA reductase is the rate limiting step in the synthesis of cholesterol. Cholesterol is an allosteric inhibitor of HMG CoA reductase Statin drugs acts as competitive inhibitors with mevalonate for binding to HMG CoA reductase. Insulin ors the active form of HMG CoA reductase and increases cholesterol synthesis. Glucagon ours the inactive form and decreases cholesterol synthesis. Ref: Biochemistry By John W. Pelley, page 89."} {"text": "### Human: Vaccine administered by following route is ### Assistant: Hlnl live (Intranasal route)"} {"text": "### Human: Coloured urine is not seen in : ### Assistant: Answer is A (Quinine) Quinine is not associated with discoloured urine."} {"text": "### Human: Schizophrenia is treated by ### Assistant: Ans. is 'b' i.e., Anti psychotics"} {"text": "### Human: Fries test is used in the diagnosis of: ### Assistant: Granuloma venereum"} {"text": "### Human: A 45-year-old woman complains of severe headaches and difficulty in swallowing. Over the past 6 months, she has noticed small, red lesions around her mouth as well as thickening of her skin. The patient has \"stone facies\" on physical examination. Which of the following antigens is the most common and most specific target of autoantibody in patients with this disease? ### Assistant: Scleroderma (refer to the image below) Autoimmune disease of connective tissue. Antinuclear antibodies are common but are usually present in a lower titer than in patients with SLE. Antibodies viually specific for scleroderma include:- Nucleolar autoantibodies (primarily against RNA polymerase) Antibodies to Scl-70, a non-histone nuclear protein topoisomerase; Anticentromere antibodies (associated with the \"CREST\" variant of the disease). The Scl-70 autoantibody is most common and specific for the diffuse form of scleroderma and is seen in 70% of patients. -Autoantibodies to double-stranded DNA (choice B) are seen in patients with SLE. -Autoantibodies to SS-A\/SS-B are seen in patients with Sjogren syndrome. (Fig: A fibroblastic focus is present at the arrows.) Diagnosis: Scleroderma"} {"text": "### Human: Grave's ophthalmopathy mostly presents as September 2010 ### Assistant: Ans. A: Proptosis Graves' ophthalmopathy (also known as thyroid eye disease (TED), dysthyroid\/thyroid-associated orbitopathy (TAO), Graves' orbitopathy) is an autoimmune inflammatory disorder affecting the orbit around the eye, characterized by upper eyelid retraction, swelling (edema), redness (erythema), conjunctivitis, reduce dpower convergence and bulging eyes (proptosis)."} {"text": "### Human: Earliest lesion seen in atherosclerosis is: ### Assistant: Ans: A (Fatty streaks) Ref: Robbins Pathologic Basis of Disease, 8th edition.Explanation:Fatty streaks are the earliest lesions in atherosclerosis \" Ref: RobbinsMorphology of AtherosclerosisFatty StreaksThese are the earliest lesions in atherosclerosis.They are composed of lipid-filled foamy macrophages.Beginning as multiple minute flat yellow spots, they eventually coalesce into elongated streaks 1 cm or more in length.These lesions are not significantly raised and do not cause any flow disturbanceAortas of infants less than 1 year old can exhibit fatty streaks, and such lesions are seen in virtually all children older than 10 years, regardless of geography, race, sex, or environment.The relationship of fatty streaks to atherosclerotic plaques is uncertainThey may evolve into precursors of plaques, not all fatty streaks are destined to become advanced lesions.Atherosclerotic PlaqueThe key processes in atherosclerosis are intimal thickening and lipid accumulationAtheromatous plaques impinge on the lumen of the artery and grossly appear white to yellow: superimposed thrombus over ulcerated plaques is red-brown.Plaques vary from 0.3 to 1.5 cm in diameter but can coalesce to form larger massesAtherosclerotic lesions are patchy, on cross- section, the lesions appear \"eccentric\"Local flow disturbances (e.g., turbulence at branch points) leads to increased susceptibility to plaque formation.In descending order, the most extensively involved vessels are theLower abdominal aorta (MC)The coronary arteries (Second MC)The popliteal arteriesThe internal carotid arteriesVessels of the circle of WillisVessels of the upper extremities are usually spared, as are the mesenteric and renal arteries, except at their ostia.Atherosclerotic plaques have three principal components:Cells, including smooth muscle cells, macrophages, and T cellsECM, including collagen, elastic fibers, and proteoglycansIntracellular and extracellular lipidSuperficial fibrous cap composed of smooth muscle cells and dense collagen.Beneath and to the side of the cap (the \"shoulder\") is a more cellular area containing macrophages, T cells, and smooth muscle cells.Deep to the fibrous cap is a necrotic core, containing lipid (primarily cholesterol and cholesterol esters), debris from dead cells, foam cells (lipid-laden macrophages and smooth muscle cells), fibrin, variably organized thrombus, and other plasma proteins and cholesterolThe periphery of the lesions show neovascularization (proliferating small blood vessels;Plaques generally continue to change and progressively enlarge due to cell death and degeneration, synthesis and degradation (remodeling) of ECM, and organization of thrombus.Atheromas often undergo calcificationAtherosclerotic plaques are susceptible to the following clinically important changes* Rupture, ulceration, or erosion of the intimal surface of atheromatous plaques exposes the blood to highly thrombogenic substances and induces thrombosis.* Such thrombosis can partially or completely occlude the lumen and lead to downstream ischemia.* If the patient survives the initial thrombotic occlusion, the clot may become organized and incorporated into the growing plaque.* Hemorrhage into a plaque. Rupture of the overlying fibrous cap, or of the thin-walled vessels in the areas of neovascularization, can cause intra-plaque hemorrhage; a contained hematoma may expand the plaque or induce plaque rupture.* Atheroembolism. Plaque rupture can discharge atherosclerotic debris into the bloodstream, producing microemboli* Alienism formation. Atherosclerosis-induced pressure or ischemic atrophy of the underlying media, with loss of elastic tissue, causes weakness resulting in aneurysmal dilation and potential rupture."} {"text": "### Human: Flipped LDH indicating Myocardial Infarction is represented by: ### Assistant: When LDH-1 is greater than LDH-2, it is referred to as flipped LDH and is indicative of MI. LDH-1 isoenzyme is found primarily in hea muscle and is lesser in concentration than the LDH-2 Isoenzyme which is primarily found in RBC. When the concentration of LDH-1 is observed to be greater than LDH2. It is referred to as flipped LDH and is indicative of Myocardial infarction. LDH has five isoenzymes: LDH-1 LDH-1 is found primarily in hea muscle 17% to 27% LDH-2 LDH-2 is primarily found in Red blood cells 27% to 37% LDH-3 LDH-3 is highest in the lung 18% to 25% LDH-4 LDH-4 is highest in the kidney, placenta, and pancreas 3% to 8% LDH-5 LDH-5 is highest in the liver and skeletal muscle 0% to 5% The LDH test helps determine the location of tissue damage. Thus in Normal ratios LDH-1 is less than LDH-2 and LDH-5 is less than LDH-4. When LDH-1 is greater than LDH-2: It is referred to as flipped LDH. It is indicative of MI. When your LDH-5 is greater than your LDH-4: It could mean damage to the liver or liver disease. This includes cirrhosis and hepatitis. Ref: Harrisons 19th edition"} {"text": "### Human: Blood supply of stomach is\/are: ### Assistant: A. i.e. Left gastric aery; B. i.e. Sho gastric aery; C. i.e. Left gastroepiploic aery"} {"text": "### Human: Which of the following ultrasound finding has the highest association with aneuploidy? ### Assistant: All the above mentioned are ultrasound findings associated with increased risk of aneuploidy although the highest association is seen with cystic hygroma Nuchal translucency and cystic hygroma are both measured in the first trimesterTrisomy 21 is the most common aneuploidy associated with increased NT and cystic hygroma While monosomy X presents as second-trimester hygroma."} {"text": "### Human: Which of the following is not a component of transpo system? ### Assistant: Functions: Transpo Systems An essential role of biomembranes is to allow movement of all compounds necessary for the normal function of a cell across the membrane barrier. These compounds include a vast array of substances like sugars, amino acids, fatty acids, steroids, cations and anions to mention a few. These compounds must enter or leave the cells in an orderly manner for normal functioning of the cell. A. 1. Ion Channels Ion channels are transmembrane channels, pore-like structures composed of proteins. Specific channels for Na+, K+, Ca++, and Cl- have been identified. Cation conductive channels are negatively charged within the channel and have an average diameter of about 5 to 8 nm. All ion channels are basically made up of transmembrane subunits that come together to form a central pore through which ions pass selectively. All channels have gates and are controlled by opening and closing. Types of Gates Two types of gated channels. They are a. Ligand-gated channels: In this, a specific molecule binds to a receptor and opens the channel. Example: Acetylcholine receptor is present in the postsynaptic membrane. It is a complex of five subunits, having a binding site for acetylcholine. Acetylcholine released from the presynaptic region binds with the binding site of the postsynaptic region, which triggers the opening of the channel and influx of Na+. b. Voltage-gated channels: These channels open or close in response to changes in membrane potential. Some propeies of ion channels * Composed of transmembrane protein subunits. * Highly selective. * Well regulated by the presence of \"gates\". * Two main types of gates: Ligand-gated and voltage-gated. * Activities are affected by ceain drugs. * Mutations of genes encoding transmembrane proteins can cause specific diseases. 2. Ionophores Ceain microorganisms can synthesize small organic molecules, called ionophores, which function as shuttles for the movement of ions across the membrane. Structure: These ionophores contain hydrophilic centres that bind specific ions and are surrounded by peripheral hydrophobic regions. Types: Two types: (a) Mobile ion carriers: Like valinomycin (Refer uncouplers of oxidative phosphorylation). (b) Channel formers: Like gramicidin. 3. Water Channels (Aquaporins) In ceain cells, e.g. in red blood cells, and cells of the collecting ductules of the kidney, the movement of water by simple diffusion is enhanced by movements of water through water channels, composed of tetrameric transmembrane proteins called aquaporins. About five distinct types of aquaporins have been recognised. CLINICAL ASPECT Recently mutations in the gene encoding AP-2 (Aquaporin 2) protein, have been shown to be the cause of one type of nephrogenic diabetes insipidus. 4. Gap Junction Ceain cells develop specialized regions on their membranes for intercellular communications which are in close proximity. Function: They mediate and regulate the passage of ions and small molecules upto 1000 to 2000 mol wt, through a narrow hydrophilic core connecting the cytosol of adjacent cells. Structure: They are primarily composed of protein, called connexon which contains four membrane-spanning a-helices. the mitochondrial membrane also has paicularly energy coupling mechanism.Ref: MN Chatterjea Textbook of Medical Biochemistry, 8th edition"} {"text": "### Human: Burst supression pattern on EEG is typically seen in: ### Assistant: Ref: Ref: Harrisons 18\/e p e-45 chapter\n\u00a0As the depth of coma increases, the EEG becomes nonreactive and may show a burst-suppression pattern, with bursts of mixed-frequency activity separated by intervals of relative cerebral inactivity. In other instances there is a reduction in amplitude of the EEG until eventually activity cannot be detected. Such electrocerebral silence does not necessarily reflect irreversible brain damage, because it may occur in hypothermic patients or with drug overdose.\nThe prognosis of electrocerebral silence, when recorded using an adequate technique, depends upon the clinical context in which it is found. In patients with severe cerebral anoxia, for example, electrocerebral silence in a technically satisfactory record implies that useful cognitive recovery will not occur."} {"text": "### Human: To feel comfoable and cool in a factory the conrrected effective temperature ( in degree Farenheit) should be - ### Assistant: Comfo zones: In deg C of corrected effective temperature Pleasant and cool 20 Comfoable and cool 20-25 (69-76F) Comfoable 25- 27 (77-80F) Hot and uncomfoable 27-28 Extremely hot 28+ Intolerably hot 30+. Park's Textbook of Preventive and Social Medicine, 25th Edition, Pg 792"} {"text": "### Human: Test used for factor VIII deficiency identification is- ### Assistant: Ans. is 'b' i.e., APTT o Factor VIII is a part of the intrinsic pathway of coagulation. The test used to identity1 the defects in the intrinsic and common pathways of coagulation is PTT. Thus prolonged PTT is a feature of patient with factor VIII deficiency.o Patients with hemophilia have deficiency of factor VIII that results in prolonged PTT.o Tests used to evaluate different aspects of hemostasis.Bleeding timeo It is not a test for coagulation rathers it tests the ability of the vessels to vasoconstrict and the platelets to form a hemostatic plug.o It is the time taken for a standardized skin puncture to stop bleeding,o Normal reference value is between 2-9 minutes.o Prolongation generally indicates the defect in platelet number or function.Prothrombin time (PT)o This assay tests the extrinsic and common coagulation pathway.o So, a Prolonged PT can result from deficiency of factor V, VII. X, prothrombin or fibrinogen.Partial thromboplastin time (PTT)o This assay tests the intrinsic and common coagulation pathways.o So, a prolonged PTT. can results from the deficiency of factor V, VHI, IX, X, XI, XII, prothrombin or fibrinogen.Thrombin timeo It is the time taken for clotting to occur when thrombin is added to the plasma,o It tests the conversion of fibrinogen to fibrin and depends on adequate fibrinogen level,o Prolonged thrombin time results from decreased level of fibrinogen.Defective coagulation pathwayPTAPTTExtrinsicIntrinsicCommonIncreasedNormalIncreasedNormalIncreasedIncreased"} {"text": "### Human: A 19 year old female presents with pain in the neck for 5 days. She is not able to wear tie for her job because of neck pain. H\/O fatigue and lethargy for 10 days. She had flu like symptoms 20 days ago which resolved spontaneously. BP 110\/80 mmHg, Pulse 48\/min. Extremities are cold and dry. Neck is very tender. ECG normal. TSH is elevated. ESR 30 mm\/hr. Next appropriate step ### Assistant: Answer: c) Aspirin (SCHWARTZ 19TH ED, P-1525; SABISTON 19TH ED, P-895)Granulomatous or Subacute or De Quervain's thyroiditisMost commonly occurs in 30- 40 year-old womenStrong association with the HLA-B35Fever with Sudden or gradual onset of neck painH\/o preceding URI; Viral etiologyGland - enlarged, tender, firmClassically progresses through four stages: Hyperthyroid-Euthyroid-Hypothyroid-EuthyroidA few patients develop recurrent diseaseEarly stages: | ESR, Tg, T4, and T3 levels are elevated, TSH decreasedDuring Hypothyroid stage: elevated TSHLow radio-iodine uptakeFNAC - multinucleated giant cells of an epithelioid foreign body type and aggregates of lymphocytes activated macrophages, and plasma cellsSelf-limiting diseaseAspirin and NSAIDs are used for pain reliefPrednisolone for severe casesShort-term thyroid replacement may be needed and may shorten the duration of symptoms"} {"text": "### Human: Headache, apathy and deteriorating level of consciousness occurring weeks after head injury suggests; ### Assistant: Chronic subdral haematoma"} {"text": "### Human: A 27-year female wearing high heels stumbled and sustained inversion injury at ankle. Which ligament is most likely to be injured in the given case? ### Assistant: Ans. A. (A)A. Anterior Talofibular ligamentAnkle sprains: Usually caused by the falls from height or twists of ankle. When the plantar-flexed foot is excessively inverted, the anterior and posterior talofibular and calcaneofibular ligaments are stretched and torn. The anterior talofibular ligament is most commonly torn."} {"text": "### Human: Asymmetric tonic neck reflex (ATNR) disappears at ______ ### Assistant: Asymmetric tonic reflex (ATNR) diappears by 6-7 months postnatally The tonic neck reflex is produced by manually rotating the infant&;s head to 1 side and observing for the characteristic fencing posture (extension of the arm on the side to which the face is rotated and flexion of the conralateral arm) An obligatory tonic neck response, in which the infant becomes \"stuck\" in the fencing posture, is always abnormal and implies a CNS disorder Ref: Nelson 21st edition pgno: 3059"} {"text": "### Human: Ligament teres is a remnant of ? ### Assistant: Umbilical vein"} {"text": "### Human: Sho structured primi gravida has height less then ### Assistant: 140 cm"} {"text": "### Human: Which of the following is the most specific and sensitive screening test for renovascular hypeension? ### Assistant: Hypeension due to an occlusive lesion of a renal aery is renovascular hypeensionIn the initial stages, the mechanism of hypeension generally is related to activation of the renin-angiotensin system. Obstruction of the renal aery leads to decreased renal perfusion pressure, thereby stimulating renin secretion. Over time, possibly as a consequence of secondary renal damage, this form of hypeension may become less renin-dependent.As a screening test, renal blood flow may be evaluated with a radionuclide -ohoiodohippurate (OIH) scan, or glomerular filtration rate may be evaluated with a - diethylenetriamine pentaacetic acid (DTPA) scan before and after a single dose of captopril (or another ACE inhibitor).Contrast aeriography remains the \"gold standard\" for evaluation and identification of renal aery lesions.Harrison 19e pg: 1618"} {"text": "### Human: Role of egg lecithin in propofol formulation is? ### Assistant: Propofol is an insoluble drug that requires a lipid vehicle for emulsification. Current formulations of propofol use a soybean oil as the oil phase and egg lecithin as the emulsifying agent."} {"text": "### Human: A 63 year old man presents with a triad of angina, syncope and congestive hea failure. Which of the following valvular hea lesion can be suspected- ### Assistant: Even severe AS may exist for many years without producing any symptoms because of the ability of the hyperophied LV to generate the elevated intraventricular pressures required to maintain a normal stroke volume. Most patients with pure or predominant AS have gradually increasing obstruction over years, but do not become symptomatic until the sixth to eighth decades. Exeional dyspnea, angina pectoris, and syncope are the three cardinal symptoms. Dyspnea results primarily from elevation of the pulmonary capillary pressure caused by elevations of LV diastolic pressures secondary to reduced left ventricular compliance and impaired relaxation. Angina pectoris usually develops somewhat later and reflects an imbalance between the augmented myocardial oxygen requirements and reduced oxygen availability. Exeional syncope may result from a decline in aerial pressure caused by vasodilation in the exercising muscles and inadequate vasoconstriction in nonexercising muscles in the face of a fixed CO, or from a sudden fall in CO produced by an arrhythmia. ( Harrison&;s principle of internal medicine,18th edition,pg no.1939 )"} {"text": "### Human: Which of the following is released from hypothalamus? ### Assistant: There are 6 established hypothalamic releasing and inhibiting hormones : \n\nCorticotropin-releasing hormone (CRH),\nThyrotropin releasing hormone (TRH),\nGrowth hormone releasing hormone (GRH),\nGrowth \u00a0hormone inhibiting hormone (GIH),\nLuteinizing hormone releasing hormone (LHRH) and\nProlactin inhibiting hormone(PIH)."} {"text": "### Human: Vestibulo-Ocular Reflex Mediated by: ### Assistant: Ans:A (Flocculonodular Lobe) Ref: Pal GK. Cerbellurn. In: Textbook of Medical Physiology. Ahuja Publishing House. 2007: 116: 871.Explanation:Functional Division of CerebellumVestibulo cerebellum\/Arch-cerebellum (Flocculonodular lobe)Connections with vestibular apparatusConcerned with equilibrium and learning induced changes in vesiihulo-occular reflexSpino cerebellumVermis - controls muscle movements of axial bodyIntermediate zone - controls muscle movements of limbsCerbrocerbellum (Ponto cerebellum\/Neo cerbellurn) Overall planning programming of motor activities"} {"text": "### Human: Which statement best describes the cranial fossa? ### Assistant: The floor of the middle cranial fossa is made up of the body and greater wing of the sphenoid as well as the squamous temporal bone. The cribriform plate forms pa of the floor of the anterior cranial fossa The middle cranial fossa contains the pituitary gland. The internal acoustic meatus lies in the posterior cranial fossa."} {"text": "### Human: Which of the following vitamin enhances intestinal absorption of calcium? ### Assistant: Vitamin Functions Deficiency Disease D Maintenance of calcium balance Enhances intestinal absorption of Ca2+ and mobilizes bone mineral Regulation of gene expression and cell differentiation Rickets = poor mineralization of bone Osteomalacia = bone demineralization Reference: Harper; 30th edition; Page no: 550"} {"text": "### Human: Which of these is the most impoant indication for Strassmans Metroplasty done for a bicornuate uterus? ### Assistant: Habitual aboion is the most impoant indication for surgical treatment of women who have a double uterus. The aboion rate in women who have a double uterus is to three times greater than that of the general population. Best diagnosis of the condition is made by laparohysteroscopy. Strassmans Metroplasty"} {"text": "### Human: A 2-year-old child had burns on buttocks, both legs, face, neck and singeing of hair. Total surface area burnt: JIPMER 14 ### Assistant: Ans. 37%"} {"text": "### Human: Which test helps to differentiate between concomitant squint and paralytic squint ### Assistant: This procedure causes breakdown of binocular fusion mechanism thus helpful to differentiate."} {"text": "### Human: Best disinfectant of cholera stool is- ### Assistant: Cresol is an excellent coal-tar disinfectant. It is 3 to 10 times as powerful as phenol, yet no more toxic. Cresol is best used in 5 to 20 percent strength for disinfection of farces and urine. Cresol is an all-purpose general disinfectant. (refer pgno :128 park 23 rd edition)"} {"text": "### Human: Black eye is a type of - ### Assistant: The synopsis of forensic medicine & toxicology ; Dr k.s.narayan reddy ; 28th edition ; Pg .no . 107 Black eye is the most common example for ectopic bruising or percolated contusion ."} {"text": "### Human: In a patient with a type of familial dyslipidemias he presented with palmar Xanthomas and he ias at increased risk of atherosclerosis and CAD.In Lipid profile there was elevation of triacylglycerols and cholesterol.And the IDL and chylomicrons were elevated.What is the pathophysiology of the condition? ### Assistant: Reference: Harpers illustrated biochemistry 30th edition page 275"} {"text": "### Human: Endolymphatic duct drains into : ### Assistant: Ans:A.)Subdural Perilymph drains into subarachnoid space through the aqueduct of cochlea... Endolymph is absorbed in subdural space."} {"text": "### Human: Usually associated with parvovirus B19 infection in those with hereditary spherocytosis- ### Assistant: Ans. is 'b' i.e., Aplastic crisis o Parvovirus B19 selectively infects erythroidprecursors and is the most common aetiological agent that induces aplastic crisis in patients with hereditary spherocytosis (and other Hemolytic disorders).Transient aplastic crisiso Persons with decreased erythrocytes caused by conditions such as iron deficiency anemia, human immunodeficiency virus sickle cell disease, spherocytosis or thalassemia are at risk of transient aplastic crisis if infected with parvovirus B19.o The virus causes a cessation of erythrocyte production,o Parvovirus infection may be the first manifestation in HS.o It begins with reticulocytosis and thrombocytosis."} {"text": "### Human: A patient with cushinoid features presents with hemoptysis; he shows no response to dexamethasone suppression test; most likely diagnosis here is: ### Assistant: Answer is C (Ca lung with ectopic ACTH production); Lack of suppression to dexamethasone challenge (ACTH producing tumor) and presence of hemoptysis (Carcinoma lung) in a patient with Cushinoid features suggests a diagnosis of Carcinoma lung with ectopic ACTH secretion. As a method of diagnostic workup of any patient with suspected Cushings we screen the patient with plasma coisol levels at 8:00 a.m. in the morning. Those with increased coisol levels are subjected to a dexamethasone supression test. In the given question there is lack of suppression to dexamethasone challenge so option (d) is out. Out of the choices remaining the presence of hemoptysis points towards the diagnosis of carcinoma lung with ectopic ACTH."} {"text": "### Human: Symptomatic treatment is not required in withdrawal of: ### Assistant: Since cannabis causes very mild withdrawal symptoms hence, no symptomatic treatment is required. LSD and other hallucinations also do not cause any withdrawal symptoms ."} {"text": "### Human: Following death of a person, what happens to potassium level in the vitreous. ### Assistant: Potassium concentration raises at the rate of 0.17-0.25 mmol per hour following death. It\u00a0helps in determination of time since death."} {"text": "### Human: Under transplantation of humans organs act, the punishment for the doctor involved is ### Assistant: The Transplantation of Human Organs Act 1994, amended in 2011, and 2012 Chapter VI and VII deals with the trial and punishment of offences under this act. Under this act, doctors involved will be punished for a period of 2-5yrs. Ref: FORENSIC MEDICINE AND TOXICOLOGY Dr. PC IGNATIUS PAGE NO: 389"} {"text": "### Human: Staphylococcus aureus does not cause which of the following skin infection: ### Assistant: Ans. is. a. Ecthyma gangrenosum"} {"text": "### Human: A young lady with symptoms of hypehyroidism with elevated T4 and TSH levels. On examination, patient was not experiencing Tachycardia or excessive sweating.Fuher examination reveals bitemporal hemianopia. Next step of management ### Assistant: Hypehyroidism with elevated levels of T4 and TSH levels and bitemporal hemianopia is highly suggestive of TSH secreting adenoma. Urgent MRI should be done to confirm the diagnosis of TSH secreting pituitary adenoma along with antithyroid drugs. Thyrotropin-secreting pituitary tumors (TSH-omas) are a rare cause of hypehyroidism and account for less than 1% of all pituitary adenomas. - Origin - anterior pituitary- MC cause of hyper pituitarism - Pituitary adenoma C\/F Functional (Secreting) Non-Functional (Non-Secreting)Early Presentation due to physiologic Delayed presentation (Big enough toeffect cause neurological deficit) | Bitemporal Hemianopia Seen on 3rd \/ 4th Decade- M = F- MC type of pituitary adenoma is Prolactinoma IOC for diagnosis - MRIOther work-up done Full Endocrinologic ProfileFormal visual field testing TreatmentBromocriptine (Dopamine agonist) -Prolactinoma- Shrink prolactinoma in 6 to 8 weeks- Growth hormone security tumors - shrinkage occur < 20% Pts | Agent Octreotide | GH level in 70% Pts , | Tumor volume in 30% Indications of Sx in pituitary adenoma- GH Secreting tumors- Primarily Cushing Disease- Adenoma causing acute visual deterioration- Non-Prolactin secreting macroadenoma causing symptoms by mass effect Surgical ApproachIntranasal Trans-Sphenoidal , Sub labial Indications of Radiosurgery in pituitary adenoma- Used as primary therapy- Adjuvant therapy after subtotal resection- Recurrent disease"} {"text": "### Human: Statuary rape is - ### Assistant: Ref:Textbook of Forensic medicine and toxicology (V.V.Pillay) 17th edition, page no.388 Minimum age at which female can give consent for sex by what age is 18 years. As per Sec.375IPC unlawful sexual intercourse by man with a woman with or without her consent when she is under 18 years of age. The criminal law (Amendment) Act,2013 has increased the previous recomended age of consent from 16 to 18 years, which means any sexual activity irespective of presence of consent with a woman below the age of 18 will constitute statutory rape."} {"text": "### Human: A patient on amiodarone is diagnosed to have cornea verticillata. What should be management \u2013 ### Assistant: Cornea Verticillata\n\nThis is a whorl-like opacity in the corneal epithelium seen in patients on long-term treatment with medication such as amiodarone, chloroquine, phenothiazines and indomethacin.\nIt is also seen in patients with Fabry disease and its carrier state. The condition is generally asymptomatic, harmless and reversible on stopping the drug.\nThe whorl-like pattern shows the direction of migration of corneal epithelial cells. Occasionally the condition had been known to cause glare and surface discomfort which response to topical lubricants."} {"text": "### Human: Most aggressive lung CA: ### Assistant: Ans. (c) Small cell lung CARef: Robbings 9th ed.\/ 717* Small cell carcinoma is a highly malignant tumor with a strong relationship to cigarette smoking.* Around 1% occurs in nonsmokers.* They are the most aggressive of lung tumors, metastasizing widely and are always fatal."} {"text": "### Human: Chemoprophylaxis is indicated for all except- ### Assistant: Chemoprophylaxis There is no role of Chemoprophylaxis in Typhoid. For chemoprophylaxis in meningococcal meningitis, Rifampicin, Ciprofloxacin & Ceftriaxone is used. Tetracycline is used in Cholera and Plague."} {"text": "### Human: In pontaic fever, which antigen is seen in urine? ### Assistant: Ans. is 'a' i.e., Lipopolysaccharide-1 Legionella are classified into serogroup on the basis ofgroup specific lipopolysaccharide (somatic antigen or 'O' antigen). Legionella pneumophila sero-group-1 (LP-1) is the most common infecting organism. Urine test detect LP-1."} {"text": "### Human: Mediastinal lymph node calcification is seen in which one of the following- ### Assistant: Ref:Robbins Basic Pathology (9th Edition),page no.478,479 Sarcoidosis Although sarcoidosis is considered here as an example of a restrictive lung disease, it is impoant to note that sarcoidosis is a multisystem disease of unknown etiology characterized by noncaseating granulomas in many tissues and organs. MORPHOLOGY The diagnostic histopathologic feature of sarcoidosis is the noncaseating epithelioid granuloma, irrespective of the organ involved. This is a discrete, compact collection of epithelioid cells rimmed by an outer zone of largely CD4+ T cells. The epithelioid cells are derived from macrophages and are characterized by abundant eosinophilic cytoplasm and vesicular nuclei. It is not uncommon to see intermixed multinucleate giant cells formed by fusion of macrophages. A thin layer of laminated fibroblasts is present peripheral to the granuloma; over time, these proliferate and lay down collagen that replaces the entire granuloma with a hyalinized scar. Two other microscopic features are sometimes seen in the granulomas: (1) Schaumann bodies, laminated concretions composed of calcium and proteins; and (2) asteroid bodies, stellate inclusions enclosed within giant cells. Their presence is not required for diagnosis of sarcoidosis--they also may occur in granulomas of other origins. Rarely, foci of central necrosis may be present in sarcoid granulomas, suggesting an infectious process. Caseation necrosis typical of tuberculosis is absent The lungs are involved at some stage of the disease in 90% of patients. The granulomas predominantly involve the interstitium rather than air spaces, with some tendency to localize in the connective tissue around bronchioles and pulmonary venules and in the pleura (\"lymphangitic\" distribution). The bronchoalveolar lavage fluid contains abundant CD4+ T cells. In 5% to 15% of patients, the granulomas eventually are replaced by diffuse interstitial fibrosis, resulting in a so-called honeycomb lung. Intrathoracic hilar and paratracheal lymph nodes are enlarged and calcification in 75% to 90% of patients, while a third present with peripheral lymphadenopathy. The nodes are characteristically painless and have a firm, rubbery texture. Unlike in tuberculosis, lymph nodes in sarcoidosis are \"nonmatted\" (nonadherent) and do not ulcerate."} {"text": "### Human: \"Candle-wax spots\" in the retina are present in ### Assistant: (A) Sarcoidosis # OCULAR MANIFESTATIONS OF SARCOIDOSIS:> Anterior segment: Conjunctival involvement has been reported in patients with ocular sarcoidosis.> Sarcoidosis granulomas are solitary, yellow \"millet-seed\" nodules.> Anterior uveitis occurs in 22%- 70% of patients with ocular sarcoidosis, and is usually granulomatous & chronic.> Iris nodules have been reported in up to 12.5% of patients with sarcoidosis associated uveitis.> Exacerbations of granulomatous uveitis are often associated with an appearance of fresh iris or fundus nodules.> Posterior synechiae, cataract and glaucoma are common complications.> Corneal band keratopathy develops in a few patients and is usually associated with hypercalcemia.> Posterior segment: The most common manifestations at the posterior segment are vitritis, intermediate uveitis, panuveitis, posterior uveitis, retinal vasculitis & optic nerve involvement. Other manifestations include choroidal nodules & exudative retinal detachment.> Overall, patients with chronic posterior uveitis and panuveitis have significantly more complications than do patients with anterior uveitis.> \"Candle wax drippings\" and \"punched-out\" lesions can be seen in patients with uveitis secondary to sarcoidosis."} {"text": "### Human: The number of doses recommended for oral Ty21a typhoid vaccine is: ### Assistant: The Ty21a vaccine is licensed for use in individuals > 5 years. A three dose regimen is recommended. Vaccine is administered on alternate days: 0n days 1, 3 and 5. Ref: Park 21st edition, page 215."} {"text": "### Human: The drug used in petit mal seizures and has a narrow spectrum of antiepileptic activity is ### Assistant: Ethosuximide and valproate are the drugs of choice for absence seizures and are more effective than lamotrigineReference: Katzung Pharmacology; 12th edition; Page no: 418"} {"text": "### Human: Which test is not useful in a patient with history of Syncopal attack? ### Assistant: Answer is C (PET Scan) : PET scan has- not been mentioned as a test to investigate syncope in Harrisons text. It is the single best answer of exclusion. Electrophysiological tests and 24 hour ECG Holter monitoring are indicated in patients where history suggests a cardiac disease. Upright tilt table testing is indicated for suspected neurogenic \/ vasodepressor syncope in patients with normal history \/ examination. Invasive Cardiac Electrophysiologic testing : provides diagnostic and prognostic information regarding Sinus Node function, AV conduction and Supraventricular and Ventricular arrhythmia. Holter monitor : provides 24 - 48 hours monitoring of ECG rhythm on an outpatient basis and is indicated for detection of conduction abnormalities. Upright tilt table testing : Is a test for vasodepressor \/ neurogenic syncope in patients with normal history \/ examination. In susceptible patients upright tilt at an angle between 60deg to 80deg for 30 to 60 minutes induces a vasovagal episode. Indications for Table Tilt test include : Recurrent syncope - Single syncope episode that caused injury - Single syncope event in high risk setting (pilot, commercial vehicle driver) Syncope The choice of diagnostic test should be guided by the history and physical examination For All patients Serum electrolytes Glucose Haematocrit"} {"text": "### Human: Magic syndrome is seen in: ### Assistant: \u201cMAGIC syndrome\u201d (Mouth and Genital ulcers with In\ufb02amed Cartilage) has been proposed to describe\u00a0patients with clinical features of both relapsing polychondritis and Behcet disease."} {"text": "### Human: Acute hemorrhagic conjunctivitis is caused by ? ### Assistant: Ans. is 'a' i.e., Enterovirus 70 Acute hemorrhagic conjunctivitis (AHC) may be caused by adenoviruses, but two enteroviruses, enterovirus 70 and coxsackie A24 variant, are the major causes."} {"text": "### Human: In a 24 year old man weight of the upper limb is transmitted to the axial skeleton by: ### Assistant: The axial skeleton shapes the longitudinal axis of the human body. Coracoclavicular ligament suspends the scapula from the lower one third of the clavicle and forms a strong bond between them. The weight of the upper limb is transmitted to the axial skeleton through this ligament. A fracture of the clavicle, medial to the attachment of this ligament leads to drooping of upper limb. Ref: TB of Anatomy and Physiology PR Ashalatha, 1st Ed, Page 110"} {"text": "### Human: Primary health care was proposed in 1978 at: ### Assistant: Answer: A. Alma ata. (Ref Park's Textbook of PSM 22nd\/Pg.832)The new approach to health care came into existence in 1978 following an international conference at Alma Ata (USSR).The Alma-Ata conference defines the primary health care as follows: Primary health care is essential health care made universally accessible to individuals and acceptable to them, through their full participation and at a cost the community and the country can afford.Millennium Development Summit was held in New York in 2000 where MDG are formulated.World Health Assembly is held in Geneva, which is a head quarter of WHO.Note: The term \"comprehensive health care\" (womb to tomb) was first used by Bhore committee in 1946."} {"text": "### Human: Gene involved in Cowden syndrome is ### Assistant: (B) PTENo\"Phosphatase and Tensin'' homolog (PTEN) - protein in humans encoded by the PTEN gene. Gene mutations promotes development of cancers.[?]Cowden's disease\/Multiple Hamartoma Syndrome:-Part of PTEN hamartoma tumor syndrome-An autosomal dominant syndrome-Trichilemmomas - Numerous tumors of hair follicles in face-Multiple hamartomatous polyps in GI tract, Lipomas, Granulomas-Very high risk of breast, Follicular endometrail carcinoma & thyroid carcinomaoTreatment: Bilateral mastectomies recommended-Contraindicated are mammography & other radiation exposure of breast tissue.-Mean age at presentation <10 years-Very high risk of breast, follicular carcinoma of thyroid & Endometrial carcinomaoPTEN (phosphatase and tensin homologue) is a membrane-associated phosphatase encoded by a gene on chromosome 10q23 that is mutated in Cowden syndrome, an autosomal dominant disorder marked by frequent benign growths, such as skin appendage tumors, and an increased incidence of epithelial cancers, particularly of the breast, endometrium, and thyroid.oPTEN acts as a tumor suppressor by serving as a brake on the PI3K\/AKT arm of the receptor tyrosine kinase pathway.oPTEN gene function is lost in many cancers through deletion, deleterious point mutations, or epigenetic silencing.SELECTED TUMOR SUPPRESSOR GENES & ASSOCIATED FAMILIAL SYNDROMES & CANCERS, SORTED BY CANCER HALLMARKS*Gene (Protein)Familial SyndromesAssociated CancersInhibitors of Mitogenic Signaling PathwaysAPC (Adenomatous polyposis coli protein)Familial colonic polyps and carcinomasCarcinomas of stomach, colon, pancreas; melanoma*. NF1 (Neurofibromin-1)Neurofibromatosis type 1 (neurofibromas and malignant peripheral nerve sheath tumors)Neuroblastoma, juvenile myeloid leukemia*. NF2 (Merlin)Neurofibromatosis type 2 (acoustic schwannoma and meningioma)Schwannoma, meningioma*. PTCH (Patched)Gorlin syndrome (basal cell carcinoma, medulloblastoma, several benign tumors)Basal cell carcinoma, medulloblastoma*. PTEN (Phosphatase and tension homologue)Cowden syndrome (variety of benign skin, GI, and CNS growths; breast, endometrial, and thyroid carcinoma)Diverse cancers, particularly carcinomas and lymphoid tumors*. SMAD2, SMAD4 (SMAD2, SMAD4)Juvenile polyposisFrequently mutated (along with other components of TGFb signaling pathway) in colonic & pancreatic CaInhibitors of Ceil Cycle Progression*. RB Retinoblastoma (RB) proteinFamilial retinoblastoma syndrome (retinoblastoma, osteosarcoma, other sarcomas)Retinoblastoma; osteosarcoma carcinomas of breast, colon, lung*. CDKN2A p16\/INK4a & pU\/ARFFamilial melanomaPancreatic, breast, and esophageal carcinoma, melanoma, certain leukemiasInhibitors of \"Pro-growth\" Programs of Metabolism and Angiogenesis*. VHL (Von Hippel Lindau (VHL) protein)Von Hippel Lindau syndrome (cerebellar hemangioblastoma, retinal angioma, renal cell carcinoma)Renal cell carcinoma*. STK11 (Liver kinase B1 (LKB1) or STK11)Peutz-Jeghers syndrome (GI polyps, GI cancers, pancreatic carcinoma and other carcinomas)Diverse carcinomas (5%-20% of cases, depending on type)*. SDHB, SDHD (Succinate dehydrogenase complex subunits B & D)Familial paraganglioma, familial pheochromocytomaParaganglioma"} {"text": "### Human: Supraventricular crest lies between- ### Assistant: Ans. is 'a' i.e., Pulmonary orifice and atrioventricular orifice Interior of right ventricle* It is anteroinferior chamber and projects to the left of right atrium. It is in contact with the sternum.* It is divided into -i) Rough inflowing part (ventricle proper)# It is developed from right half of primitive ventricle.# Its interior is rough due to the presence of muscular ridges known as trabeculae carneae, which are more prominent in apical region.# Trabeculae carneae are of 3 types: -1. Ridges - Linear elevations.Supraventricular crest - a ridge present between the pulmonary and atrioventricular orifices, extends downwards in the posterior wall of the infundibulum.2. Bridges -'Muscular elevations with fixed ends on ventricular walls, the center being free.Septomarginal trabecula: It is a specialized bridge which extends from the right of ventricularseptum to the base of anterior papillary muscle. It contains the right branch of atrioventricular bundle.3. Papillary muscles are conical projections of muscle fiber bundles. Their base is attached to the ventricular wall and the apex is attached to the chordae tendinae, which are further attached to the cusps of atrioventricular (AV) valves. There are three papillary muscles in the right and two in the left ventricle. They regulate closure of atrioventricular valves.ii) Smooth outflowing part (infundibulum or conus arteriosus)# Smooth outflowing part develops from mid portion of bulbus cordis and surrounds pulmonary valveSupraventricular crest (crista supraventriculars) or infundibuloventricular crest separates tricuspid (AV) orifice and pulmonary orifice, i.e. inlet and outlet parts."} {"text": "### Human: Which of the following is freely filtered by kidney across glomerular capillariesa) Albumin (across glomerular capillaries)b) Globulinc) Creatinined) HCO3 e) Glucose ### Assistant: Freely filterable substances by glomerulus\n-\u00a0\u00a0\u00a0\u00a0\u00a0\u00a0\u00a0 Water\n-\u00a0\u00a0\u00a0\u00a0\u00a0\u00a0\u00a0 Na+\n-\u00a0\u00a0\u00a0\u00a0\u00a0\u00a0\u00a0 Cl-\n-\u00a0\u00a0\u00a0\u00a0\u00a0\u00a0\u00a0 HCO3-\n-\u00a0\u00a0\u00a0\u00a0\u00a0\u00a0\u00a0 Inulin\n-\u00a0\u00a0\u00a0\u00a0\u00a0\u00a0\u00a0 Glucose\n-\u00a0\u00a0\u00a0\u00a0\u00a0\u00a0\u00a0 Creatinine\n\u00a0\n-\u00a0\u00a0\u00a0\u00a0\u00a0\u00a0\u00a0 Free Calcium or phosphate"} {"text": "### Human: Antibodies associated with GTCS in SLE : ### Assistant: Ans.A Anti neuronal ab (Ref: Harrison's Internal medicine 18th\/Table 319-1; 17th edition, P. 2076). Autoantibodies in Systemic Lupus Erythematosus (SLE) Antibody Prevalence, % Antigen Recognized Clinical Utility Antinuclear antibodies 98 Multiple nuclear Best screening test; repeated negative tests make SLE unlikely Anti-dsDNA 70 DNA (double-stranded) High titers are SLE-specific and in some patients correlate with disease activity, Anti-Sm 25 Protein complexed to 6 species of nuclear U1 RNA Specific for SLE; no definite clinical correlations; most patients also have anti Anti-RNP 40 Protein complexed to U1 RNA Not specific for SLE; high titers associated with syndromes that have overlap blacks than whites Anti-Ro (SS-A) 30 Protein complexed to hY RNA, primarily 60 kDa and 52 kDa Not specific for SLE; associated with sicca syndrome, predisposes to subacute cutaneous with decreased risk for nephritis Anti-La (SS-B) 10 47-kDa protein complexed to hY RNA Usually associated with anti-Ro; associated with decreased risk for nephritis Antihistone 70 Histones associated with DNA (in the nucleosome, chromatin) More frequent in drug-induced lupus than in SLE Antiphospholipid 50 Phospholipids, 2 glycoproteins 1 cofactor, prothrombin Three tests available ELISAs for cardiolipin and 2G1, sensitive prothrombin Antierythrocyte 60 Erythrocyte membrane Measured as direct Coombs' test; a small proportion develops overt hemolysis Antiplatelet 30 Surface and altered cytoplasmic antigens on platelets Associated with thrombocytopenia but sensitivity and specificity are not good; Antineuronal (includes anti-glutamate receptor) 60 Neuronal and lymphocyte surface antigens In some series a positive test in CSF correlates with active CNS lupus. Antiribosomal P 20 Protein in ribosomes In some series a positive test in serum correlates with depression or psychosis Also remember: Antibody Best screening test for Antibody ANA SLE Anti-Histone Drug-induced lupus Antinuclear antibodies SLE, nonspecific Anti-dsDNA; Anti-Sm Specific for SLE Anti-sm (Nelson's Paediatric Textbook) Marker of CNS lupus Anti-DNA Level of which correlates with disease activity Anti-Ro (SS-A) SACL, neonatal lupus Anti-Ro and Anti-La ie risk of nephritis (protective against nephritis) Antiphospholipid Hematological and Fetal Loss Antiribosomal protein Depression and Psychosis Anti-basement membrane Goodpasture syndrome Anticardiolipin, lupus anticoagulant SLE, antiphospholipid syndrome Anticentromere Limited scleroderma (CREST syndrome) Anti--Scl--70 (anti--DNA topoisomerase 1) Scleroderma (diffuse) Anti--SSA, anti--SSB (anti-Ro, anti--La) Sjogren syndrome Anti--Jo--1, anti--SRP, anti--Mi--2 Polymyositis, dermatomyositis Anti--TSH receptor Graves disease Anti--U1RNP (ribonucleoprotein) Mixed connective tissue disease c-ANCA (PR3-ANCA) Granulomatosis with polyangiitis (Wegener) IgA antiendomysial, IgA anti--tissue transglutaminase Celiac disease p-ANCA (MPO--ANCA) Microscopic polyangiitis, Churg--*JStrauss syndrome Anti--ACh receptor Myasthenia gravis Rheumatoid factor (antibody, most commonly IgM, specific to IgG Fc region), anti--CCP Rheumatoid arthritis Anti--glutamate decarboxylase Type 1 diabetes mellitus Anti--desmoglein Pemphigus vulgaris Anti--hemidesmosome Bullous pemphigoid Antimicrosomal, antithyroglobulin Hashimoto thyroiditis Antimitochondria! 1*< biliary cirrhosis Anti--smooth muscle Autoimmune hepatitis ALSO KNOW: LE cells Neutrophils with LE bodies seen in SLE, RA Penicillamine drug induced Hematoxilin bodies SLE endocarditis (Libmann-Sack) 'Wire-loop' lesions SLE type IV nephritis"} {"text": "### Human: Tonsillar fossa is bounded anteriorly by ### Assistant: Tonsilar fossa is bounded by Anterior - Palatoglossal fold containing palatoglossal muscle Posterior - Palatopharyngeal fold containing palato pharyngeal muscle Apex - Soft palate, where both arches meet Base - Dorsal surface of posterior one-third of tongue. (Ref: Anatomy of head, neck and brain. Vishram Singh. P 227 )"} {"text": "### Human: \"Castration anxiety\" is seen in which phase of Sigmond Freud's psychosexual stages of development ### Assistant: Phallic phase (3-5years): Male child develops Oedipus complex (sexual feeling towards mother) and will be afraid that father may castrate him for this termed as \"Castration anxiety\"."} {"text": "### Human: Shock causes: ### Assistant: Ans: A (Stagnant hypoxia)Ref: Pat GK, Hypoxia and Oxygen Therapy, In: Textbook of Medical Physiology, 2nd ed, Abuja Publishing House 2011: 94: 754-55Explanation:Refer the explanation of previous question"} {"text": "### Human: Calcium silicate based material is: ### Assistant: Mineral trioxide aggregate was developed by Dr Torabinejad in 1993.\u00a0\nIt contains tricalcium silicate, dicalcium silicate, tricalcium aluminate, bismuth oxide, calcium sulfate and tetracalcium aluminoferrite.\npH of MTA is 12.5.\n\nTextbook of Endodontics Nisha Garg 3rd Ed"} {"text": "### Human: Which of the following statements about Total Infra\u2013Venous Anaesthesia (TIVA) is true \u2013 ### Assistant: Total intravenous anaesthesia refers to a technique in general anaesthesia using a compination of agents given solely by IV route and in the absence of all inhalation agents.\nTIVA (Total intravenous anaesthesia) is produced by IV propoful. Propofol decreases the cerebral metabolic rate.\nPropofol does not impair renal function, does not trigger malignant hyperthermia and does not inhibit hypoxic pulmonary vasoconstriction."} {"text": "### Human: In Profound MR, IQ is ### Assistant: IQ range for categoriesICD-10DSM-IVMild50-6950-55 to 70Moderate35-4935-40 to 50-55Severe20-3420-25 to 35-40ProfoundBelow 20Below 20-25Reference: Oxford Textbook of Psychiatry; 3rd edition; Chapter 18; Learning Disability"} {"text": "### Human: Which of the following iodinated compound is present in a maximum concentration in the thyroid? ### Assistant: THYROID HORMONE SYNTHESIS AND SECRETIONThyroid hormones are synthesized in the colloid, near the apical cell membrane of the follicular cells. Catalyzed by the enzyme thyroidal peroxidase, iodide in the thyroid cell is oxidized to iodine.The iodine enters the colloid and is rapidly bound at the 3 position to tyrosine molecules attached to thyroglobulin, forming monoiodotyrosine (MIT).MIT is next iodinated at the 5 position, forming diiodotyrosine (DIT).Two DIT molecules then condense in an oxidative process (\"coupling reaction\") to form one thyroxine (T4) molecule.Some T3 is probably formed within the thyroid gland by condensation of MIT with DIT. A small amount of reverse T3 (rT3) is also formed.In the normal thyroid, the average distribution of iodinated compounds is 23% MIT, 33% DIT, 35% T4, 7% T3, and 2% reverse T3."} {"text": "### Human: Silicosis caused by ? ### Assistant: Ans. is 'd' i.e., Silica"} {"text": "### Human: Human development index includes - ### Assistant: Ans. is 'c' i.e., Life expectancy at bih According to 22nd\/e of Park, options 'd & e' are not the components of HDI. o Expected years of schooling (not adult literacy rate) and GNI per capita (not GDP) are the components of HDI."} {"text": "### Human: H. capsulatum, a dimorphic fungus, is found in soil heavily contaminated with bird droppings. Which of the following statements best describes the presence of the organism in tissue biopsies? ### Assistant: Histoplasma capsulatum is a dimorphic fungus that forms two types of spores: tuberculate macroconidia and microconidia. Inhalation of the microconidia transmits infection. Inhaled spores (microconidia) are engulfed by macrophages and develop into yeast forms. Most infections remain asymptomatic; small granulomatous foci heal by calcification. However, pneumonia can occur. The heterophile antibody test is useful for early diagnosis of infectious mononucleosis. The figure below illustrates the oval budding yeasts."} {"text": "### Human: Budding reproduction in tissue is seen in ### Assistant: Cryptococcus is a yeast, candida is a yeast-like fungus. Both are reproduced by budding Histoplasma is a dimorphic fungus Mucor and Rhizopus are produced by asexual means sporangiospores Reference: Textbook of Microbiology; Baveja; 4th edition"} {"text": "### Human: True about polio: ### Assistant: False 90-95% of polio infections are asymptomatic False Acute flaccid paralysis is seen in Polio & not spastic paralysis True Risk of paralytic polio is increased by tonsillectomy, strenous physical exercise, tooth extraction,Injection (intramuscular) False OPV drops are given to all children less than 5 yr age in pulse polio immunisation"} {"text": "### Human: The retention of dowel in a pin retained crown is increased by ### Assistant: Post should be at least 2\/3 the root length, parallel sided and serrated for increased retention in the canal"} {"text": "### Human: Which of the following drugs can be administered by subcutaneous route? ### Assistant: terbutaline can be given as s.c (0.25mg) in asthma."} {"text": "### Human: Mycosis fungoides which is not true: ### Assistant: Mycosis Fungoides:\nMycosis fungoides is synonymous with Cutaneous T cell lymphoma."} {"text": "### Human: Bacterial pyogenic parotitis affecting the parotid gland is most common after: March 2008 ### Assistant: Ans. C: Debilitation after major surgery Acute bacterial parotitis is now infrequent, but its historical impoance. Mumps and bacterial parotitis were differentiated by 1800, but neither was effectively treated. The moality rate for bacterial parotitis was 80%. Before antibiotics and intravenous administration of fluids were available, bacterial parotitis occurred in postoperative patients or other severely ill patients who became dehydrated and contributed to their demise as an incurable sepsis."} {"text": "### Human: Patient diagnosed as squamous cell intraepithelial lesion which of the following has the highest risk for progression to carcinoma; ### Assistant: High grade squamous intraepithelial lesion have a propensity to progress and become invasive, therefore need investigations and treatment Ref: Shaw Gynecology 17 e pg 410."} {"text": "### Human: Autosomal recessive polycystic kidney disease features include: ### Assistant: Ans. A. Can be diagnosed intrauterineExplanationClinical features of ARPKD (Autosomal recessive polycystic kidney disease) are:Classic ARPKD is generally diagnosed in utero or within the neonatal period and characterized by greatly enlarged echogenic kidneys in diseased fetuses. (Option a)Reduced fetal urine production may contribute to oligohydramnios and pulmonary hypoplasia.About 30% of affected neonates die shortly after birth due to respiratory insufficiency. Close to 60% of mortality occurs within the first month of life.In the classic group, most patients are bom with renal insufficiency and ESRD (End stage renal disease). (Option b)However, infants often have a transient improvement in their GFR; death from renal insufficiency at this stage is rare.Some patients are diagnosed after the neonatal stage and form the older group. Morbidity and mortality in this group often involve systemic hypertension, progressive renal insufficiency, and liver manifestations.The hallmarks of ARPKD liver disease are biliary dysgenesis due to a primary ductal plate malformation with associated periportal fibrosis, namely congenital hepatic fibrosis (CHF) and dilatation of intrahepatic bile ducts (Caroli's disease). CHF and Caroli's disease can then lead to portal hypertension exhibiting hepatosplenomegaly, variceal bleeding, and cholangitis.Some patients with the diagnosis of ARPKD at 1 year of age with nephromegaly exhibit slowly declining renal function over 20 years with only minimally enlarged kidneys at ESRD (Option c and e) and markedly atrophic kidneys following renal transplantation.The slow progression of renal disease is likely due to increasing fibrosis rather than the development of cysts.Systemic hypertension is common in all ARPKD patients, even those with normal renal function. (option d)"} {"text": "### Human: Postponing paying attention of conscious impulse or conflict is a mature defence mechanism known as - ### Assistant: Suppression is pushing of unwanted feelings into unconscious which has reached conscious awareness."} {"text": "### Human: The commonest site of perforation during colonoscopy is - ### Assistant: \u2022 Risks of colonoscopy: Perforation and hemorrhage\n\u2022 MC site of bleeding after colonoscopy: Stalk after polypectomy.\n\u2022 MC site of perforation during colonoscopy: Sigmoid colon\n\u2022 Perforation can be caused by excessive air pressure, tearing of the antimesenteric border of the colon from excessive pressure on colonic loops, and at the sites of electrosurgical applications"} {"text": "### Human: A 74-year-old woman develops occipital headache, vomiting, and dizziness. She looks unwell, her blood pressure is 180\/100 mm Hg, pulse is 70\/min, and respirations are 30\/min. She is unable to sit or walk because of unsteadiness. Over the next few hours, she develops a decline in her level of consciousness.For the above patient with altered level of consciousness, select the most likely diagnosis. ### Assistant: Cerebellar hemorrhage, when mild, may present with only headache, vomiting, and ataxia of gait. Patients may complain of dizziness or vertigo. The eyes may be deviated to the side opposite the hemorrhage. Nystagmus is not common, but an ipsilateral sixth nerve palsy can occur. This is the only type of intracerebral hemorrhage that commonly benefits from surgical intervention."} {"text": "### Human: Which is an uricosuric drug- ### Assistant: Ans. is 'b' i.e., Probenecid Uricosuric drugs are probenacid, sulfinpyrazone and benzbromarone."} {"text": "### Human: After an incised wound ,new collagen fibrils are seen along with a thick layer of growing epithelium.The approximate age of the wound is ### Assistant: Ref Robbins 9\/e p106 Incised wound for the formation of collagen and new tissue, thick layer of epithelial ,and granulation tissue formation along with new blood vessels requires atleast 4_5 days"} {"text": "### Human: After delivery, a 28 year old pre ecclamptic primigra has a soft uterus with moderate-to-heavy bleeding. There is no laceration and there is PPH diagnosed due to uterine atony. Which of the following is the best management option? ### Assistant: Ergot derivatives should not be given in patients with hypeension since this is classically known to increase blood pressure. A high BP may provoke a post paum ecclampsia Oxytocin is not orally effective . Prostaglandin F2-alpha is given IM."} {"text": "### Human: Specific poison for succinate dehydrogenase is ? ### Assistant: B i.e. Malonate"}{"text": "### Human: Most common widespread zoonotic disease in the world is - ### Assistant:

Leptospirosis is the most wide zoonotic disease in the world. Reference:Park&;s textbook of preventive and social medicine,K.Park,23rd edition,page no:291. <\\p>"}{"text": "### Human: Blood stained sputum may be the only symptom in? ### Assistant: Ans. is 'c' i.e., Adenoma bronchus"}{"text": "### Human: Inherited hyperammonemia is a result of deficiency of which enzyme of Krebs-Henseleit urea cycle? ### Assistant: Inherited hyperammonemias are a group of six diseases caused by inborn deficiencies of the enzymes of the Krebs-Henseleit urea cycle. The enzymes involved are: 1. N-acetyl glutamate synthetase 2. Arbamyl phosphate synthetase (CPS) 3. Ornithine transcarbamylase (OTC) 4. Argininosuccinic acid synthetase (citrullinemia) 5. Argininosuccinase deficiency 6. Arginase deficiency Most Severe Cases: In the most severe forms of the hyperammonemic disorders, the infants are asymptomatic at bih and during the first day or two of life, after which they refuse their feedings, vomit, and rapidly become inactive and lethargic, soon lapsing into an irreversible coma. Profuse sweating, focal or generalized seizures, rigidity with opisthotonos, hypothermia, and hyperventilation have been observed in the course of the illness. These symptoms constitute a medical emergency, but even with measures to reduce serum ammonia, the disease is usually fatal. Ref: Ropper A.H., Samuels M.A. (2009). Chapter 37. Inherited Metabolic Diseases of the Nervous System. In A.H. Ropper, M.A. Samuels (Eds), Adams and Victor's Principles of Neurology, 9e."}{"text": "### Human: Punishment for infanticide comes under - ### Assistant: Ref:Textbook of forensic medicine and toxicology (V.V.Pillay) 17th edition, page no.377 Infanticide refers to the deliberate killings of chlid below the age of one year.In England, Germany, UK, there is a clear-cut distinction between homicide and infanticides. In India, no such distinction exists, and accuse is punished under sec.302IPC,the same manner as in murder."}{"text": "### Human: Carotid artery stenosis screening invivo choice is - ### Assistant: Ans. is 'd' i.e., Doppler o Color doppler ultrasonography is the screening method of choice (not simple USG : option a is incorrect),o Carotid angiography is the investigation of choice for diagnosis,o MRI is the investigation of choice for demyelinating disorder, e.g. multiple sclerosis.Investigations in nervous systemo Investigation of choice for white matter disease - MRI (CT is second choice),o Investigation of choice of acoustic neuroma - GD enhanced MRIo Best investigation for all brain tumors - Contrast enhanced MRIo Investigation of choice for meningeal carcinomatosis - Contrast (gadolinium) enhanced MRI.o Investigation of choice for posterior fossa tumor - MRIo Investigation of choice for para meningeal rhabdomyosarcoma - MRIo Investigation of choice for nasopharyngeal angiofibroma - Contrast enhanced CT.o Investigation of choice for hydrocephalus in older patients and for low pressure hydrocephalus - MRI.o Investigation of choice for acute (<48hrs) subarachnoid hemorrhage (SAH) - Non-Contrast CT scan,o Investivation of choice to know the etiology of SAH - Four vessels digital substruction angiography.o Hallmark of SAH - Blood in CSF on lumbar puncture,o Investigation of choice for chronic SAH - MRI.o Primary procedure of choice for evaluating intracranial complications of acute head injury - CT scan,o Best modality for assessing fractures of the skull base, calvarium and facial bone - CT scan,o Investigation of choice for demyelinating disorders MRI.o Investigation of choice for AV malformation and aneurysm - Angiography (MRI angiography or CT angiography),o Investigation of choice for Craniospinal infection - MRIo Investigation of choice for all intrinsic spinal cord lesions; all causes of cord compression - MRI.o Procedure of choice for emergent evaluation of acute intracerebral hemorrhage - CT scan,o Investigation of choice for myelopathy - MRI.o Shape of epidural hematomas on imaging - Biconvex, hyperdense or mixed density.o Shape of acute subdural hematoma on imaging - Crescent-shaped, hyperdense or mixed density ,o Chronic subdural hematoma on imaging - Hypodense.o Most sensitive test for ischaemic stroke - Diffusion weighted (DW) MRI."}{"text": "### Human: Proposed guideline value for Radioactivity in drinking water is: ### Assistant: Key guidelines aspects of WHO recommended drinking water quality: Colour <15 true colour units (TCU) Turbidity <1 nephlometric turbidity units (NTU) pH: 6.5-8.5 Total dissolved solids (TDS) <500 mg\/Litre Zero pathogenic microorganisms Zero infectious viruses Absence of pathogenic protozoa and infective stages of helminthes Fluoride <1.0 ppm (0.5-0.8 ppm: Optimum level) Nitrates <45 mg\/Litre Nitrites <3 mg\/Litre Gross alpha radiological activity <0.5 Bq\/Litre (new guidelines -WHO) Gross Beta radiological activity <1.0 Bq\/Litre (new guidelines-WHO) Ref: Park 25th edition Pgno: 762"}{"text": "### Human: Lens develops from? ### Assistant: Ans. c (Surface ectoderm). (Ref. Human Embryology by IB Singh, 6th\/352)LENS# LENS develops from surface ectoderm.# Compared to the cornea, it has a much lower converging power of 17 diopters because of the relatively similar refractive indices of aqueous humor, lens, and vitreous body (cornea 42 D).# However, the lens is the only part of the refractive apparatus that is adjustable.# Adjustment to near vision (accommodation) therefore involves only the lens.# In addition, the lens functions as a UV filter for wavelengths between 300 and 400 nm and thus has a protective function for the macula.# Notching of the lens = \"pseudocoloboma\".# Wilson's disease can induce a yellow-green lens discoloration (chalcosis\/ sunflower cataract).# Granular gold deposits located under the lens (chrysiasis lends) can occasionally occur after prolonged treatment with gold preparations (e. g. in primary chronic polyarthritis ).# Other medications, e.g., amiodarone or chlorpromazine, are sometimes deposited in the lens.# In patients with cataract, glittering cholesterol crystals are not infrequently found in the lens (so-called Christmas tree\/ decoration cataract)."}{"text": "### Human: Palm of one hand is placed horizontally across the mouth and nostrils in ### Assistant: palmar strangulation Here the palm of one hand is placed horizontally across the mouth and nostrils, it's pressure being reinforced by placing the other palm on the top of it at right angles, the heel of the palm above pressing upon the front of the neck. Ref: TEXTBOOK OF FORENSIC MEDICINE AND TOXICOLOGY KRISHNAN VIJ FIFTH EDITION PAGE 134"}{"text": "### Human: In RDS all of the following statements are true except ### Assistant: Administration of 100% oxygen may increase the O2 load on fetal lung tissue. Hence, it is not given . Reference: GHAI Essential pediatrics, 8th edition"}{"text": "### Human: National program for prevention and control of Cancer, Diabetes, Cardiovascular disease and stroke (NPCDCS), true is - ### Assistant: Ans. is 'c' i.e., District hospital has specialised facilities Sational program for prevention and control of cancer, diabetes, cardiovascular diseases and stroke (NPCDCS1 Introduction Single centre fi- cancer, diabetes, cardiovascular disease, stroke. 100 districts in 21 states being covered in 1 1 th live year plan. 20,000 subcentres and 700 community health centres (CHCs) covered. o Activities as sub-centres Health promotion for behaviour and lifestyle change. Oppounistic screening of BP. blood glucose (strip method) in age > 30 years. Referral to CHC of cases of DM, HT. o Activities at CHCs : Diagnosis and management at NCD clinic. Home visits by nurse for bedridden cases. Referral to district hospital for complicated cases. o Activities as District hospital : Health promotion Screening of population > 30 years. Diagnosis and management of cardiovascular disease. Home-based palliative care for chronic, debilitating progressive patients. Specialized facilities. Urban health check-up shceme JOr diabetes and high BP : Screen urban slum population. Screen population > 30 years and pregnant females. Cancer control in NPCDCS : Regional cancer control scheme : Regional cancer centres to act as referral centres for complicated cases. Oncology wing development scheme. Decentralized NGO scheme : IEC activities and early cancer detection. lEC at central level. Research and mining."}{"text": "### Human: All of the following are ACTH independent Cushing syndrome except : ### Assistant: ACTH dependent Cushing syndrome is caused by : ACTH producing pituitary adenoma Ectopic ACTH Bronchial or pancreatic carcinoid Small cell cancer of lung Medullary carcinoma of thyroids Pheochromocytoma ACTH Independent Cushing syndrome is caused by : Adrenocoical adenoma\/ carcinoma Primary pigmented nodular adrenal disease Adrenal hyperplasia McCune Albright Syndrome Petrosal\/Peripheral ACTH ratio > 2 indicates the presence of a pituitary adenoma-secreting ACTH. NOTE: Overall the most common cause of Cushing's syndrome is Iatrogenic steroids but if asked regarding ACTH dependent cause then answer should be given as Pituitary adenoma."}{"text": "### Human: Hereditary spherocytosis is characterized by: March 2004 ### Assistant: Ans. D i.e. All of the above"}{"text": "### Human: Ketamine can be given by all of following routes except- ### Assistant: Ketamine has been administered intravenously, orally , rectally, intramuscularly and epidural route and very rare intranasal but not subcutaneous"}{"text": "### Human: All of the following are features of Devic's syndrome (neuromyelitis optica), except? ### Assistant: Devic's disease is a variant of multiple sclerosis. Fulminant demyelinating process is seen with another variant called Marburg's variant."}{"text": "### Human: A 7 years old boy presented with painful boggy swelling of scalp, multiple sinuses with purulent discharge, easily pluckable hairs and lymph nodes enlarged in occipital region. Which one of the following would be most helpful for diagnostic evaluation? ### Assistant: Ans. c. KOH mount"}{"text": "### Human: All the following are seen in Hyperparathyroidism except ### Assistant: Calvarial thickening is a feature of Hypoparathyroidism."}{"text": "### Human: Spore forming anaerobic gram positive bacilli ? ### Assistant: Ans. is 'b' i.e., Clostridia There are two medically impoant spore forming bacteria. Both of them are 'gram positive' `bacilli' :-Aerobic : BacillusAnaerobic (obligate anaerobes) : Clostridia."}{"text": "### Human: Drug of choice for Falciparum Malaria is:- ### Assistant: DOC for Falciparum Malaria - ACT.( Aemisinin based Combination therapy ) In Noh Eastern states:- ACT-AL co-formulated tablet of Aemether-20 mg \/ Lumefantrine - 120 mg. Primaquine 0.75 mg\/ kg Body weight on day 2. 2. In Other States:- ACT-SP -Aesunate 50 mg tablet for 3 days and Sulphadoxine (500mg) - Pyremethamine (25mg) tablets for 1 Day. Primaquine tablets should be given on Day 2."}{"text": "### Human: Factor V Leiden is caused due to a mutation that results in a substitution at position 506 glutamine to which of the following: ### Assistant: Mutation in factor V (called the Leiden mutation, after the city in the Netherlands where it was discovered) causes recurrent DVTs. The mutation results in a glutamine to arginine substitution at position 506 that renders factor V resistant to cleavage by protein C. As a result, an impoant antithrombotic counter-regulatory pathway is lost Ref: Robbins 8th edition Chapter 4."}{"text": "### Human: Kunkel's test is done to demonstrate presence of ................. in blood. ### Assistant: Carbon monoxide (CO)"}{"text": "### Human: In which case cystometric study is indicated - ### Assistant: Cystometric studies are urodynamic studies in which the pressure changes in the bladder is simultaneously measured with bladder filling and during micturition.\nIt helps in accurate assessment of detrusor and sphincter activity especially if a neurogenic abnormality is suspected.\nThough it is also used in stress incontinence but the aim here is to rule out any neurogenic cause."}{"text": "### Human: A 49-year-old female taking ibuprofen for increasing joint pain in her hands presents with increasing pain in her midsternal area. Gastroscopy reveals multiple, scattered, punctate hemorrhagic areas in her gastric mucosa. Biopsies from one of these hemorrhagic lesions reveal mucosal erosions with edema and hemorrhage. No mucosal ulceration is seen. ### Assistant: Gastritis is a nonspecific term that describes any inflammation of the gastric mucosa. Acute gastritis refers to the clinical situation of gastric mucosal erosions (not mucosal ulcers). Acute gastritis is also known as hemorrhagic gastritis or acute erosive gastritis. Acute gastritis is associated with the use of nonsteroidal anti-inflammatory drugs, such as aspirin, ibuprofen, and coicosteroids, and also with alcohol, chemotherapy, ischemia, shock, and even severe stress. Two types of stress ulcers are Curling's ulcers, seen in patients with severe burns, and Cushing's ulcers, seen in patients with intracranial lesions. Grossly acute gastritis appears as multiple, scattered, punctate (less than 1 cm) hemorrhagic areas in the gastric mucosa. This is helpful in differentiating acute gastritis from peptic ulcers, which tend to be solitary and larger. Microscopically the gastric mucosa from a patient with acute gastritis is likely to reveal mucosal erosions, scattered neutrophils, edema, and possibly hemorrhage. Acute gastritis. Erosion and complete effacement of the epithelium is observed. The residual glands, on the left, display regenerative changes with basophilic epithelium. Reference: Robbins & Cotran Pathologic Basis of Disease, 9edition"}{"text": "### Human: Sedimentation coefficient of Ig E is: ### Assistant: Ans : b 8S Ref: Ananthanarayanan 8th editionSvedberg unit - a sedimentation constant of 1 x 1013sec. Sedimentation unit is studied by ultracentrifugation - for diversity of antibody moleculeIgG -7IgA -7IgM -19Ig D -7IgE -8"}{"text": "### Human: DNA dependent RNA synthesis is inhibited by: ### Assistant: Rifampicin"}{"text": "### Human: Iliac crest involvement is common in which condition: March 2007 ### Assistant: Ans. A: Ankylosing Spondylitis The basic pathologic lesion of ankylosing spondylitis occurs at the entheses, which are sites of attachment to bone of ligaments, tendons, and joint capsules. Enthesopathy results from inflammation, with subsequent calcification and ossification at and around the entheses. Inflammation with cellular infiltration by lymphocytes, plasma cells, and polymorphonuclear leukocytes is associated with erosion and eburnation of the subligamentous bone. The process usually stas at the sacroiliac joints. Other enthesopathic sites include the iliac crest, ischial tuberosity, greater trochanter, patella, and calcaneum. Ankylosing spondylitis\/ AS\/ Bechterew's disease\/ Bechterew syndrome\/ Marie Strumpell disease\/Spondyloahritis is a chronic, painful, degenerative inflammatory ahritis primarily affecting spine and sacroiliac joints, causing eventual fusion of the spine. AS is a systemic rheumatic disease and one of the seronegative spondyloahropathies. About 90% of the patients express the HLA-B27 genotype. Men are affected more than women by a ratio in excess of 10:1. Typical signs of progressed AS are the visible formation of syndesmophytes on X-rays and abnormal bone outgrowths similar to osteophytes affecting the spine. Complete fusion results in a complete rigidity of the spine, a condition known as bamboo spine. In 40% of cases, ankylosing spondylitis is associated with iridocyclitis causing eye pain and photophobia. 4.1 AS is also associated with ulcerative colitis, Crohn's disease, psoriasis, and Reiter's disease Other complications are aoic regurgitation, Achilles tendinitis, AV node block and amyloidosis & restrictive lung disease. A clinical examination and X-ray studies of the spine, which show characteristic spinal changes and sacroiliitis, are the major diagnostic tools. The Schober's test is a useful clinical measure of flexion of the lumbar spine performed during examination."}{"text": "### Human: A 40-year-old man presents with a 2-week history of recur- rent oral ulcers, genital ulcers, intermittent arthritic pain of the knees, and abdominal pain. Physical examination reveals shallow ulcerations of the mucosa of the glans penis, as well as oral aphthous ulcers and conjunctivitis. Which of the following is the most likely diagnosis? ### Assistant: Behget disease is a systemic vasculitis characterized by oral aphthous ulcers, genital ulceration, and ocular inflammation, with occasional involvement of the nervous, gastrointestinal, and cardiovascular systems. The mucocutaneous lesions show a non-specific vasculitis of arterioles, capillaries, and venules. The cause of the necrotizing inflammation of small blood vessels is not known, but an association with specific HLA subtypes suggests an immune basis. Herpes (choice B) does not present with arthritis.Diagnosis: Behqet disease"}{"text": "### Human: About biotransformation not true ? ### Assistant: Ans. is 'b' i.e., Polar to less polar"}{"text": "### Human: In which phase of cell cycle, proof reading occurs? ### Assistant: EVENTS PHASE of cell cycle Proofreading S phase Most repairs G1 phase Mismatch repair G2 phase"}{"text": "### Human: The number of ATPs generated in krebs cycleare ? ### Assistant: Ans. is 'b' > b' i.e., 24 One turn of the TCA cycle, staing with acetyl CoA produces 10 ATPs. When the staing molecule is pyruvate, the oxidative decarboxylation of pyruvate, the oxidative decarboxylation of pyruvate yields 2.5 ATPs and therefore, 12.5 ATPs are produced when staing compound is pyruvate. Since, two molecules of pyruvate enter the TCA cycle when glucose is metabolized (glycolysis produces 2 molecules of pyruvate), the number of ATPs is doubled. Therefore, 25 ATP molecules, per glucose molecule, are produced when pyruvate enters the TCA cycle.Note : Previously calculations were made assuming that NADH produces 3 ATPs and FADH generates 2 ATPs. This will amount a net generation of 30 ATP molecules in TCA per molecule glucose and total 38 molecules from staing. Recent experiments show that these values are overestimates and NADH produces 2.5 ATPs and FADH produces 1.5 ATPs. Therefore, net generation during TCA is 25 ATPs and complete oxidation of glucose through glycolysis plus citric acid cycle yield a net 32 ATPs.Energy yield (number of ATP generated) per molecule of glucose when it is completely oxidized through glycolysis plus citric acid cycle, under aerobic conditions, is as follows :- Method ofNo of ATPsNo of ATPsPathwayStepEnzymeSource ATP formationgained perglucose(new calculation)As per oldcalculationGlycolysis1Hexokinase MinusMinus 1Do3Phosphofructokinase MinusMinus 1Do5Glyceraldehyde-3-p DHNADH Respiratory chain2.5 x 2 = 53 x 2 = 6Do61,3-BPGkinase ATPSubstrate level1 x 2 -- 21 x 2 = 2Do9Pyruvate kinaseATP Substrate level1 x 2 = 21 x 2 = 2Pyruvate to?PyruvateNADH Respiratory chain2.5x2= 53x2= 6Acetyl CoA Dehydrogenase TCA cycle3Isocitrate DHNADH Respiratory chain2.5x2= 53 x 2= 6Do4Alpha keto glutarate DHNADH Respiratory chain2.5x2= 53x2= 6Do5Succinate thiokinaseGTP Substrate level1 x 2 = 21 x 2 = 2Do6Succicinate DHFADH2Respiratory chain1.5x2= 32 x 2= 4Do8Malate DHNADH Respiratory chain2.5x2= 53 x 2= 6Net generation in glycolytic pathway 9 minus 2= 7 10 minus 2= 8Generation in pyruvate dehydrogenase reaction 5 * 6Generation in citric acid cycle 20 * 24Net generation of ATP from one glucose mole 32 * 38"}{"text": "### Human: Child with mild squint. Intrauterine, bih history, developmental history till date all normal. Corneal reflex normal. All other eye parameters normal except exaggerated epicanthal fold. Diagnosis ? ### Assistant: Ans. is 'a' i.e., Pseudostrabismus Epicantnic fold causes Pseudoesotropia (Pseudostrabismus)."}{"text": "### Human: Pain-sensitive intracranial structure is: ### Assistant: C i.e.Duramater"}{"text": "### Human: Not used for iron poison in: FMGE 09; NEET 13 ### Assistant: Ans. Penicillamine"}{"text": "### Human: Posterior relations of the head of the pancreas include all of the following, except: ### Assistant: The posterior relations of the head of the pancreas include the common bile duct, inferior vena cava and the right and left renal veins terminating into it. The first pa of the duodenum is an anterior relation of the head of the pancreas.Ref: Clinical Anatomy, By Richard S. Snell, 6th edition, Page 230; Clinical Anatomy (A Problem Solving Approach), By Neeta V Kulkarni, 2nd edition, Page 689."}{"text": "### Human: Which of the following events is involved in cleavage of the zygote during week 1 of development? ### Assistant: Cleavage is a series of mitotic divisions by which the large amount of zygote cytoplasm is successively paitioned among the newly formed blastomeres. Although the number of blastomeres increases during cleavage, the size of individual blastomeres decreases until they resemble adult cells in size."}{"text": "### Human: A female child with virilization, hypertension with low plasma renin diagnosis is : ### Assistant: 9] Congenital adrenal hyperplasia represents a group of autosomal recessive inherited, metabolic errors,\neach characterized by decency or total lack of a particular enzyme involved in the biosynthesis of cortical steroids,\nSteroidogenesis is then channelled into other pathways leading to increased production of androgens,\nwhich accounts for virilization.\nSimultaneously, the decency of cortisol results in increased secretion of ACTH resulting in adrenal hyperplasia.\nFollowing enzymes are implicated in the congenital adrenal hyperplasia 21 \u03b1 hydroxylase decency (partial or complete) 17 \u03b1 hydroxylase deficiency 11\u03b2 hydroxylase\ndeficiency 21 \u03b1 hydroxylase deficiency\n\u2022 Two forms of this deficiency include -\nA. Salt-wasting adrenogenital ism\nB. Simple Virilizing adrenogenital ism\nA) Salt-wasting syndrome (complete lack) The salt wasting syndrome results from complete lack of 21 hydroxylases.\nThere is no synthesis of mineralocorticoids and glucocorticoids in the adrenal cortex.\nDecreased mineralocorticoids causes marked sodium loss in the urine, resulting in hyponatremia, hyperkalemia, acidosis and hypotension.\nBecause of the enzyme block, there is increased formation of 17 - hydroxyprogesterone, which is then shunted into the production of testosterone.\nThis may cause virilism (pseudo-hermaphroditism) in female infants.\nThat is (XX) Female with 21 hydroxylase deficiency develops ovaries,\nfemale ductal structures and external male genitalia. But in the male child, the effect of increased testosterone will not be manifested at the time of birth.\nThe complete21 hydroxylase deficiency or salt wasting syndrome usually comes to light only after the birth because in utero the electrolytes and uids\ncan be maintained by maternal kidneys. Males with this disorder comes to clinical attention 5 to 15 days later because of salt losing crisis while\nfemales come to attention soon after the birth because of the virilization.\nB) Simple Virilizing adrenogenital syndrome (Partial deficiency) Occurs in individuals\nwith partial deficiency of 21 hydroxylases Less severe deficiency of mineralocorticoid, is sufficient for salt reabsorption, but the lowered glucocorticoid fails to cause\nfeedback inhibition of ACTH secretion.\nThus level of aldosterone is mildly reduced, testosterone is increased and ACTH elevated with resultant adrenal hyperplasia.\n11B hydroxylase deficiency-. Rare Leads to decreased cortisol and increased ACTH.\nThis, in turn, leads to the accumulation of DOC (deoxycorticosterone) and 11 deoxycortisol\nboth of which are strong mineralocorticoids. This results in increased sodium retention by the kidneys and hypertension, hypokalemia.\nPatients also develop virilization due to androgen excess. 17 \u03b1 hydroxylase deficiency - Patients with deficiency of 17 hydroxylases also have impaired cortisol production,\nincreased ACTH and secondary increased DOC.\nThese patients, however, cannot synthesize normal amount of androgens and estrogens.\nThis is because the gene that codes for 17 \u03b1 hydroxylase is the same for the enzyme in the adrenal cortex and the gonads and the decency is same in both organs.\nBecause of decreased sex hormones, genotypic females develop primary amenorrhoea and fail to develop secondary sex characteristics while genotypic\nmales will present as pseudohermaphrodite.\n3 \u03b2 hydroxylase deficiency 3-\u03b2 hydroxylase deficiency is a rare genetic disorder of steroid biosynthesis that results\nin decreased production of all three groups of adrenal steroids which include mineralocorticoid, glucocorticoid and sex steroids.\nDecreased mineralocorticoid secretion\nresults in varying degrees of salt wasting in both males and females.\nDecent androgen production results in ambiguous genitalia in males\nThe usual presentation is a male child with ambiguous genitalia and severe salt wasting. Sometimes it may also occur in females"}{"text": "### Human: 'Patterned' abrasion is variety of: ### Assistant: Ans. is 'b' ie pressure abrasion (Ref: Parikh; 6\/e, p4.3).An abrasion is a superficial injury involving only the superficial layers of the skin (epidermis only), caused by friction and \/ or pressure between the skin and some rough object or surface. It bleeds very slightly, heals rapidly in a few days and leaves no scar.They can be classified into 3 types, depending on the manner they are created.ScratchIs a linear injury produced by a sharp object, such as pin, thorn or finger nail.Graze (Sliding, scraping or grinding abrasion).Is produced when a broad surface of skin slides against a rough surface.It is also known as brush or friction burn because it is caused by the frictional force and resembles a burn after dryingIt is commonly found in road traffic accidents.Imprint, Pressure or contact abrasion.Is produced as a result of direct impact or pressure of or contact with some object at right angles to the skin surface.The cuticle is crushed, and the object marks a reproduction of its shape and pattern on the to the skin, therefore it is known as patterned abrasion.Example are ligature marks in hanging and strangulation, nail and thumbs marks in throttling, teeth marks in biting, radiator, grill or tyre mask in vehicular accidents."}{"text": "### Human: Pimozide belongs to class of: ### Assistant: Thioxanthenes are flupenthioxol. Butyrophenones are haloperidol, trifluperidol , penfluridol. Phenothiazines are Chlorpromazine, triflupromazine , thioridazine, flufenazine. Promazine belongs to the group of other of other hetero cyclics that is diphenyl butyl piperidine. Ref: KD Tripathi 8th ed."}{"text": "### Human: Following poisoning retards putrefaction: NEET 13 ### Assistant: Ans. Arsenic"}{"text": "### Human: True statements about parotid gland ### Assistant: The duct runs forwards for a sho distance between the buccinator and the oral mucosa. Finally, the duct turns medically and opens into the vestibule of the mouth(gingivobuccal vestibule)opposite the crown of the upper second molar tooth. Notes: Parotid abscess may be caused by spread of infection from the opening of parotid duct in the mouth cavity. REF.BDC VOL.3,Sixth edition pg 110"}{"text": "### Human: Which of the following statement is false about hydrocele? ### Assistant: Treatment of Hydrocele Congenital Hydrocele: Usually spontaneous obliteration by 2 years age. If persists then Herniotomy. Treatment of Vaginal Hydrocele 1. Small hydrocele: Lord's procedure (Plication of sac)Q 2. Medium hydrocele: Jaboulay's procedure (Eversion of sac)Q 3. Large hydrocele: Excision of sacQ"}{"text": "### Human: A normally developing 10 months old child should be able to do all of the following except \u2013 ### Assistant: A child starts trying to build a tower of cubes (2 cubes) by the age of 1 year.\n\nAbout other options\n\nChild can stand with support by 9 months.\nChild can play peak-a-boo game by 10 months.\nChild can pick up a pellet with thumb and index finger (pincer grasp) by 9 months."}{"text": "### Human: Lines of Blaschko&;s are along ### Assistant: Blaschko&;s lines, are lines of normal cell development in the skin. These lines are invisible under normal conditions."}{"text": "### Human: All the teeth in patient\u2019s mouth are similarly affected as shown in the radiograph. What is the most likely anomaly? ### Assistant: Dentinal dysplasia\nDentinal dysplasia is an inherited condition affecting both dentitions. It is divided into two types: Type I (radicular dentinal dysplasia) and Type II (coronal dentinal dysplasia). It occurs less frequently than dentinogenesis imperfecta.\nClinical features\nType I is the more frequent type. Teeth erupt normally with no obvious abnormality but rapidly develop mobility resulting from limited root support. In type II, the deciduous teeth are similar to those in dentinogenesis imperfecta, whereas the permanent teeth appear normal.\nRadiological signs\nIn Type I, all the teeth, except the canines, have extremely short roots. The roots are conical or blunted. There is pulpal obliteration and the inferior aspects of the crown exhibit linear crescent-shaped radiolucent lines. Periapical radiolucencies are commonly seen associated with non-carious teeth in Type I dentinal dysplasia. In Type II dentinal dysplasia, obliteration of the pulp occurs post eruption in the deciduous dentition. In the permanent dentition, there is alteration in the configuration of the pulp resulting in enlarged pulp chambers extending well down into the root. The pulp contains multiple calcifications.\nRef: Interpreting Dental Radiographs by Keith Horner, Jon Rout and Vivian E Rushton"}{"text": "### Human: Middle aged female with mass in sella turcica hormone increased is - ### Assistant: The sella turcica is a saddle shaped depression of the sphenoid bone.\nIt forms the caudal border of the pituitary gland. Sella turcica is an anatomically complex area with a number of different potential pathologies especially neoplastic process.\nPathologies of sella turcica can lead to important clinical presentations such as hormonal imbalances from pathologies affecting the pituitary gland and neurological symptoms from the mass effect.\nSella turcica is critically located as several structures pass close to it due to compression of the adjacent structures.\n\nImportant anatomical structures related to sella\n\nOptic chiasm\nPituiatry gland\nInternal carotids\u200b\nGavernous sinus and cranial nerves\nSphenoid sinuses"}{"text": "### Human: Hyoid bone fracture most common occurs in ? ### Assistant: Ans. is 'a' i.e., Manual strangulation As manual strangulation (throttling) is among the most violent form of asphyxia, hyoid fracture and other injury to neck structures is more common."}{"text": "### Human: \"Acute Meningoencephalitis\" is caused by: ### Assistant: Nageleria"}{"text": "### Human: All are major symptoms of sinusitis except ### Assistant: According to Rhinosinusitis task force definition of sinusitis,Major criteria-Facial pain, pressure, congestion, nasal obstruction, nasal\/postnasal discharge, hyposmia and feverMinor criteria- Headache, halitosis, and dental painRef: Hazarika; 3rd ed; Pg 328"}{"text": "### Human: Most sensitive nerve fiber to hypoxia ### Assistant: Question repeated"}{"text": "### Human: Nasal septum is formed except by ### Assistant: The bony pa is formed by: 1.Vomer 2.Perpendicular plate of ethmoid 3.Nasal spine of frontal bone 4.Rostrum of sphenoid 5.Nasal crest of nasal, palatine and maxillary bones. NOTES: The cailaginous pa is formed by; 1.Septal cailage 2.Septal process of inferior nasal cailages The cuticular pa is formed by; 1.Fibrofatty tissue Ref.BDC volume3,Sixth edition pg 240"}{"text": "### Human: Metoprolol is preferred over Propranolol as it: September 2010 ### Assistant: Ans. A: Is more potent in blocking beta-1 receptors Metoprolol is cardioselective beta blocker, more potent in blocking beta-1 than beta-2 adrenergic receptors."}{"text": "### Human: All will predispose to atherosclerosis, except ### Assistant: Elevated plasma levels of\u00a0 homocysteine are\u00a0associated with increased risk of atherosclerosis,\u00a0thrombosis\u00a0and\u00a0hypertension.\nLipoprotein A\u00a0inhibits fibrinolysis, therefore predisposes to atherosclerosis.\nAtherosclerosis (Greek\u00a0 athere-mush)\u00a0 is\u00a0a\u00a0complex disease characterized by thickening or\u00a0hardening\u00a0of\u00a0arteries due\u00a0to\u00a0the accumulation of lipids\u00a0(particularly cholesterol, free,\u00a0and\u00a0esterified), collagen, fibrous tissue, proteoglycans,\u00a0calcium deposits, etc. in the inner arterial wall.\u00a0\nSatyanarayana, Ed 3, Pg No 152"}{"text": "### Human: Most serious complication seen in other eye after traumatic injury to one eye: March 2005, September 2008 ### Assistant: Ans. C: Sympathetic ophthalmia Sympathetic ophthalmia (SO) is a condition in which serious inflammation attacks the sound eye after injury (including intraocular surgeries) to the other. It is the most dreaded complication of unilateral severe eye injury, as it can leave the patient completely blind. Symptoms may develop from days to several years after a penetrating eye injury. Sympathetic ophthalmia is thought to be an autoimmune inflammatory response toward ocular antigens, specifically a delayed hypersensitivity to melanin-containing structures from the outer segments of the photoreceptor layer of the retina. It is thought that Louis Braille, who injured his left eye as a child, lost vision in his right eye due to SO Floating spots and loss of accommodation are among the earliest symptoms. The disease may progress to severe iridocyclitis with pain and photophobia. Commonly the eye remains relatively painless while the inflammatory disease spreads through the uvea. The retina, however, usually remains uninvolved. Papilledema, secondary glaucoma, vitiligo and poliosis of the eyelashes may accompany SO. Diagnosis is clinical, seeking a history of eye injury. An impoant differential diagnosis is Vogt-Koyanagi-Harada syndrome (VKH), which is thought to have the same pathogenesis, without a history of surgery or penetrating eye injury. Definitive prevention of SO requires prompt (within the first 7 to 10 days following injury) enucleation of the injured eye. Evisceration--the removal of the contents of the globe while leaving the sclera and extraocular muscles intact--is easier to perform, offers long-term orbital stability, and is more aesthetically pleasing. But evisceration may lead to a higher incidence of SO compared to enucleation. Immunosuppressive therapy is the mainstay of treatment for SO. When initiated promptly following injury, it is effective in controlling the inflammation and improving the prognosis."}{"text": "### Human: Least commonly seen in conduct disorder seen in girls:- ### Assistant: Conduct disorder - Persistent pattern of antisocial behavior in which the individual repeatedly breaks social rules and carries out aggressive acts.in conduct disorder they do it deliberately usually characterized by aggression and violation of the rights of others boys with conduct disorder show physical & relationship aggression but In girls relationship aggression is more predominant than physical aggression children with conduct disorder usually have behaviors characterized by aggression to persons or animals, destruction of propey, deceitfulness or theft, and multiple violations of rules, such as truancy from school."}{"text": "### Human: Familial adenomatous polyposis is characterized by all except ### Assistant: Familial adenomatous polyposis (FAP) Is a rare, inherited condition caused by a defect in the adenomatous polyposis coli (APC) gene - located in 5q 21 chromosome. FAP is autosomal dominant- >50% chance of inheritance in family members Has > 100 adenomatous polyps 100% RISK of malignancy Polyp formation - by 15 yrs. 100% develop malignancy - by 40 yrs. TOC: Total proctocolectomy + ileal pouch-anal anastomosis (TPC + IPAA) M\/C\/C of death after TPC = Peri-ampullary Ca (polyps in duodenum - periampullary region) Two variants of FAP Gardner's syndrome Osteoma (mandible) Congenital hyperophy of the retinal pigmented epithelium (CHRPE) lesions Sebaceous cysts Benign lymphoid polyposis of ileum Desmoid tumors Supernumerary teeth Turcot's syndrome: Brain tumors Medulloblastoma - m\/c associated with FAP Glioblastoma multiforme - m\/c associated with HNPCC"}{"text": "### Human: In Steiner's analysis, SNA demonstrates: ### Assistant: Relating the Maxilla to the Skull: The angle SNA is formed by joining the lines S-N and N-A . The mean\u00a0reading for this angle is 82\u00b0.\nSteiners analysis demonstrates the position of maxilla and mandible in relation to the cranial base.\u00a0\nIf the angular reading is more than 82\u00b0, it would indicate a relative forward positioning or protrusion\u00a0of the maxilla. Conversely, should the reading be less than 82\u00b0, it would indicate a relative backward or recessive location of the maxilla."}{"text": "### Human: . Sulphasalazine is NOT used for the treatment of- ### Assistant: Sulhasalazine is not used for treatment of sarcoidosis As a treatment for sarcoidosis, these drugs are most likely to be effective in people who have skin symptoms or a high level of calcium in their blood. Hydroxychloroquine (Plaquenil) and chloroquine (Aralen) are antimalarial drugs that are used to treat sarcoidosis Ref Davidson 23rd edtion pg 981"}{"text": "### Human: Post traumatic stress disorder is differentiated from other anxiety disorders by which of the following symptoms? ### Assistant: This question is not about the diagnostic features of PTSD. This is about the differential diagnosis. PTSD is an anxiety disorder and it shares symptoms with other disorders. Impoant features in distinguishing the condition are given below. Differential diagnosis: Point 1: Etiologically significant trauma should be present for a diagnosis of PTSD. E.g. A traumatic event in the past. Point 2: 'Intentionality' or 'aboutness' is an impoant factor for PTSD. Nightmares, flashbacks or reliving experiences should be related to the past event. PTSD concerns memory - Intrusion of past stressors into the present. Point 3: Avoid a 'stimulus' or 'activity' that provokes the memory of the past event. In the above question, only reliving experience is mentioned as connected to the past experiences. Hence, that is the answer for this question. All other symptoms can be see in other anxiety disorders. Ref: Kaplan & Sadock's, Comprehensive Textbook of Psychiatry, 9th Edition, Page 2659"} {"text": "### Human: Which muscle is responsible for unlocking of knee? ### Assistant: Ans. A PopliteusRef: BDC, vol II pg. 155* Quadriceps femoris is the main and only extensor of knee.* It produces locking action as a result of medial rotation of the femur during the last stage of extension.* To reverse this lock popliteus muscle comes into action and does so by the lateral rotation of femurRemember: Lock is: Quadriceps femoris muscle and, Key is popliteus muscle.Muscles producing movements at the knee jointMovementPrincipal musclesA. Flexion * Biceps femoris* Semitendinosus* SemimembranosusB. ExtensionQuadriceps femorisC. Medial rotation of flexed leg* Popliteus* Semimembranosus* SemitendinosusD. Lateral rotation of flexed leg * Biceps femorisExtra Mile* Quadriceps femoris incudes: Rectus femoris, Vastus Lateralis, Vastis Medialis, Vastus Intermedius.* Rectus femoris is also known as \"kicking muscle\""}{"text": "### Human: Polymorphic ventricular tachycardia can occur when terfenadine(antihistaminic): ### Assistant: Torsades de pointes (polymorphic ventricular tachycardia) occurs\u00a0when terfenadine is taken in higher doses or when hepatic\u00a0metabolism is impaired by disease or by drugs which inhibit the\u00a0cytochrome P450 responsible for metabolism of terfenadine.\u00a0Drugs include erythromycin, clarithromycin, ketoconazole and\u00a0itraconazole. Azithromycin and fluconazole that are excreted\u00a0unchanged in the urine have not been associated with impaired\u00a0metabolism of terfenadine. Terfenadine blocks the delayed rectifier\u00a0potassium channels and prolongs cardiac repolarization and the\u00a0QT interval."}{"text": "### Human: Which component transfers four protons: ### Assistant: A i.e. NADH-Q Oxidoreductase; C i.e. Cytochrome C-Q oxidoreductase"}{"text": "### Human: All of the following are true about Asthma, Except: ### Assistant: Answer is D (Small airways are not involved): Asthma involves both large and small airways but alveoli are not involved Charcot Leyden crystals may be seen in sputum: Pathology by woolf lsr (1998)\/434 Sputum from Asthmatics show ceain characteristic features, including charcot leydencrystas The microscopically identifiable features described in sputum are three 'C's Charcot Leyden crystals - Derived from granules of eosnophils andfbund only in Asthma Curshmann spirals - Curiously twisted casts of airways : Whorls of shed epethelium Creola bodies - Clumps of cells or isolated metaplastic cells Reversible Airflow obstruction is a charachteristic feature (Davidsons 18th\/55, 326) Reversible Bronchoconstriction is the hallmark of Asthma The charachteristic feature of Asthma is Intermittent and Reversible airflow obstruction. Narrowing of airways is usually reversible, but in some patients with chronic asthma there may be an element of irreversible airflow obstruction Asthma involves both large and small airways Asthma involves Large and small airways hut not alveoli - 'Allergic Diseases: Diagnosis Treatment' 3rd\/ 120 The large and small airways are both involved in Asthma -'Childhood Asthma: Diagnosis, Treatment & Management' r\/15 Intermittent Asthma responds better to bronchodilator therapy than persistant Asthma Bronchodilators are effective drugs for the treatment of acute exacerbations or intermittent episodes of Asthma but they are not sufficient to control asthma in patients with persistant symptoms (persistant asthma) Bronchodilators are not sufficient to control Asthma in patients with persistant symptoms'"}{"text": "### Human: In emergency, which aery can be safely ligated ? ### Assistant: Ans. is 'a' i.e. Internal iliac"}{"text": "### Human: Late metabolic acidosis is seen in- ### Assistant: Ans. is 'b' i.e., Preterm baby getting cow milk Late metabolic acidosis (LMA) o Late metabolic acidosis is a special type of acidosis in apparently healthy premature infant. o This hyperchloremic acidosis appeares during second and third week of life, and resolves spontaneously within a couple of weeks. Definition \"An acidosis occuring after second day of life, in which base excess values are lower than -5meq\/L on two consecutive estimations done at least 24 hours apa\". Etiology o Prematurity is the most impoant single predisposing factor in the development of LMA. o The incidence varies considerably depending upon the gestational maturity and protein content of feeding formula. o In preterm neonates fed on a modified cow's milk the risk of developing LMA is very high."}{"text": "### Human: Chemotherapeutic drugs can cause? ### Assistant: Ans. is 'c' i.e., Both necrosis and apoptosis o Injurious stimuli, e.g., radiation, cytotoxic anticancer durgs, heat and hypoxia, can induce apoptosis if the insult is mild, but large doses of same stimuli result in necrotic cell death."}{"text": "### Human: Which of the following nerve is involved in Frey's syndrome? ### Assistant: Syndrome (Gustatory Sweating) aka auriculotemporal syndrome: Frey's syndrome arises as a complication of parotid surgery usually manifesting several months after the operation. lt is characterised by sweating and flushing of the preauricular skin during mastication causing nuisance to the person or social embarrassment. It is the result of damage to auriculotemporal nerve and aberrant innervation of sweat glands by parasympathetic secretomotor fibres which were destined for the parotid. Now instead of causing salivary secretion from the parotid, they cause secretion from the sweat glands. The condition can be treated by tympanic neurectomy which intercepts these parasympathetic fibres at the level of middle ear. Some people like to place a sheet of fascia lata between the skin and the underlying fat to prevent secretomotor fibres reaching the sweat glands. Generally, no treatment other than reassurance is required in most of these patients. Also known as Crocodile tears (gustatory lacrimation): There is unilateral lacrimation with mastication. This is due to faulty regeneration of parasympathetic fibres which now supply lacrimal gland instead of the salivary glands. It can be treated by section of greater superficial petrosal nerve or tympanic neurectomy."}{"text": "### Human: Opening of mouth is caused by: ### Assistant: Ans. A Lateral pterygoidRef: Grays, 41st ed. pg. 507-508* The primary function of the lateral pterygoid muscle is to pull the head of the condyle out of the mandibular fossa along the articular eminence to protrude the mandible.* The effort of the lateral pterygoid muscles acts in helping lower the mandible and open the jaw whereas unilteral action of a lateral pterygoid produces contralateral excursion (a form of mastication), usually performed in concert with the medial pterygoids.* Unlike the other three muscles of mastication, the lateral pterygoid is the only muscle of mastication that assists in depressing the mandible i.e. opening the jaw. At the beginning of this action it is assisted by the digastric, mylohyoid and geniohyoid muscles."}{"text": "### Human: All are true about streptococcus Except ### Assistant: Mucoid colonies are due to the production of the capsule of hyaluronic acid not due to M-protein. Growth characteristic of Strep. pyogenes: On blood agar, virulent stains form matt or mucoid colonies while avirulent from glossy colonies. M, T, R are proteins found on the outer pa of the cell wall and forms the basis of Griffith classification. M protein - acts as virulence factor and antibody against it is protective. T and R protein - No relation to virulence Ref : 208"}{"text": "### Human: What is monitored in a patient of Pre eclamsia ? ### Assistant: Ans. is 'd' i.e. All of the abov Investigations in a case of Pre eclampsia* URINE for proteins\/albumin. 24 hours urine protein* CBC: There is hemoconcentration so HB values are false elevated. Low platlets indicate HELLP syndrome* Sr uric acid: It is a biochemical marker of preeclampsia. Raised levels (>4.5 mg\/dl) indicate renal involvement and also correlate with severity of preeclampsia, volume contraction and fetal jeopardy.* LFT: SGOT, SGPT, Bilirubin* RFT: Sr creatinine* Coagulation profile may be required in severe cases:i) BTii) CTiii) PT, APTTiv) Fibrinogen levelsv) FDP"}{"text": "### Human: In a young female of reproductive age an absolute contraindication for prescribing oral contraceptive pills is: ### Assistant: Ans-D i.e., Impaired liver function Repeat Q No 173 (Nov. 2004)Absolute contraindications include:A personal h\/O thromboembolic venous, arterial or cardiac disease or severe or multiple risk factor for these. Transient cerebral ischaemic attacks without a headache.Infective hepatitis, until 3 months after liver function tests has become normal, and another liver disease including disturbances of hepatic excretion e.g. cholestatic jaundice, Dubin Johnson and Rotor syndromes.A migraine, if there is a typical aura, focal features or if it is severe and lasts > 72 hours despite t\/t or is treated with an ergot derivative.Carcinoma of the breast or the genital tractOther conditions including SLE, porphyria, following the evacuation of a hydatidiform mole (until urine and plasma gonadotropin concentrations are normal), undiagnosed vaginal bleeding.Relative contraindications of OCP'sThe family history of venous thromboembolism, arterial disease or a known prethrombotic condition e.g. Factor V Leiden (pretreatment coagulation investigation is advised).Diabetes mellitus which may be precipitated or become more difficult to control (avoid if there are diabetic complications).Hypertension (avoid if B.P. exceeds 160\/100)Smoking > 40 Cigarettes per day (15 Cigarettes\/day enhances the risk of circulatory disease and constitutes an absolute contraindication for women over 35 years.Long-term immobility (e.g. due to Leg plaster, confinement to bed.Breastfeeding (until weaning or for 6 months after birth).Obesity"}{"text": "### Human: A tracheostomised patient, with poex tracheostomy tube, in the ward, developed sudden complete blockage of the tube. Which of the following is best next step in the management? ### Assistant: Immediate removal of the tracheostomy tube is best next step in the management in the above case -Suction of tube with sodium bicarbonate is done to avoid the blockage. -Suction of the tube with saline can't be used when there's a complete obstruction. - Jet ventilation is done below the level of larynx, when we're operating on the larynx."}{"text": "### Human: All of the following are well recognised predisposing factors for adult respiratory distress syndrome, except: ### Assistant: Status asthmaticus is an example of type 2 respiratory failure while ARDS is type 1 respiratory failure. All other choices lead to cytokines release and damage the alveoli leading to ARDS."}{"text": "### Human: Gottron papules is seen in ? ### Assistant: Ans. is 'a' i.e., Dermatomyositis"}{"text": "### Human: Which among the following is the BEST irrigating fluid during ECCE? ### Assistant: BSS plus a balanced salt solution enhanced with bicarbonate, dextrose and glutathione. It is specially used for surgeries requiring prolonged irrigation such as phacoemulsification, pars plana vitrectomy and automated extracapsular cataract extraction. Balanced sterile solution is a sterile physiologically balanced irrigating fluid. It is the most frequently used solution during ophthalmic surgery to keep the cornea from drying out. BSS plus solution causes less corneal edema and endothelial cell damage following vitrectomy. The glutathione protects against depletion of endothelial ATP levels and maintains the integrity of the cell membrane by counteracting endogenous and exogenous oxidative agents. Ref: Surgical Technology for the Surgical Technologist: A Positive Care Approach By Association of Surgical Technologists page 223. Ocular Toxicity of Intraoperatively Used Drugs and Solutions By Rudolph Marie Matheus Antonius Nuijts page 16. Clinical Ophthalmology: Contemporary Perspectives, 9\/e By Gupta page 43."}{"text": "### Human: Acute Gouty ahritis is seen early in treatment following - ### Assistant: All of the above Rapid lowering of urate level in chronic gout, by any means, may precipitate an attack of acute gout, probably by causing the dissolution of tophi. o It is therefore usual to give prophylactic suppressive treatment with indomethacine, colchicine or steroid cover during first 2 months of allopurinol or uricosurics. o Amongs the given options, allopurinol, probenecid and Rasburicase are the drugs that lower serum urate level. So, all three can precipitate acute gout. However, among these three, allopurinol is used most commonly therefore is the best answer here."}{"text": "### Human: Chandler's index is associated with: September 2006 ### Assistant: Ans. B: Hook worm Morbidity and moality from hookworm infection depend much on the worm load. Chandler worked out an index on the basis of an average number of hookworm eggs per gram of feces for the entire community, Chandler's index is still used in epidemiological studies of hookworm disease. By this index, worm loads in different population groups can be compared and also the degree of reduction of egg output after mass treatment."}{"text": "### Human: Heparin is the commonly used anticoagulant in cardiac surgery. All of the following are true about heparin except- ### Assistant: Ans. is 'a' Weakest acid found in living organism Heparin is a direct acting anticoagulant Chemical nature and preparationHeparin is a sulfated mucopolysaccharide which occurs in the secretory granules of mast cells.It is the strongest organic acid in the body and in the solution it carries an electronegative charge.It is prepared commercially from a variety of animal tissues (generally porcine intestinal mucosa or bovine lung).Mechanism of Action of HeparinHeparin acts by activating antithrombin which is a naturally occuring inhibitor of activated coagulation factors of intrinsic and common pathway. 'Adverse effects of HeparinBleedingHeparin induced thrombocytopeniaOccurs more commonly in heparin derived from bovine lung.Heparin induced thrombocytopenia should be suspected in whom the platelet count falls by 50% or more after starting heparin and usually occurs 5 or more days after starting therapy.In patients with heparin induced thrombocytopenia following drugs should be substituted.Danaparoid sodiumHirudinArgatrobanOsteoporosis - (most frequently seen in pregnancy)Hypersensitivity reactions & skin necrosis"}{"text": "### Human: One of the following is the first enzyme to be released at the site of wound? ### Assistant: Ref - Krishan Vij textbook of forensic medicine and toxicology 5e pg - 202"}{"text": "### Human: Sign of diabetic ketoacidosis is- ### Assistant: Physical findings of diabetic ketoacidosis are tachycardia, dehydration\/hypotension, hypothermia, tachypnea\/Kussmaul respirations\/ respiratory distress, abdominal tenderness (may resemble acute pancreatitis or surgical abdomen), lethargy\/obtundation\/cerebral edema\/possibly coma. Reference : page 2418 Harrison's Principles of Internal Medicine 19th edition"}{"text": "### Human: Which of the following drug is used in scabies as single oral dose agent: March 2011 ### Assistant: Ans. C: Ivermectin Ivermectin is used as single oral dose of 200 microgram\/ kg body weight It can be repeated after 2 weeks It is indicated in epidemics of scabies in orphanages and Norwegian scabies Scabies: Incubation period: 4 weeks Pathognomic lesion: Burrow, which lies in stratum corneum MC site in infants: Scalp, face Most severe form: Norwegian scabies Drug used orally: Ivermectin"}{"text": "### Human: Most common complication after ERCP is ### Assistant: Complication perforation(1.3%)\/hemorrhage (1.4%),pancreatitis (4.3%) and sepsis (3-30%). Bailey & Love 26th, 208"}{"text": "### Human: Which of the following clinical situations is associated with an increase in predominantly conjugated (\"direct\") bilirubin? ### Assistant: Of the conditions listed, only pancreatic head tumors are associated with an increase in conjugated (\"direct\") bilirubin (obstructive jaundice). Increased levels of unconjugated (\"direct\") bilirubin result from hemolysis over liver defects that impair uptake or conjugation mechanism in liver cells (Gilbe's syndrome, Crigler- Najjar syndrome) positive. Unconjugated bilirubin may cross the immature blood brain Barrier of the newborn and cause Kernicterus. The physiologic jaundice of the newborn observed during the first week of bih is usually mild and due to relatively immature liver conjugation. Ref: Cothren C., Biffl W.L., Moore E.E. (2010). Chapter 7. Trauma. In F.C. Brunicardi, D.K. Andersen, T.R. Billiar, D.L. Dunn, J.G. Hunter, J.B. Matthews, R.E. Pollock (Eds), Schwaz's Principles of Surgery, 9e."}{"text": "### Human: Regarding carcinoma Gallbladder true statement ### Assistant: Clinical features Most commonly presents with right upper quadrant pain often mimicking cholecystitis and cholelithiasis Weight loss, jaundice and abdominal pain are less common presenting symptoms Chronic cholecystitis with a recent change in quality or frequency of the painful episodes in 40% patients Malignant biliary obstruction with jaundice, weight loss and RUQ pain Ref: Sabiston 20th edition Pgno : 1512-1514"}{"text": "### Human: Which anaesthetic agent increases intracranial tension among the following ### Assistant: All inhalational anaesthetics directly act on cerebral blood vessels\u2192vasodilatation\u2192increase cerebral blood flow\u2192 increase intracranial tension"}{"text": "### Human: Granules of Eleidin are present in which of the following layer? ### Assistant: Stratum Lucidum - Also called clear cell layer. Present only in skin of palms and soles. Translucent due to presence of refractile Eleidin granules"}{"text": "### Human: Antidepressant drug used in nocturnal enuresis is: ### Assistant: Sleep related enuresis- Treatment Bed alarms (behavioral therapy)- TOC Desmopressin (DOC) Imipramine (TCA Antidepressant)"}{"text": "### Human: Five days after an uneventful cholecystectomy, an asymptomatic middle-aged woman is found to have a serum sodium level of 125 mEq\/L. Which of the following is the most appropriate management strategy for this patient? ### Assistant: The initial, and often definitive, management of hyponatremia is free-water restriction. Symptomatic hyponatremia, which occurs at serum sodium levels less than or equal to 120 mEq\/L, can result in headache, seizures, coma, and signs of increased intracranial pressure and may require infusion of hypeonic saline. Rapid correction should be avoided so as not to cause central pontine myelinolysis, manifested by neurologic symptoms ranging from seizures to brain damage and death. Additionally, a search for the underlying etiology of the hyponatremia should be undeaken. Acute severe hyponatremia sometimes occurs following elective surgical procedures due to a combination of appropriate stimulation of antidiuretic hormone and injudicious administration of excess free water in the first few postoperative days. Other potential etiologies include hyperosmolarity with free-water shifts from the intra- to the extracellular compament (eg, hyperglycemia), sodium depletion (eg, gastrointestinal or renal losses, insufficient intake), dilution (eg, drug-induced), and the syndrome of inappropriate secretion of antidiuretic hormone (SIADH)."}{"text": "### Human: Reversible cause of dementia is - ### Assistant: Ans. is 'c' i.e., Hypothyroidism Causes of dementiaReversible causesSurgically treatableMedically treatableo Normal pressure hydrocephaluso Brain tumors (frontal lobe tumor)o Meningiomao Subdural hematoma (Head injury)o Hydrocephaluso Hypothyroidismo Depressiono HIV infectiono Alcohol abuseo Vitamin B12, Folate, Niacin deficiencyo Any metabolic or endocrine disturbanceo Neurosyphiliso Hashimoto's encephalopathy o Wilson's diseaseo Celiac disease or Whipple's diseaseo Chronic meningoencephalitiso Drugs and toxin (toxic dementia)Irreversible causeso Alzheimer's diseaseo Huntington's choreao Lewy body dementiao Vascular (Multi-infarct) dementiao Parkinson's diseaseo Creutzfeld Jakob diseaseo Pick's diseaseo Overall Alzheimer's disease is the most common cause. Vascular dementia is the 2nd most common cause."}{"text": "### Human: A obese female having hirsutism on laboratory investigation has high level of LH and androgens. Likely cause is- ### Assistant: Ans. is 'a' i.e., PCOS o The clinical and laboratory features of the patient described in the question match with those of PCOS as depicted in the table. So the most appropriate answer is PCOS.Clinical features of polycystic ovarian diseaseClinical featureHannonalSequelaeo Young womano | E2. levelo Diabaetes (15%)o Central obesityo | FSH | LH > 10 IU\/mlo CVS disorder# SMI > 30kg\/cm2o | FSK\/LH ratioo Lipidaemia# Waist line > 35o | Androgenso Hypertensiono Gligomenorrhoea, amenorrhoeao Testosterone,epiandrostenedione. |dehydropepiandrosterone |o Endometrial cancero Infertility (20%)o Breast cancero Hirsutismo Premature ovariano Acanthosis nigra due to insulin resistance, Thick pigmented skin over the nape of neck, inner thigh and axillao 17-alpha-hvdroxy progesterone> 800 ng.'dLfailure following surgeryo Testosterone > 2 ng\/mlo Most androgens from ovaryo | fasting insulin > l0mlu\/Lo Protactin |o Sex hormone binding globulin (SHBG)o | E2\/oestrone (E1) ratioo F glucose,'insulin ratio <4-5 (normal 2-4-4-5)"}{"text": "### Human: All are used in the management of head injury patient except? ### Assistant: Glucocoicoids are useful for management of vasogenic edema from tumor or brain abscess. In contrast glucocoicoids are avoided in case of head trauma, ischemic and hemorrhagic stroke. Norepinephrine or pressor therapy is used to maintain a MAP to maintain CPP> 60 mm Hg. Sedation is done with propofol or midazolam. Neuromuscular paralysis will be necessary and the patient will need ventilator suppo. This will ensure securing the airway and keeping pCO2 between 30-35mmHg."}{"text": "### Human: Not a premalignant condition: March 2005 ### Assistant: Ans. A: Retinitis pigmentosaGIT premalignant conditionsOf the four major primary small-bowel tumors (adenocarcinomas, lymphomas, carcinoid, and leiomyosarcomas), adenocarcinomas and lymphomas are associated with diseases that seem to increase the risk of developing these malignancies.Immunoproliferative small intestinal disease and celiac disease, are thought to predispose patients to the development of primary lymphoma.Increased risk is also associated with conditions, such as immunodeficiency syndromes, nodular lymphoid hyperplasia, Crohn's disease, the gastrointestinal polyposis syndromes, hereditary nonpolyposis colon cancer, neurofibromatosis, long-standing ileostomy, and urinary diversion procedures.Patient with long standing ulcerative colitis are at risk of developing colonic epithelial dysplaia and carcinoma.Oral cavity premalignant conditionsMany oral SCCs develop from premalignant conditions of the oral cavity.A wide array of conditions have been implicated in the development of oral cancer, including leukoplakia, erythroplakia, palatal lesion of reverse cigar smoking, oral lichen planus, oral submucous fibrosis, discoid lupus erythematosus, and hereditary disorders such as dyskeratosis congenital and epidermolysis bullosaOther pre-malignant conditions include actinic keratosis, Barrett's esophagus and cervical dysplasia."}{"text": "### Human: Which of the following is false about Takotsubo Cardiomyopathy ### Assistant: Echo comes back to normal in two weeks."}{"text": "### Human: Hereditary non-polyposis colorectal cancer (HNPCC) has an increased risk of all the following except ### Assistant: Hereditary non-polyposis colorectal cancer (Lynch syndrome)Hereditary non-polyposis colorectal cancer (HNPCC) is characterised by an increased risk of colorectal cancer and also cancers of the endometrium, ovary, stomach and small intestine.It is an autosomal dominant condition caused by a mutation in one of the DNA mismatch repair genes. The most commonly affected genes are MLH1 and MSH2. The lifetime risk of developing colorectal cancer is 80%, and the mean age of diagnosis is 45 years. Most cancers develop in the proximal colon. Females have a 30-50% lifetime risk of developing endometrial cancerRef: Bailey and Love, 27e, page: 1260"}{"text": "### Human: Iodine RDA is - ### Assistant: Ans. is 'c' i.e., 150 microgram o The RDA of iodine for adults is 150 microgram.GroupRecommended daily intakePreschool children (0-59 months)School children (6 - 12 years)Adults (>12 years)Pregnancy and lactation90 meg120 meg150 meg250 meg"}{"text": "### Human: Spanish windlass was practices in Spain as a method of execution. It is a type of: ### Assistant: Garrotting Garrotting: One of the methods of hemicidal strangulation The victim is attacked from behind without warning and strangled by grasping throat or by throwing a ligature over the neck and tightening it quickly. In this way a single assailant can kill a healthy robust adult. Garrotting was practices as a mode of execution in Spain. Pougal and Turkey. In Spain it was known as `spanish windlass' in which an iron collar around the neck was tightened by a screw for strangling Other common methods of homicidal strangulation are: Throttling - by hand Bansdola - In this type of strangulation the neck is compressed between two sticks of bamboos one in front and other behind the neck. Both the ends are tied with a rope squeezing the victim to the death. Sometimes a single stick is placed across the .front of the neck with a foot on each end of the stick. Mugging - Strangulation is caused by holding the neck of the victim in the bend of the elbow."}{"text": "### Human: Dispropoionately increased blood urea levels compared to serum creatinine (urea creatinine ratio 20:1) can be seen in the following except - ### Assistant: Intrinsic or intrarenal acute renal failure (ARF) occurs when direct damage to the kidneys causes a sudden loss in kidney function. The most common causes of intrinsic acute renal failure are acute tubular necrosis (ATN), acute glomerulonephritis (AGN), and acute interstitial nephritis (AIN) . Ref Harrison20th edition pg 236"}{"text": "### Human: For RNA, which blotting technique is used - ### Assistant: Ans. is 'b' i.e., Northern blot TechniqueSample analyzedGel usedProbeSouthern blotDNAQYesRadioactive DNAAllele specific oligonucleotide (ASO)DNANoAllele specific oligonucleotideMicroarravm-RNA or c-DNANoDNA probeNorthern blotRNAQYesDNA probeWestern (immuno) blotProtein QYesLabeled antibody QSouth Western blotProtein DNANoDNA probeELISAProtein or antibodiesNoAntibody Q (Specific for protein to be measured)ProteomicsProtein QYes-"}{"text": "### Human: Which bacteria acts by inhibiting protein synthesis? ### Assistant: Ans. is 'a' i.e., Pseudomonas"}{"text": "### Human: Panthothenic acid is coenzyme of which of the following reaction (s): ### Assistant: D i.e. Acetylation Pantothenic acid is necessary for synthesis of coenzyme A (CoA)Q, which functions as a coenzyme for acetylation reactionsQ"}{"text": "### Human: Oesophageal motility disorder are best diagnosed by? ### Assistant: Answer is B (Endoscopy): Dysphagia for solids alone suggests a probable mechanical cause for dysphagia such as carcinoma, stricture or esophageal web. The investigation of choice for such cases is Endoscopy."}{"text": "### Human: Quinine given to a patient of falciparum malaria caused sweating and palpitation, the likely cause is? ### Assistant: Ans. is 'c' i.e., Hypoglycemia o Sweating and palpitations suggest hypoglycemia; which can occur due to quinine. Cinchonism is characterized by ringing in ears, nausea, vomiting, veigo, headache, mental confusion, difficulty in hearing and vision. Hypotension and cardiac arrhythmias develop on rapid i.v. injection."}{"text": "### Human: Good pasture's syndrome is characterized by all of the following, except: ### Assistant: Answer is B (Leucocytoclastic Vasculitis) Leucocytoclastic vasculitis is not a typical feature of Good pasture's syndrome Triad of Goodpasture's syndrome (Ferri 's Color Atlas & Text of Clinical Medicine) Glomerulonephritisdeg (RPGN; crescents) Pulmonary HaemorrhageQ (Diffuse Alveolar Haemorrhage) Antibodies to basement membrane antigensQ (Anti-GBM antibodies)"}{"text": "### Human: Main source of energy derived from ### Assistant: Glycogen is the reserve carbohydrate in animals. It is stored in liver and muscles.The phosphorylated enzyme is less sensitive to allosteric inhibitors. Thus even if cellular ATP and glucose-6-phosphate are high, Phosphorylase will be active. The glucose-1-phosphate produced from glycogen in the liver may be conveed to free glucose for release to the blood."}{"text": "### Human: Reverse Coarctation is seen in ### Assistant: Reverse Coarctation ;weak or absent pulse in upper limb + pulse present in lower limb."}{"text": "### Human: Measures involved in sentinel surveillance includes all of the following except. ### Assistant: Ans. (d) Identifying cases free of disability* Surveillance: Is the ongoing systematic collection and analysis of data and the provision of information which leads to action being taken to prevent and control a disease, usually one of an infectious nature* Surveillance is of many types:- Passive Surveillance: Data is itself reported to the health system; For e.g., A patient with fever coming on his own to the PHC, CHC, Dispensary, Private Practitioner, Hospital- Active Surveillance: Health system seeks out 'actively' the collection of data, i.e., goes out to community to collect data; For e.g., Stool sample collection from home in Polio Program.- Sentinel Surveillance: Monitoring of rate of occurrence of specific conditions to assess the stability or change in health levels of a population, It is also the study of disease rates in a specific cohort, geographic area, population subgroup, etc. to estimate trends in larger population; For e.g., Use of health practitioners to monitor trends of a health event in a population- 1. Sentinel Surveillance helps in 'identifying missing cases' and 'supplementing notified cases'ALSO REMEMBER* Most of the national health programs in India rely on Passive Surveillance for morbidity and mortality data collection.* Active Surveillance: Is seen in NVBDCP (Health worker goes house to house every fortnight to detect fever cases, collect blood slides and provide presumptive treatment under malaria component) and National Leprosy Elimination Program (Modified Leprosy Elimination Campaigns)* Sentinel Surveillance is done in National AIDS Control Program wherein STD Clinics, ANC Clinics have been identified as sentinel sites to monitor trends of HIV\/AIDS in the country"}{"text": "### Human: Thymic hypoplsia is seen in which of the following ### Assistant: Ref Robbins 9\/e p224 Thymic Hypoplasia: DiGeorge Syndrome DiGeorge syndrome results from a congenital defect in thymic development with deficient T cell maturation. T cells are absent in the lymph nodes, spleen, and peripheral blood, and infants with this defect are extremely vulnera- ble to viral, fungal, and protozoal infections. Patients are also susceptible to infection with intracellular bacteria, because of defective T cell-mediated immunity. B cells and serum immunoglobulins are generally unaffected. The disorder is a consequence of a developmental mal- formation affecting the third and fouh pharyngeal pouches, structures that give rise to the thymus, parathy- roid glands, and poions of the face and aoic arch. Thus, in addition to the thymic and T cell defects, there may be parathyroid gland hypoplasia, resulting in hypocalce- mic tetany, as well as additional midline developmental abnormalities. In 90% of cases of DiGeorge syndrome there is a deletion affecting chromosomal region 22q11, as discussed in Chapter 6. Transplantation of thymic tissue has successfully treated some affected infants. In patients with paial defects, immunity may improve spontane- ously with age."}{"text": "### Human: Rett's syndrome occurs due to deficiency of ? ### Assistant: Ans. is 'b' i.e., Biotin"}{"text": "### Human: The Triat in which the parents are clinically normal and only siblings are affected but males and females are affected in equal propoions is: ### Assistant: AR"}{"text": "### Human: True about primary peritonitis ### Assistant: (D) Occurs with Cirrhosis of liver # # PRIMARY (SPONTANEOUS) BACTERIAL PERITONITIS> Peritonitis is either primary (without an apparent source of contamination) or secondary.> The types of organisms found and the clinical presentations of these two processes are different. In adults, primary bacterial peritonitis (PBP) occurs most commonly in conjunction with cirrhosis of the liver (frequently the result of alcoholism).> However, the disease has been reported in adults with metastatic malignant disease, postnecrotic cirrhosis, chronic active hepatitis, acute viral hepatitis, congestive heart failure, systemic lupus erythematosus, and lymphedema as well as in patients with no underlying disease. PBP virtually always develops in patients with ascites.> While enteric gram-negative bacilli such as Escherichia coli are most commonly encountered, gram-positive organisms such as streptococci, enterococci, or even pneumococci are sometimes found.> In PBP, a single organism is typically isolated; anaerobes are found less frequently in PBP than in secondary peritonitis, in which a mixed flora including anaerobes is the rule. In fact, if PBP is suspected and multiple organisms including anaerobes are recovered from the peritoneal fluid, the diagnosis must be reconsidered and a source of secondary peritonitis sought."}{"text": "### Human: Osgood Schlatter disease is associated with osteochondritis of ### Assistant: Types of Osteochondritis and Nomenclature Traction apophysitis (chronic strain injuries) Tibial Tuberosity Osgood-Schlatter Calcaneum Sever The lower pole of patella Sinding-Larsen&;s"}{"text": "### Human: Datura seeds resemble - ### Assistant: \u21b5The seeds of datura closely resemble those of the capsicum universally used over India as a condiment, and the difficulty of their detection is enhanced by the fact that, apart from the bitterness, the poison-seeds have little taste and as impure salt is generally used in India."}{"text": "### Human: Measles is infective for: ### Assistant: Measles\/English Measles is spread through respiration (contact with fluids from an infected person&;s nose and mouth, either directly or through aerosol transmission), and is highly contagious -- 90% of people without immunity sharing a house with an infected person will catch it. The infection has an average incubation period of 14 days (range 6-19 days) and Period of communicability in measles is approximately 4 days before and 5 days after the appearance of the rash. Measles is an infection of the respiratory system caused by a virus, specifically a paramyxovirus. REF : ananthanarayana 10th ed"}{"text": "### Human: True about Caspases is? ### Assistant: ANSWER: (A) Involved in apoptosisREF: Robbins pathology 7th edition page 27-29Caspases are essential in cells for apoptosis, or programmed cell death, in development and most other stages of adult life, and have been termed \"executioner\" proteins for their roles in the cell Some Caspases are also required in the immune system for the maturation of lymphocytes"}{"text": "### Human: Metachromatic granules are stained by? ### Assistant: Ans. is 'a' i.e., Ponder's stain"}{"text": "### Human: Finger print bureau was first established in the following country: ### Assistant: Dactylography or fingerprint system or dermatoglyphics or Galton system was first used in India in 1858, by Sir William Herschel in Bengal. Sir Francis Galton systematised this in 1892. Finger print bureau was first established in Kolkata.Fingerprints are impressions of patterns formed by the papillary or epidermal ridges of the fingeips. Ref: The essentials of forensic medicine and toxicology by Dr K S Narayan Reddy, 27th edition, Page 76."}{"text": "### Human: Which of the following drug acts against Pseudomonas - ### Assistant: Ans. is 'a' i.e., Piperacillin Drugs acting against Pseudomonas. o Penicillins- Piperacillin, Carbenicillin, Ticarcillin, Mezlocillin. o Cephalosporins - Ceftazidime, Cefoperazone, Cefepime. o Carbapenems- Imipenem, meropenem o Monobactams - Aztreonam. o Aminoglycosides - Tobramycin, Gentamycin, Amikacin. o Fluroquinolones - Ciprofloxacin, Levofloxacin, Norfloxacin.. o Other - Polymixin B, Colistin."}{"text": "### Human: Cicatrial Alopecia is seen in: ### Assistant: C. i.e. DLE"}{"text": "### Human: According to Spetzler-Main criteria, how much score is given for a 5 cm nidus with AV malformation? ### Assistant: Spetzler-Main AVM grading scale. Graded Feature Points Assigned Size of AVM < 3 cm 3-6 cm >6 cm 1 2 3 Eloquence1 of adjacent brain Noneloquent Eloquent 0 1 Venous drainage Superficial Deep 0 1 'Eloquent areas include: visual, language, and sensorimotor coex; the thalamus and hypothalamus; the internal capsule; the brainstem; the cerebellar peduncles; and the deep cerebellar nuclei."}{"text": "### Human: Which of these following statements is wrong about Tacrolimus? ### Assistant: The side effects of Tacrolimus includes, nephro and hepato toxicity. It can also cause hypeension, tremors, seizure, diabetes mellitus and blurred vision. It should be better avoided with other hepato and renal toxicity drugs. Ref: Manzoor M. Khan (2008), Chapter 4 \"Immunesupressive Agents\", In the book, \"Immunopharmacology\", Springer Publications, USA, Page 91 ; Katzung, 9th Edition, Pages 941, 942"}{"text": "### Human: Macrosomia is a ### Assistant: Macrosomia:- A condition where a baby is large before bihFactors causing macrosomia:Mother having diabetesMother having gestational diabetesObese mother(Refer: Nelson's Textbook of Pediatrics, SAE, 1st edition, pg no. 777, 897 - 899)"}{"text": "### Human: Which one of the following device conves radioactive emissions to light for detection? ### Assistant: Scintillation counter: It measures ionizing radiation. The sensor, called a scintillator, consists of a transparent crystal, usually phosphor, plastic, or organic liquid that fluoresces when struck by ionizing radiation. A sensitive photomultiplier tube (PMT) measures the light from the crystal. The PMT is attached to an electronic amplifier and other electronic equipment to count and possibly quantify the amplitude of the signals produced by the photomultiplier. Geiger counter: Is a type of paicle detector that measures ionizing radiation. They detect the emission of nuclear radiation: alpha paicles, beta paicles or gamma rays. It detects radiation by ionization produced in a low pressure gas in a Geiger Muller tube."}{"text": "### Human: Enterobius commonly inhabit the: September 2011 ### Assistant: Ans. D: Caecum Adult Enterobius vermicularis worms inhabit the caecum, appendix and adjacent poions of the ascending colon, lying closely applied to the mucosal surface. Pinworm (genus En terobius)\/Threadworm\/Seatworm, It is a nematode (roundworm) and a common human intestinal parasite, especially in children. It inhabits the caecum, appendix and adjacent poion of the ascending colon, lying closely applied to the mucosal surface It causes enterobiasis, or less precisely as oxyuriasis in reference to the family Oxyuridae. The pinworm appears as a white, small and delicate nematode. The adult female has a sharply pointed posterior end, is 8 to 13 millimeters long, and 0.5 millimeter thick. The adult male is considerably smaller, measuring 2 to 5 millimeters long and 0.2 millimeter thick, and has a curved posterior end. The eggs are translucent and have a surface that adheres to environmental objects. The eggs measure 50 to 60 micrometers by 20 to 30 micrometers, and have a thick shell that is flattened on one side. The small size and colorlessness of the eggs make them invisible to the naked eye, except in barely visible clumps of thousands of eggs. Eggs may contain a developing embryo or a fully developed pinworm larva.Inside the host, the larvae grow to 140-150 micrometers in length Pinworms spread through human-to-human transmission, by ingesting (i.e., swallowing) infectious pinworm eggs and\/or by anal sex. The eggs are hardy and can remain ble (i.e., infectious) in a moist environment for up to three weeks.They do not tolerate heat well, but can survive in low temperatures: two-thirds of the eggs are still ble after 18 hours at -8 degrees Celsius (18 degF). After the eggs have been initially deposited near the anus, they are readily transmitted to other surfaces through contamination. The surface of the eggs is sticky when laid, and the eggs are readily transmitted from their initial deposit near the anus to fingernails, hands, night-clothing and bed linen.From here, eggs are fuher transmitted to food, water, furniture, toys, bathroom fixtures and other objects.Household pets often carry the eggs in their fur, while not actually being infected. Dust containing eggs can become airborne and widely dispersed when dislodged from surfaces, for instance when shaking out bed clothes and linen.Consequently the eggs can enter the mouth and nose through inhalation, and be swallowed later. Although pinworms do not strictly multiply inside the body of their human host, some of the pinworm larvae may hatch on the anal mucosa, and migrate up the bowel and back into the gastrointestinal tract of the original host. This process is called retroinfection. Despite the limited, 13 week lifespan of individual pinworms, autoinfection (i.e., infection from the original host to itself), either through the anus-to-mouth route or through retroinfection, causes the pinworms to inhabit the same host indefinitely"}{"text": "### Human: Which of the following results in increase in secretion of ADH? ### Assistant: Ans: C HypovolemiaRef: Guyton 12th\/e p. 9051Hypovolemia (Decreased ECF) stimulates the release of ADH.Vasopressin secretion is increased by: - i) Increased osmolarity (osmotic pressure) of plasma; ii) Decreased ECF volume; iii) Pain, emotion, stress, exercise; iv) Nausea and vomiting; v) Standing; vi) Clofibrate, Carbamazapine; vii) Angiotensin II."}{"text": "### Human: Urine osmolality in Diabetes insipidus is ### Assistant: Diabetes insipidus is condition characterised by excessive thirst excretion of large amounts of severely diluted urine Those with diabetes insipidus continue to uinate large amount of urine in spite of water deprivation.Ref: DM Vasudevan, 7th edition, page no: 373"}{"text": "### Human: 1-a hydroxylation in Vitamin-D metabolism takes place in ### Assistant: Vitamin D is a fat-soluble vitamin molecule found in fish liver oils, and also produced in the skin when subjected to ultraviolet rays from sunlight. The main function of the vitamin is to increase the utilization of calcium and phosphorus in bones and teeth. A mixture of several forms of vitamin D: Vitamin D3 is synthesized in the skin upon exposure to ultraviolet light; in contrast, vitamin D2 is obtained only from the diet. Both vitamins D2 and D3 are metabolized to 25-hydroxyvitamin D in the liver, and then to the active 1,25 dihydroxy form in the kidney. Vitamin D has a major role in the intestinal absorption of calcium, bone calcium balance and renal excretion of calcium.Ref: Ganong&;s review of medical physiology;24th edition; page no-379"}{"text": "### Human: Magaldrate is conveed by gastric acid to ? ### Assistant: Ans. is `c' i.e., Magnesium hydroxide and Aluminium hydroxide Magaldrate: Magaldrateis a common antaciddrug that is used for the treatment of duodenal and gastric ulcers, esophagitis from gastroesophageal reflux. Magaldrate is a hydroxymagnesium aluminate complex that is conveed rapidly in gastric acid to Mg(OH), and Al(OH)3, which are absorbed poorly and thus provide a sustained antacid effect."}{"text": "### Human: Which of the following nerves are affected after spinal anaesthetic block \u2013 ### Assistant: Local anesthetics block generation and conduction of nerve impulse at all part of neuron where they come in contact, without causing structural damage. Thus not only sensory but motor impulses and autonomic control is also interrupted."}{"text": "### Human: Drug of choice for Treponema Pallidum is? ### Assistant: ANSWER: (A) Penicillin GREF: Microbiology, Volume 2; Volume 9, Part 2 edited by E. Edward Bittar, Neville Bittar p; 241See APPENDIX-65 for \"SEXUALLY TRANSMITTED DISEASES\"\"Parenteral penicillin G remains the treatment of choice for syphilis (Treponema pallidu and resistance to penicillin has not been reported\""}{"text": "### Human: If hemoglobin status of a population with mean value is 10.3 gm % with SD 2 gm%, then 5% population will be below what value of Hb? ### Assistant: ."}{"text": "### Human: Inclusion body conjunctivitis true is all except: ### Assistant: Ans. Present only in infants"}{"text": "### Human: EBV causes all EXCEPT: ### Assistant: ANSWER: (C) Verrucous lymphomaREF: Jawett's 24th edition Section IV. Virology > Chapter 33. Herpesviruses Cancer by EBV:EBV is recognized as a cause of Burkitt's lymphoma, nasopharyngeal carcinoma, Hodgkins disease, and some other lymphomas. Sera from patients with Burkitt's lymphoma or nasopharyngeal carcinoma contain elevated levels of antibody to virus-specific antigens, and the tumor tissues contain EBV DNA and express a limited number of viral genes. EBV-associated B cell lymphomas are a complication for immunodeficient patients"}{"text": "### Human: All of the following are required more during lactation as compared to pregnancy, except ? ### Assistant: Ans. is 'a' i.e., Iron Required more during lactation as compared to pregnancy : Energy, Vitamin A, thiamin, riboflavin, Vitamin C, niacin, and Vitamin B12. Required more during pregnancy : Iron, protein, and folate. Same requirment in pregnancy and lactation : Fat, calcium, zinc, magnesium, and vitamin B6."}{"text": "### Human: Not a B cell marker - ### Assistant: . CD 134"}{"text": "### Human: In sebaceous glands, accumulation of sebum leads to: ### Assistant: Ans. b. Acne4 steps in pathogenesis of Acne are:Sebum productionHyperkeratosis of pilosebaceous unitPropionibacterium acnes colonization of pilosebaceous unit Inflammation"}{"text": "### Human: A young girl presents with abdominal pain and a recent change in bowel habit, with passage of mucus in stool. There is no associated blood in stool and symptoms are increased with stress. The most likely diagnosis is- ### Assistant: Childhood Chronic ill health in childhood or adolescent IBD may result in growth failure, metabolic bone disease and delayed pubey. Loss of schooling and social contact, as well as frequent hospitalisation, can have impoant psychosocial consequences.Treatment is similar to that described for adults and may require glucocoicoids, immunosuppressive drugs, biological agents and surgery. Monitoring of height, weight and sexual development is crucial. Children with IBD should be managed by specialised paediatric gastroenterologists and transitioned to adult care in dedicated clinics . Pregnancy A women's ability to become pregnant is adversely affected by active IBD. Pre-conceptual counselling should focus on optimising disease control. During pregnancy, the rule of thirds applies: roughly one-third of women improve, one-third get worse and one-third remain stable with active disease. In the post-paum period, these changes sometimes reverse spontaneously. Drug therapy, including aminosalicylates, glucocoicoids Delayed growth and pubeal development: chronic active inflammation, malabsorption, malnutrition and long-term glucocoicoids contribute to sho stature and delayed development, with physical and psychological consequences. * Metabolic bone disease: more common with chronic disease beginning in childhood, resulting from chronic inflammation, dietary deficiency and malabsorption of calcium and vitamin D. * Drug side-effects and adherence issues: young people are more likely to require azathioprine or biological therapy than adults. Poor adherence to therapy is more common than with adults, as younger patients may feel well, lack self-motivation to adhere and believe that drugs are ineffective or cause side-effects. * Loss of time from education: physical illness, surgery, fatigue in chronic inflammatory bowel disease, privacy and dignity issues, and social isolation may all contribute. * Emotional difficulties: may result from challenges in coping with illness, problems with forming interpersonal relationships, and issues relating to body image or sexual function. Ref Davidson edition23rd pg823"}{"text": "### Human: Length of umbilical cord is? ### Assistant: ANSWER: (C) 30-100 cmsREF: Dutta 6th ed p. 40The normal length of umblical cord is around 50cms with usual variation between 30-100 cms"}{"text": "### Human: Vitamin B12 intrinsic factor absorption occurs in ### Assistant: Most of the vitamins are absorbed in the upper small intestine, but vitamin B12 is absorbed in the ileum.Vitamin B12 binds to intrinsic factor and the complex is absorbed across the ileal mucosa.Vitamin B12 and folate absorption are Na+ independent.Other vitamins are absorbed by carriers that are Na+ cotranspoers.(Ref: Ganong&;s Review of medical physiology, 23 rd edition, page 458)"}{"text": "### Human: Splenunculi are commonly seen in ### Assistant: A. (Hilum of spleen) (1103-B & L 25th)ACCESSORY SPLEEN (Splenunculi) - probably results from a failure of infusion of splenic embryonic tissues* They are located near the hilum of the spleen in 50% of cases and related to the splenic vessels or behind the tail of pancreas in 30%. The remainder are located in the mesocolon or the splenic ligaments* These are functionally similar to the spleen and while performing splenectomy for blood dyscrasias removal of these accessory spleens are obligatory, otherwise there always remains a chance of recurrence* Axis of spleen enlargement projects into- Greater sac**"}{"text": "### Human: Causative agent of Favus is: ### Assistant: Ans. A. Tinea schoenleiniiFavous usually affect scalp, but occurring occasionally on any part of the skin and even at times on mucous membrane, caused by Tinea schoenleinii."}{"text": "### Human: Zona Pellucida disappears by:- ### Assistant: Zona pellucida is thick membranous covering made up of glycoprotein, the microvilli of oocyte and follicular cells project into zona pellucida for nutrition. it prevent polyspermy and implantation. Blastocyst forms at 4th day and by the end of 5th day, zona pellucida disappears and implantation takes place at 6th -7th day after feilization."}{"text": "### Human: Storage form of thyroid hormone- ### Assistant: The synthesis and storage of thyroid hormones occurs between the follicular cells and the colloid. The storage form of thyroid hormone is thyroglobulin. Thyroglobulin is a large glycoprotein synthesized in the follicular cells and has a molecular weight of around 650 000 with about 140 tyrosine residues, depending on the form of thyroglobulin. Approximately one quaer of these residues is iodinated at the apical-colloid interface. Once iodinated, thyroglobulin is taken up into the colloid of the follicle where, still incorporated in the protein, a coupling reaction between pairs of iodinated tyrosine molecules occurs. The coupling of two tyrosine residues each iodinated at two positions (di-iodotyrosine, DIT) produces tetra-iodothyronine or thyroxine (T4 ) whilst the combination of DIT with mono-iodotyrosine (MIT) produces tri-iodothyronine (T3 ). Such coupling can occur within a single molecule of thyroglobulin or between dimerized molecules of the protein. This coupling is catalyzed by TPO. Thyroid hormones are stored in this state and are only released when the thyroglobulin molecule is taken back up into the follicular cells. Stimulated by TSH, thyroglobulin droplets are captured by the follicular cells by a process of pinocytosis. Fusion of the droplets with lysosomes results in hydrolysis of the thyroglobulin molecules and release of T3 and T4. Reference : page 2285 Harrison's Principles of Internal Medicine 19th edition"}{"text": "### Human: Congenital hepatic fibrosis is a characteristic feature of ### Assistant: Infantile and Iuvenile forms are associated with congenital Hepatic fibrosis."}{"text": "### Human: MHC restriction is a pa of all except aEUR' ### Assistant: Autoimmune disorder Major histocompatibility complex : Major histocompatibility complex, also called the human leucocyte antigen (HLA) complex is a 4 megahase region on chromosome 6. This region is densely packed with genes. These genes encodes two major proteins or antigens i.e., MHC I & MHC II. These proteins play an impoant role in the differentiation of foreign cells from host cell. Presence of these proteins is necessary for identification of host cells. Identification of the cells is impoant for immune responses. If the cell is recognised as self, the immune system will not mount an immune response against it. How does MHC helps in this ??? Cells continuously display small pieces of their own internal protein, carrying them, outsie the cell membrane where the immune system can see them. These peptides are just pieces of the normal proteins found inside the cell. These peptides are held together at the cell surface by major histocompatibility complex, which holds these peptides together for the immune system to examine them. Each persons immune system is customized early in life to ignore these peptides, so healthy cells are not able to trigger the immune system and they are left alone. But if any virus or other microorganism is multiplying inside the cell, the peptides synthesized by it also is displayed on the surface. The MHC displays these unusual (foreign) protein on the surface of the cell. The immune system is triggered because of these peptides, staing, a series of event that will eventually lead to killing of the cell containing foreign peptide. MHC is also the cause of tissue rejection during skin grafts and organ grafts. This is how the MHC protein got its name; the term histocompatibility refers to the difficulty of finding compatible grafts between a donor and a patient. Each person has their own collection of MHC molecules. If you graft a piece of skin that has a different collection of MHC types they will trigger the immune system to destroy the cell. So a compatible donor is needed such as a relative who has a similar collection of MHC molecules. MHC is also active against cancer Cancer cells, like normal cells, display pieces of their own protein on their surface. So if any of these proteins carries recognizable cancer mutations, this provides a signal to the immune system that something is wrong and the cell is killed. MHC plays role in activation of T helper cells (Th) MHC class II molecules are present on the membranes of dendritic cells, macrophages. These cell function as antigen presenting cell during immune response. They capture the antigen and present it to T helper cells. In response, the T helper cells secrete cytokines that stimulate B cell proliferation and antibody production. The impoant point about T helper cells is that they respond to only those antigens which have been presented to them by cells that have host matching MHC complex on the presenting cell. In general the antigen presenting cell .first phagocytoses the protein antigen and cleaves it to peptides in the lysosomes. These peptides are then extruded and attached to the presenting cell's surface MHC complex. T helper cells first, make sure, that these MHC class II molecules are host's own MHC and then responds appropriately. So, we can say that receptor on the T helper cell is thus responsive to the combination of antigen and host matching MHC complex on the presenting cell. Helper \/ inducer T cells (CD4)---) MHC class II restriction - Suppressor T cells (CD8) MHC class I restriction Cytolytic cytotoxic T cells (CD8) MHC class I restriction (Kills viral infected cells, tumor cells, facilitates graft rejection)"}{"text": "### Human: During cardiac imaging the phase of minimum motion of hea is: ### Assistant: D i.e. Mid diastole- During cardiac imaging (such as MRI, CT, electron beam tomography), mid diastolic phase (or diastasis) of cardiac cycle is usually (but not always) associated with lowest (minimum) mean motion of hea.Q- Optimum phase (i.e. with minimum cardiac motion) for cardiac and coronary vessel imaging is mid-diastole (diastasis) at low or intermediate hea rates; but is variable and may occur in late systole at high hea rates.Cardiac Motion and Imaging- Radiological imaging by CT and MRI can detect coronary aery disease in early stage. However, the use of these tools to cardiac imaging is severely limited because of motion aifacts produced by constantly in motion hea completing an entire cycle of contraction and relaxation in about 1 second.- So radiological cardiac imaging requires a 'window' of minimum cardiac motion to reduce motion aifacts and achieve meaningful images. The phase of minimum cardiac motion is although variable and depends on several factors especially hea rate and the chamber (i.e. right \/left - atrium or ventricle) to be specifically imaged. However, the phase of minimum cardiac motion (= optimum phase for cardiac & coronary vessel imaging)."}{"text": "### Human: Mandibular nerve does not supply:? ### Assistant: A. i.e. Buccinator"}{"text": "### Human: A 68-year-old man is admitted to the coronary care unit with an acute myocardial infarction. His postinfarction course is marked by congestive hea failure and intermittent hypotension. On the fouh day in hospital, he develops severe midabdominal pain. On physical examination, blood pressure is 90\/60 mm Hg and pulse is 110 beats per minute and regular; the abdomen is soft with mild generalized tenderness and distention. Bowel sounds are hypoactive; stool Hematest is positive. Which of the following is the most appropriate next step in this patient's management? ### Assistant: In the absence of peritoneal signs, angiography is the diagnostic test of choice for acute mesenteric ischemia. Patients with peritoneal signs should undergo emergent laparotomy. Acute mesenteric ischemia may be difficult to diagnose. The condition should be suspected in patients with either systemic manifestations of aeriosclerotic vascular disease or low cardiac-output states associated with a sudden development of abdominal pain that is out of propoion to the physical findings. Because of the risk of progression to small-bowel infarction, acute mesenteric ischemia is an emergency and timely diagnosis is essential. Although patients may have lactic acidosis or leukocytosis, these are late findings. Abdominal films are generally unhelpful and may show a nonspecific ileus pattern. Since the pathology involves the small bowel, a barium enema is not indicated. Upper gastrointestinal series and ultrasonography are also of limited value. CT scanning is a good initial test, but should still be followed by angiography in a patient with clinically suspected acute mesenteric ischemia, even in the absence of findings on the CT scan. In addition to establishing the diagnosis in this stable patient, angiography may also assist with operative planning and elucidation of the etiology of the acute mesenteric ischemia. The cause may be embolic occlusion or thrombosis of the superior mesenteric aery, primary mesenteric venous occlusion, or nonocclusive mesenteric ischemia secondary to low-cardiac output states. A moality of 50% to 75% is repoed. The majority of affected patients are at high operative risk, but early diagnosis followed by revascularization or resectional surgery or both are the only hope for survival. Celiotomy must be performed once the diagnosis of aerial occlusion or bowel infarction has been made. Initial treatment of nonocclusive mesenteric ischemia includes measures to increase cardiac output and blood pressure. Laparotomy should be performed if peritoneal signs develop"}{"text": "### Human: Drugs undergoing acetylation include all except: ### Assistant: Ans. B. MetoclopramideAcetylation by N-acetyl transferases (NAT)e.g, S - SulfonamidesH - HydralazineI - IsoniazidP - Procainamide, PASD - DapsoneC - Clonazepam, Caffeine"}{"text": "### Human: A group of expes discussing a topic in front of the audience without any specific order. This mode of communication is known as: September 2008 ### Assistant: Ans. A: Panel discussion In panel discussion there is a chairman\/moderator and 4-8 speakers. Each speaker prepares separately, the other speakers hear one another for the time at the session itself. Panel discussions, however, differ from team presentations. Their purpose is different. In a team presentation, the group presents agreed-upon views; in a panel discussion, the purpose is to present different views. Also in a team presentations, usually speakers stand as they speak; in panel discussions, usually speakers sit the whole time. Technically, a panel discussion consists of questions and answers only, and a symposium consists of a series of prepared speeches, followed by questions and answers. The chairman\/ moderator must monitor time and manage questions. If each paicipant is making a speech for a set period of time, he should signal the speakers at the one minute to go mark and at the stop mark. If a speakers goes more than one or two minute he can stop them to gave the equal rights to each speakers. The chairman\/ moderator must be a biased person; he is neither in our nor against the topic. At the end the compare should summarize the discussion and thank the panel members."}{"text": "### Human: Starvation and diabetes mellitus can lead on to ketoacidosis which of the following features is in our of ketoacidosis due to diabetes mellitus. ### Assistant: A i.e. Increase in glucagons\/ insulin ratio, increased C- AMP & increased blood sugar.In diabetes little glucose is oxidized as fuel, except by the brain. The rest of tissues burn a large amount of fat, paicularly the liver where the amount of acetyl COA formed from fatty acids exceeds the capacity of the tricarboxylic acid cycle to oxidize it. The excess acetyl COA is conveed to ketone bodies l\/t ketonemia, ketonuria & ketoacidosisDiabetics not only have a defect in the tissue utilization of glucose but also appears to be metabolically poised to produce maximum amount of glucose from amino acids (gluconeogenesis) and to prevent glucose from being utilized to to form fat. (fat synthesis)Insulin is decreased & glucagons is increased 1\/t Dglucagon\/ insulin ratioQ. Glucagon acts elevation in C- AMPQ"}{"text": "### Human: ARDS true? ### Assistant: ARDS is a clinical syndrome of severe dyspnoea of rapid onset, hypoxaemia and diffuse pulmonary infiltrates leading to respiratory failure.ARDS is characterised by Po2\/Fio2(inspiratory O2 fraction)<200 mmHg.There will be no hypercapnia, so the patient will have type 1 respiratory failure. \nThere will be a stiff-lung syndrome with collapse and decreased compliance of the lungs due to loss of surfactant."}{"text": "### Human: Hypeension is seen with all except- ### Assistant: Ans. is d i.e., Levodopa Drug causing hypeension Cocaine MAO inhibitors Oral contraceptives Clonidine withdrawl Tricyclic antidepressants Cyclosporine Glucocoicoids Rofecoxib (NSAID) Erythropoietin Valdecoxib (NSAID) Sympathomimetics"}{"text": "### Human: Beta 2 agonist used in rescue therapy in acute respiratory conditions are all except? ### Assistant: Ans. is 'd' i.e., Ketotifen Formoterol, Salbutomol and bambuterol are p 2 agonists. o Ketotifen is a mast cell stabilizer."}{"text": "### Human: The post mortem report in case of death in police custody should be recorded as ### Assistant: Ans. b (Video recording). (Ref. Parikh, FMT, 5th ed., 130)Installation of video camera in jail and recording of postmortem in case of death in police custody is according to human rights."}{"text": "### Human: Reversible loss of polarity with abnormality in size and shape of cells is known as? ### Assistant: Dysplasia is reversible\nAnaplasia is irreversible"}{"text": "### Human: During autopsy for virology study which agent is used for storing tissue: NEET 14 ### Assistant: Ans. 50% glycerine"}{"text": "### Human: The protective bacterium in the normal vagina is: ### Assistant: Vagina has inhabitant bacteria called as Doderleins bacteria which is a lactobaccilli, and converts the glycogen present in vaginal epithelium into lactic acid.\nThus, pH of the vagina is acidic\nThe pH of the vagina in an adult woman is 4 -5.5 with an average of 4.5.\nThe pH of vagina varies with age \u2014 for further details see preceding text."}{"text": "### Human: Bence jones proteinuria is derived from? ### Assistant: Ans. (b) Light chain globulins(Ref: R 9th\/pg 598-602)Excretion of light chains in the urine has been referred to as Bence Jones proteinuria.Light chains includes k and l (kappa and lambda)"}{"text": "### Human: Pradhan mantra swasthya suraksha yojana was launched in: ### Assistant: Pradhan mantra swasthya suraksha yojana was approved in 2006 with the objective of correcting imbalances in availability of affordable teiary level healthcare in the country. Ref: National Health Programmes in India, J.Kishore, 10th edition pg: 84"}{"text": "### Human: Which of the following measures sensitivity: September 2011 ### Assistant: Ans. B: True positives Sensitivity denotes true positives"}{"text": "### Human: Caput succedaneum in a newborn is: Karnataka 07 ### Assistant: Ans. Collection of sero-sanguineous fluid in the scalp"}{"text": "### Human: Zero order kinetics is seen in all except ### Assistant: Zero order kinetics is seen in high dose of salicylates,ethanol and phenytoin. Ref-KDT 6\/e p31"}{"text": "### Human: In a normal healthy person the arterial oxygen is considered satisfactory if SPO2 is more than ### Assistant: Ans. c (90) (Ref H-17th\/l590-91; pg. A-15)In a normal healthy person the arterial oxygen is considered satisfactory if SP02 is more than 90.Arterial O2 tensionPaO212.7 +- 0.7 kPa (95 +- 5 mm Hg)Arterial CO2 tensionPaCO25.3 +- 0.3 kPa (40 +- 2 mm Hg)Arterial O2 saturationSaO20.97 +- 0.02 (97 +- 2%)Arterial blood pHpH7.40 +- 0.02Arterial bicarbonateHCO3-24 + 2 mEq\/LBase excessBE0 +- 2 mEq\/LDiffusing capacity for carbon monoxide (single breath)DLCO0.42 mL CO\/s\/mm Hg (25 mL CO\/min\/mm Hg)Dead space volumeVD2 mL\/kg body wtPhysio dead space; dead space-tidal vol ratioVD\/VTKnow at rest & exercise(a) Rest < 35% VT(b) Exercise < 20% VTAlveolar-arterial difference for O2P(A -a) O2< 2.7 kPa (20 mm Hg)Educational Point# Alveolar hypoventilation exists by definition when arterial PC02 (PaC02) increases above the normal range of 37-43 mm Hg, but in clinically important hypoventilation syndromes PaC02 is generally in the range of 50-80 mm Hg.# A decrease in the Anion Gap can be due to:- an increase in unmeasured cations;- the addition to the blood of abnormal cations, such as lithium (lithium intoxication) or cationic immunoglobulins (plasma cell dyscrasias);- a reduction in the major plasma anion albumin concentration (nephrotic syndrome); (A fall in serum albumin by 1 g\/dL from the normal value decreases the anion gap by 2.5 mEq\/L).- a decrease in the effective anionic charge on albumin by acidosis; or- hyperviscosity and severe hyperlipidemia."}{"text": "### Human: A 40-year-old man presents with 5 days of productive cough and fever. Pseudomonas aeruginosa is isolated from a pulmonary abscess. CBC shows an acute effect characterized by marked leukocytosis (50,000\/mL) and the differential count reveals shift to left in granulocytes. Which of the following terms best describes these hematologic findings? ### Assistant: Circulating levels of leukocytes and their precursors may occasionally reach very high levels (>50,000 WBC\/mL). These extreme elevations are sometimes called leukemoid reactions because they are similar to the white cell counts observed in leukemia, from which they must be distinguished. The leukocytosis occurs initially because of accelerated release of granulocytes from the bone marrow (caused by cytokines, including TNF and IL-1) There is a rise in the number of both mature and immature neutrophils in the blood, referred to as a shift to the left. In contrast to bacterial infections, viral infections (including infectious mononucleosis) are characterized by lymphocytosis Parasitic infestations and ceain allergic reactions cause eosinophilia, an increase in the number of circulating eosinophils. Leukopenia is defined as an absolute decrease in the circulating WBC count."}{"text": "### Human: A patient present with dysphagia of 4 weeks duration. Now he is able to swallow liquid food only. Which of the following is the one investigation to be done: ### Assistant: Answer is B (Upper GI endoscopy is to be done) The patient is presenting with new onset dysphagia for solids alone (able to swallow liquids). This suggests dysphagia due to mechanical obstruction (carcinoma, stricture, web) which is best diagnosed by endoscopy. Note: Barium swallow is the recommended initial investigation Endoscopy is the investigation of choice."}{"text": "### Human: Inhalation of fungal spores can cause primary lung infections. Of the following organisms, which one is most likely to be associated with this mode of transmission? ### Assistant: C. albicans and Candida tropicalis are opportunistic fungi, and as part of the normal flora are not transmitted by inhalation. C. immitis is a dimorphic fungus and inhalation of the spores transmits the infection. Sprothrix is also a dimorphic fungus but its portal of entry is cutaneous. Trychophyton is a dermatophyte and one of the causes of athlete's foot."}{"text": "### Human: Fleeting skin lesions are often present in patients with: ### Assistant: Fleeting skin lesions are often presents in patients with hepatitis B. Ref: Schiff's diseases of liver, Edition -11, Page - 3."}{"text": "### Human: In which of the following locations, Carcinoid tumor is most common ### Assistant: carcinoid tumour commonly occurs in appendix(45%),ileum(25%),rectum(15%).other sites are(15%)other pas of GIT ,bronchus, and testis SRB,5th,877."}{"text": "### Human: S100 is a marker used in the diagnosis of all Except ### Assistant: Acidic protein, 100% Soluble in ammonium sulfate at neutral pH (derivation of name) S100 protein family is multigenic group of nonubiquitous cytoplasmic EF-hand Ca2+-binding proteins, sharing significant structural similarities at both genomic and protein levels; S100 protein family has 24 known human members each coded by a separate gene; at least 19 on chromosome 1q21 Helps regulate cell contraction, motility, growth, differentiation, cell cycle progression, transcription and secretion Structurally similar to calmodulin"}{"text": "### Human: A Patient who is on antihypertensive drug develops a dry cough. Which of the following drug might be responsible for the condition: ### Assistant: The angiotensin-converting enzyme normally helps in conversion of angiotensin I to angiotensin II, which acts as a vasoconstrictor in case of hypovolemia.\nIn addition to that, ACE also helps in the breakdown of bradykinin.\nWhen ACE inhibitors (captopril) are given, it inhibits ACE and in addition inhibits the breakdown of bradykinin also, which leads to accumulation of bradykinin level in the body.\nThis increased level of bradykinin presents with a Dry cough, rashes, urticaria and angioedema."}{"text": "### Human: Mad Cow disease (Spongiform disease) occurs due to ### Assistant: Mad cow disease or bovine spongiform encephalopathy i.e. Caused by \u201cprions\u201d\n\u201cPRIONS\u201d are unconventional transmissible agents, proteinaceous in nature.\u201d\nDiseases caused by prions are\nKuru\nCreutzfeldt Jakob Disease.\nGerstmann Straussler Scheinker Syndrome\nFatal Familial Insomnia.\nScrapie Of Sheep\nTransmissible Encephalopathy of Mink.\nBovine Spongiform Encephalopathy (Mad Cow Disease)"}{"text": "### Human: A patient on lithium therapy was found to be hypeensive also. Which of the following antihypeensive drugs is contraindicated in a patient on lithium therapy in order to prevent toxicity? ### Assistant: Diuretics (thiazide, furosemide) by causing Na+ loss promote proximal tubular reabsorption of Na+ as well as Li+ --plasma levels of lithium rise. ref : KD Tripati 8th ed."}{"text": "### Human: In marasmus, which of the following is not seen? ### Assistant: Edema is a feature of Kwashiorkor, not seen in marasmus. Kwashiorkor Marasmus Edema Present absent appetite poor Voracious appetite CNS involvement apathy Active child Hepatomegaly seen Not seen Skin and hair changes More common Less common"}{"text": "### Human: A 23-year-old woman presents with a rubbery, freely movable 2-cm mass in the upper outer quadrant of the left breast. A biopsy of this lesion would most likely histologically reveal ### Assistant: The most common benign neoplasm of the breast is fibroadenoma, which typically occurs in the upper outer quadrant of the breast in women between the ages of 20 and 35. These lesions originate from the terminal duct lobular unit and histologically reveal a mixture of fibrous connective tissue and ducts. Clinically, fibroadenomas are rubbery, freely movable, oval nodules that usually measure 2 to 4 cm in diameter. Numerous neutrophils are seen in acute bacterial infection of the breast (acute mastitis), which is usually seen in the postpaum lactating or involuting breast. Dilation of the breast ducts (ectasia) with inspissation of breast secretions is characteristic of mammary duct ectasia, which is common in elderly women. If large numbers of plasma cells are also present, the lesion is called plasma cell mastitis. Fat necrosis of the breast, associated with traumatic injury, is characterized by necrotic fat surrounded by lipid-laden macrophages and a neutrophilic infiltration. Fibroadenoma Reference: Robbins & Cotran Pathologic Basis of Disease, 9edition"}{"text": "### Human: Most common pa of larynx involved in TB ? ### Assistant: Ans. is 'b' i.e., Posterior Essential otolaryngology 2\"d\/e p. 1139] Disease affects the posterior third of larynx more commonly than anterior pa. The pas affected in descending order of frequency are :- i) Interarytenoid fold, ii) Ventricular band, iii) Vocal cords, iv) Epiglottis."}{"text": "### Human: Pyogenic granuloma is associated with ### Assistant: Pyogenic granuloma is a misnomer as there is no granuloma. It is a benign vascular lesion that bleeds easily on trauma.\nIt is associated with Inflammatory Bowel disease (Ulcerative colitis > Crohn's disease)."}{"text": "### Human: Muscle relaxant used in renal failure - ### Assistant: As Atracurium and Cis-atracurium are inactivated by Hoffman's elimination, they are the muscle relaxants of choice for both liver and renal failure."}{"text": "### Human: Ramachandran is on the surgical ward with non-seminomatous tumor of testis and more than 4 retroperitoneal lymph nodes involved. You are the resident who is making a decision about fuher management. The treatment include all of the following, except: ### Assistant: Inguinal orchiectomy with high ligation of the cord at the internal ring is the initial management of testicular tumour. Multi Agent chemotherapy and followed by surgical resection is one modality of treatment. Ref: Bailey & Love, 25th Edition, Page 1269."}{"text": "### Human: A 34-year-old female has a history of intermittent episodes of severe abdominal pain. She has had multiple abdominal surgeries and exploratory procedures with no abnormal findings. Her urine appears dark during an attack and gets even darker if exposed to sunlight. The attacks seem to peak after she takes erythromycin, because of her penicillin allergy. This patient most likely has difficulty in synthesizing which one of the following? ### Assistant: The patient has acute intermittent porphyria, which is a defect in one of the early steps leading to heme synthesis. The buildup of the intermediate that cannot continue along the pathway leads to the dark urine, and it turns darker when ultraviolet (UV) light interacts with the conjugated double bonds in the molecule. Erythromycin is metabolized through an induced P450 system, which requires increased heme synthesis. This leads to metabolite buildup to the level where the abdominal pain appears. The defect in heme synthesis does not affect creatine phosphate, cysteine, thymine, or methionine levels."}{"text": "### Human: Most common type of hypospadias is: September 2011 ### Assistant: Ans. A: Glandular Glandular hypospadias is common and does not usually require treatment Hypospadias: The external meatus opens on the underside of the penis or the perineum, and the inferior aspect of the prepuce is poorly developed (hooded prepuce) Meatal stenosis occurs Bifid scrotum 6 - 10 months of age is the best time for surgery"}{"text": "### Human: In a subclan aery block at outer border of 1st rib, all of the following aeries help in maintaining the circulation to upper limb, EXCEPT? ### Assistant: A rich anastomosis exists around the scapula between branches of subclan aery (first pa) and the axillary aery (third pa). This anastomosis provides a collateral circulation through which blood can flow to the limb when the distal pa of subclan aery or the proximal pa of axillary aery is blocked. Scapular anastomoses occur between the following branches of proximal subclan and distal axillary aery: Branches of Subclan aery: Thyrocervical trunk, Suprascapular and Deep branch of transverse cervical.Branches of Axillary aery: Subscapular, Posterior circumflex humeral and Thoracoacromial aeires."}{"text": "### Human: Lymphatics of suprarenal gland drain into lymph nodes - ### Assistant: Suprarenal glands drain into para-aortic nodes."}{"text": "### Human: All of the following can be involved in an injury to the head of the fibula, EXCEPT: ### Assistant: The common peroneal nerve is extremely vulnerable to injury as it winds around the neck of the fibula. At this site, it is exposed to direct trauma or is involved in fractures of the upper pa of the fibula. While passing behind the head of the fibula, it winds laterally around the neck of the bone, pierces peroneus longus muscle, and divides into two terminal branches: the superficial peroneal nerve and deep peroneal nerve (anterior tibial nerve). Injury to common peroneal nerve causes foot drop.Ref: Clinical Anatomy By Regions By Richard S. Snell, 9th edition, Page 485."}{"text": "### Human: Pain felt between great toe and 2nd toe is due to involvement of which nerve root - ### Assistant: First web space of foot is supplied by deep peroneal nerve (L5)"}{"text": "### Human: Which of the following is non-competitive inhibitor of intestinal alkaline phosphatase? ### Assistant: Ans. d (L-Phenylalanine) (Ref. Biochemistry by Vasudevan, 4th ed., 57; 5th ed., 55-56)ALKALINE PHOSPHATASE# The pH optimum for the enzyme reaction is between 9 and 10.# It is activated by magnesium and manganese.# Zinc is a constituent ion of ALR# It is produced by osteoblasts of bone, and is associated with the calcification process.# It is localised in cell membranes (ecto-enzyme), and is associated with transport mechanisms in liver, kidney and intestinal mucosa.# For estimation, sodium phenyl phosphate is used as the substrate, pH is adjusted to 9.9, serum sample is added as the source of ALP and incubated.# Normal serum value of ALP is 40-125 U\/L.# Raised Levels:Mild increasepregnancy.Moderate (2-3 times) increaseIn infective hepatitis, alcoholic hepatitis or hepatocellular carcinoma, Hodgkin's lymphoma, congestive cardiac failure or ulcerative colitis.Very high levels of ALP (10-12 times)In extrahepatic obstruction (obstructive jaundice) or cholestasis.Drastically high levels of ALP (10-25 times)Bone diseases where osteoblastic activity is enhanced such as Paget's disease, rickets, osteomalacia, osteoblastoma, metastatic carcinoma of bone, and hyperparathyroidism.Iso-enzymes of Alkaline Phosphatase# Alpha-1 ALP moves in alpha-1 position, it is synthesized by epithelial cells of biliary canaliculi. It is about 10% of total activity and is increased in obstructive jaundice and to some extent in metastatic carcinoma of liver.# Alpha-2 heat labile ALP is stable at 56degC; but loses its activity when kept at 65degC for 30 minutes. It is produced by hepatic cells. Therefore, exaggerated alpha-2 band suggests hepatitis. This liver iso-enzyme forms about 25% of total ALP.# Alpha-2 heat stable ALP will not be destroyed at 65degC, but is inhibited by phenylalanine. It is of placental origin, which is found in blood in normal pregnancy. An iso-enzyme closely resembling the placental form is characteristically seen in circulation in about 15% cases of carcinoma of lung, liver and gut and named as Regan iso-enzyme (after the first patient in whom it was detected) or carcinoplacental iso-enzyme. Chronic heavy smoking also increases Regan iso-enzyme level in blood. Normal level is only 1% of the total ALP.# Pre-beta ALP is of bone origin and elevated levels are seen in bone diseases. This is most heat labile (destroyed at 56degC, 10 min). Wheat germ lectin will precipitate bone isoenzyme. This constitutes about 50% of normal ALP activity.# Gamma-ALP is inhibited by phenylalanine and originates from intestinal cells. It is increased in ulcerative colitis. About 10% of plasma ALP are of intestinal variety.# Leucocyte alkaline phosphatase (LAP) is significantly decreased in chronic myeloid leukemia. It is increased in lymphomas and leukemoid reactions.ALANINE AMINO TRANSFERASE (ALT)# It is also called as serum glutamate pyruvate transaminase (SGPT).# The enzyme needs pyridoxal phosphate as co-enzyme.# Normal serum level of AL T for male is 13^4-0 U\/L and for female is 10-28 U\/L.# Very high values (100 to 1000 U\/L) are seen in acute hepatitis, either toxic or viral in origin.# Both ALT and AST levels are increased in liver disease, but ALT > AST.# Moderate increase (25 to 100 U\/L) may be seen in chronic liver diseases such as cirrhosis, Hepatoma.# A sudden fall in ALT level in cases of hepatitis is very bad prognostic sign.NUCLEOTIDE PHOSPHATASE# It is also known as 5' nucleotidase.# It is a marker enzyme for plasma membranes and is seen as an ecto-enzyme (enzyme present on the cell membrane).# Serum samples contain both ALP and NTP.# These are distinguished by Nickel ions which inhibit NTP but not ALP.- Normal NTP level in serum is 2-10 IU\/L. It is moderately increased in hepatitis and highly elevated in biliary obstruction.# Unlike ALP, the level is unrelated with osteoblastic activity and therefore unaffected by bone disease.GAMMA GLUTAMYL TRANSFERASE (GGT)# The old name was gamma glutamyl transpeptidase.# It can transfer gamma glutamyl residues to substrate.# In the body it is used in the synthesis of glutathione.# It is seen in liver, kidney, pancreas, intestinal cells and prostate gland."}{"text": "### Human: Mondors disease is: ### Assistant: Ans: D (Thrombophlebitis of superficial veins of breast) Ref: Bailey dt Love's Short Practice of Surgery, 25th Edition,833-4Explanation:Mondor's diseaseThrombophlebitis of the superficial veins of the breast and anterior chest wallCan also involve armCauseIdiopathicClinical featuresThrombosed subcutaneous vessel , usually attached to the skin - pathognomonic feature.When the skin over the breast is stretched by raising the arm. a narrow, shallow subcutaneous groove alongside the cord becomes apparent.Differential DiagnosesLymphatic permeation from occult carcinomaManagementConservativeRestriction of arm movements is advised.Resolv es within few months without any complicationAssociation with breast carcinoma is Coincidental"}{"text": "### Human: Treatment of Choice in Sleep Apnea Syndrome: ### Assistant: (A) Continuous positive pressure ventilation# Sleep apnea: Sleep apnea is a sleep disorder characterized by pauses in breathing during sleep. Each episode, called an apnea, lasts long enough so that one or more breaths are missed, and such episodes occur repeatedly throughout sleep.> The standard definition of any apneic event includes a minimum 10 second interval between breaths, with either a neurological arousal (a 3-second or greater shift in EEG frequency, measured at C3, C4, O1, or O2), a blood oxygen desaturation of 3-4deg\/: or greater, or both arousal and desaturation. Sleep apnea is diagnosed with an overnight sleep test called a polysomnogram, or a \"Sleep Study\".> Treatment: The most common treatment and arguably the most consistently effective treatment for sleep apnea is the use of a continuous positive airway pressure (CPAP) device, which 'splints' the patient's airway open during sleep by means of a flow of pressurized air into the throat. However the CPAP machine only assists inhaling whereas a BiPAP machine assists with both inhaling and exhaling, and is used in more severe cases. Medications like Acetazolamide lower blood pH and encourage respiration. Low doses of oxygen are also used as a treatment for hypoxia but are discouraged due to side effects."}{"text": "### Human: Rectal polyp most commonly presents as ### Assistant: .LOWER GI BLEED- * Bleeding in the GIT below the level of the ligament of Treitz. * Normal faecal blood loss is 1.2 ml\/day. A loss more than 10 ml\/day is significant.* Angiodysplasia. * Diveicular disease - commonest cause in Western countries. * Tumours of colon or small bowel. * Anorectal diseases--Haemorrhoids, fissure in ano. * Ulcerative colitis * Crohn's disease. * Colorectal polyps; rectal carcinomas. * Intussusception ref:SRB&;s manual of surgery,ed 3,pg no 925."}{"text": "### Human: Ulipristal acetate is a\/an: ### Assistant: Ans. is d, i.e. Selective progesterone receptor modulatorRef: Harrison's Principles of Internal medicine 19th\/ed, p2391Ulipristal acetate (trade name Ella One in the European Union, Ella in the US for contraception, and Esmya for uterine fibroid) is a selective progesterone receptor modulator (SPRM).PharmacodynamicsAs an SPRM, ulipristal acetate has partial agonistic as well as antagonistic effects on the progesterone receptor. It also binds to the glucocorticoid receptor, but is only a weak anti-glucocorticoid relative to mifepristone, and has no relevant affinity to the estrogen, androgen and mineralocorticoid receptors. Phase II clinical trials suggest that the mechanism might consist of blocking or delaying ovulation and of delaying the maturation of the endometrium.Medical Uses* Emergency contraception: For emergency contraception, a 30mg tablet is used within 120 hours (5 days) after an unprotected intercourse or contraceptive failure. It has been shown to prevent about 60% of expected, pregnancies, and prevents more pregnancies than emergency contraception with levonorgestrel.* Treatment of uterine fibroids: Ulipristal acetate is used for preoperative treatment of moderate to severe symptoms of uterine fibroids in adult women of reproductive age in a daily dose of a 5mg tablet. Treatment of uterine fibroids with ulipristal acetate for 13 weeks effectively controlled excessive bleeding due to uterine fibroids and reduced the size of the fibroids.InteractionsUlipristal acetate is metabolized by CYP3A4 in vitro. Ulipristal acetate is likely to interact with substrates of CYP3A4, like rifampicin, phenytoin, St John's wort, carbamazepine or ritonavir. Therefore, concomitant use with these agents is not recommended. It might also interact with hormonal contraceptives and progestogens such as levonorgestrel and other substrates of the progesterone receptor, as well as with glucocorticoids.Adverse EffectsCommon side effects include abdominal pain and temporary menstrual irregularity or disruption. Headache and nausea were observed under long-term administration (12 weeks), but not after a single dose.ContraindicationsUlipristal acetate should not be taken by women with severe liver diseases because of its CYP-mediated metabolism. It has not been studied in women under the age of 18.Pregnancy: Unlike levonorgestrel, and like mifepristone, ulipristal acetate is embryotoxic in animal studies. Before taking the drug, a pregnancy must be excluded."}{"text": "### Human: A patient presented with ahritis and purpura. Laboratory examination showed monoclonal and polyclonal cryoglobulins. Histopathology showed deposits of cryoglobulins around the vessels. The patient should be tested for which of the following ? ### Assistant: Hepatitis C This is caused by an RNA flavivirus. Acute symptomatic infection with hepatitis C is rare. Most individuals are unaware of when they became infected and are identified only when they develop chronic liver disease. Eighty per cent of individuals exposed to the virus become chronically infected and late spontaneous viral clearance is rare. There is no active or passive protection against hepatitis C virus (HCV). Hepatitis C infection is usually identified in asymptomatic individuals screened because they have risk factors for infection, such as previous injecting drug use , or have incidentally been found to have abnormal liver blood tests. Although most people remain asymptomatic until progression to cirrhosis occurs, fatigue can complicate chronic infection and is unrelated to the degree of liver damage. Hepatitis C is the most common cause of what used to be known as 'non-A, non-B hepatitis'. If hepatitis C infection is left untreated, progression from chronic hepatitis to cirrhosis occurs over 20-40 years. Risk factors for progression include male gender, immunosuppression (such as co-infection with HIV), prothrombotic states and heavy alcohol misuse. Not everyone with hepatitis C infection will necessarily develop cirrhosis but approximately 20% do so within 20 years. Once cirrhosis has developed, the 5- and 10-year survival rates are 95% and 81%, respectively. One-quaer of people with cirrhosis will develop complications within 10 years and, once complications such as ascites develop, the 5-year survival is around 50%. Once cirrhosis is present, 2-5% per year will develop primary hepatocellular carcinoma. Ref Davidson edition23rd pg877"}{"text": "### Human: All of the following structures passes through the Alcock canal, EXCEPT: ### Assistant: Alcock canal or pudendal canal stas from the lesser sciatic notch and runs forward on the medial surface of the ischial tuberosity up to the pubic arch where it is continuous with the deep perineal pouch. Contents of the pudendal canal are:Pudendal nervePudendal aery and veinWithin the canal pudendal nerve give rise to following branches:Perineal nerveDorsal nerve of penis or clitorisRef: Hoffman B.L., Schorge J.O., Schaffer J.I., Halvorson L.M., Bradshaw K.D., Cunningham F.G., Calver L.E. (2012). Chapter 38. Anatomy."}{"text": "### Human: Marfans Syndrome affecting eyes, skeletal system, and the cardiovascular system is caued by the mutation in the gene ### Assistant: Marfan Syndrome Is Caused by Mutations in the Gene for Fibrillin-1. It is inherited as an autosomal dominant trait. It affects the eyes (eg, causing dislocation of the lens, known as ectopia lentis), the skeletal system (most patients are tall and exhibit long digits and hyperextensibility of the joints), and the cardiovascular system (eg, causing weakness of the aoic media, leading to dilation of the ascending aoa). Abraham Lincoln may have had this condition. Most cases are caused by mutations in the gene (on chromosome 15) for fibrillin-1.Mutations in the fibrillin-1 gene have also been identified as the cause of acromicric dysplasia and geleophysic dysplasia, which are characterized by sho stature, skin thickening, and stiff joints. Congenital contractural arachnodactyly is associated with a mutation in the gene for fibrillin-2.Ref: Harper&;s Biochemistry; 30th edition; Chapter 50; The Extracellular Matrix"}{"text": "### Human: RNA primer is synthesized by - ### Assistant: Ans. is 'c' i.e., DNA primase ProteinFunctionDNA polymerasesDeoxynucleotide polymerizationHelicasesProcessive unwinding of DNATopoisomerasesRelieve torsional strain that results from helicase-induced unwindingDNA primaseInitiates synthesis of RNA primersSingle-strand binding proteinsPrevent premature reannealling of dsDNADNA ligaseSeals the single strand nick between the nascent chain and Okazaki fragments on lagging strand"}{"text": "### Human: Biochemical analytes measured in the triple test are all the following Except ### Assistant: Second-Trimester ScreeningPregnancies with fetal Down syndrome are characterized by lower maternal serum AFP levels--approximately 0.7 MoM, higher hCG levels--approximately 2.0 MoM, and lower unconjugated estriol levels--approximately 0.8 MoM. This triple test can detect 61 to 70 percent of Down syndrome.Levels of all three markers are decreased in the setting of trisomy 18.Ref: William&;s Obstetrics; 24th edition; Chapter 14"}{"text": "### Human: Yoke muscle pair is ### Assistant: Right lateral rectus and left medial rectus.\nYolk muscles are pair of muscles one from eye which contracts simultaneously during version movement."}{"text": "### Human: Causes of restrictive cardiomyopathy are - ### Assistant: Restrictive cardiomyopathy is characterised by a primary decrease in ventricular compliance , resulting in impaired ventricular filling during diastole. Restrictive cardiomyopathy can be idiopathic or associated with systemic disease that also happen to affect the myocardium for example radiation fibrosis, amyloidosis, sarcoidosis, or products of inborn errors of metabolism Robbins 9 th edition page no. 401"}{"text": "### Human: 3-year-old child comes in ER with Hv'o vomiting, loose watery motion for 3 days, on examination, the child was drowsy, sunken eye. Hypothermia and skin pinch take time to revert back, diagnosis ### Assistant: on \u00a0A child with severe dehydration will have at least two of the following four signs: sensorium is abnormally sleepy or lethargic, sunken eyes, drinking poorly or not at all, and a very slow skin pinch.\nA child with some signs of dehydraon will have two of the following: restlessness or irritability, sunken eyes, drinking eagerly or slow skin pinch.\nA child with either one or none of these signs is classified as having no signs of death"}{"text": "### Human: A 50 years old lady presented with lump in the left breast, which has developed suddenly in weeks. Perimenstrual symptoms are present. No associated family history. On examination, the lump is well circumscribed, fluctuant, 1.5 cm oval in shape. Most likely diagnosis: ### Assistant: Fibroadenoma - it occurs in 2nd to 3rd decade of life Galactocele - it occurs in 3rd to 4th decade of life. Breast cancer - there is no associated family history and lump has developed suddenly hence breast cancer is ruled out. Breast cyst - it occurs in 5th to 6th decade of life. Examination revealed fluctuant lesion which point towards breast cyst. - Often multiple, may be bilateral & can mimic malignancy. Typically present suddenly - Diagnosis can be confirmed by aspiration and \/ or ultrasound."}{"text": "### Human: HIV can - ### Assistant: it can cross blood brain barrier and cause cns infection REF:ANANTHANARAYANAN TEXT BOOK OF MICROBIOLOGY 9EDITION PGNO.576"}{"text": "### Human: Heamorrhagic external otitis media is caused by ### Assistant: Otitis externa haemorrhagica It is characterized by for mation of haemorrhagic bullae on the tympanic membrane and deep meatus. It is probably viral in origin and may be seen in influenza epidemics. The condition causes severe pain in the ear and blood-stained discharge when the bullae rupture. Treatment: with analgesics is directed to give relief from pain. Antibiotics are given for secondary infection of the ear canal, or middle ear if the bulla has ruptured into the middle ear. Ref:- Dhingra; pg num:-52"}{"text": "### Human: A 7 year old male patient presents to the clinic for routine dental examination and has history of frequent snacking and sugary beverages. Intraoral examination shows multiple pit and fissure caries and subsurface enamel porosity manifesting as milky white opacity. Tactile probing should not be used in this case because of all of the following EXCEPT? ### Assistant: Traditionally, dentists have relied upon a visual-tactile radiographic\u00a0 procedure\u00a0 for\u00a0 the detection\u00a0 of\u00a0 dental caries. This procedure involves the visual identification of demineralized\u00a0 areas\u00a0 (typically\u00a0 white\u00a0 spots)\u00a0 or\u00a0 suspicious pits or fissures and the use of the dental explorer to determine the\u00a0 presence of a\u00a0 loss of continuity\u00a0 or breaks in the enamel and to assess the softness or resilience of the enamel. Caries lesions located on interproximal tooth surfaces\u00a0 have\u00a0 generally\u00a0 been\u00a0 detected\u00a0 with\u00a0 the\u00a0 use\u00a0 of\u00a0 bitewing radiographs. These procedures have been used routinely in virtually every dental office in the United States for the past 50 years. Tactile probing with an explorer is no longer used for caries detection in most European countries, and this protocol has now been adopted by many U.S. dental schools. The primary concerns that led to the discontinuation of the probing procedure were as follows:\n1. The\u00a0 insertion\u00a0 of\u00a0 the\u00a0 explorer\u00a0 into\u00a0 the\u00a0 suspected lesion inevitably disrupts the surface layer covering very early lesions, thereby eliminating the possibility for remineralization of the decalcified area;\u00a0\n2. The probing of lesions and suspected lesions results in the transport of\u00a0 cariogenic\u00a0 bacteria\u00a0 from\u00a0 one area\u00a0 to another;\u00a0 and\u00a0\u00a0\n3. Frank lesions requiring restoration are generally apparent visually\u00a0 without\u00a0 the\u00a0 need\u00a0 for\u00a0 probing."}{"text": "### Human: Most dangerous paicles causing pneumoconiosis are of size ### Assistant: In pneumoconiosis, the most dangerous paicles range from 1-5 micron in diameter, because they may reach the terminal small airway and settle in there lining notes The solubility and cytotoxicity of paicles ,modify the nature of pulmonary response"}{"text": "### Human: Calculate the deficit for a 60 kg person,with Hb - 5 g\/dl add 1000 mg for iron stores. ### Assistant: Iron deficit = Bodyweight(kg) x 2.3 x (15 - Hb) + 1000\n\u00a0 \u00a0 \u00a0 = 60 x 2.3 x (15 - 5) + 1000\n\u00a0 \u00a0 \u00a0 = 2380 (close to 2500)"}{"text": "### Human: Which of the following is not a cause of glomerular proteinuria? ### Assistant: Multiple myeloma is characterized by tubular proteinuria. The Bence Jones proteins induce tubular damage Increased beta-2-microglobulin levels in urine is an prognostic indicator of Multiple myeloma Diabetes Mellitus and amyloidosis lead to damage to basement membrane of the kidney leading to glomerular proteinuria in form of albuminuria Nil Lesion is also called as minimal change disease and presents as glomerular cause of proteinuria leading to nephrotic presentation . Multiple Myeloma: SPEP will demonstrate a sharp peak (\"church spire\" orM band) in the gamma globulin region as shown below:"}{"text": "### Human: Which of the following does not cause Insulin release- ### Assistant: Ans. is 'a' i.e., Rosiglitazone o Oral hypoglycemic drugs may be divided into two groups. 1. Group 1 These drugs reduce plasma glucose by stimulating insulin production, therefore called insulin secretogogues. Hypoglycemia is a well known side effect. Examples are: i) Sulfonylureas: first generation (chlorpropamide, tobutamide); second generation (Glimipiride, glyburide, glipizide, gliclazide). ii) Megalitinnide\/D-phenylalanine analogues: Nateglinide, Rapaglinide. 2. Group 2 These drugs reduce blood glucose without stimulating insulin production, therefore are insulin nonsecretogogues. These durgs do not cuase hypoglycemia when used alone and can cause hypoglycemia, only when used with other oral hypoglycemics. Examples are: i) Biguanides: Metformin, Phenformin ii) Thiazolidinediones: Rosiglitazone, Pioglitazone, Troglitazone. iii) alpha-glucosidase inhibitors: Acarbose, miglital."}{"text": "### Human: Which of the following endocrine tumors is most commonly seen in MEN I ? ### Assistant: In MEN 1 : * Gastrinoma (>50%) * Insulinoma (10-30%) * Glucagonoma (<3%) * Somatotrophinoma (25%) Reference : page 2336 Harrison's Principles of Internal Medicine 19th edition"}{"text": "### Human: The following is the least useful investigation in multiple myeloma : ### Assistant: Answer is C (Bone scan) Bone scans are least useful for diagnosis of Multiple Myeloma. Cytokines secreted by myeloma cells suppress osteoblastic activity therefore typically no increased uptake is observed. In multiple myeloma the osteohlastic response to bone destruction is negligible. The bone scan therefore is often normal or may show areas of decreased uptake (photopenia).It has been found to be less useful in diagnosis than plain radiography. Ig protein \/24 hour is a major ciriteria for diagnosis \" align=\"left\" height=\"133\" width=\"100\"> Note : Serum M component is most commonly IgG Q (IgG (53%), IgA (25%), IgD (I%) Bence Jones proteins are light chain protein only Q."}